Appendix F Solutions to Exercises
1 Limits
1.1 Drawing Tangents and a First Limit
1.1.2 Exercises
1.1.2.2.
Solution.
- True: since \(y=2x+3\) is the tangent line to \(y=f(x)\) at the point \(x=2\text{,}\) this means the function and the tangent line have the same value at \(x=2\text{.}\) So \(f(2)=2(2)+3=7\text{.}\)
- In general, this is false. We are only guaranteed that the curve \(y=f(x)\) and its tangent line \(y=2x+3\) agree at \(x=2\text{.}\) The functions \(f(x)\) and \(2x+3\) may or may not take the same values when \(x\ne 2\text{.}\) For example, if \(f(x)=2x+3\text{,}\) then of course \(f(x)\) and \(2x+3\) agree for all values of \(x\text{.}\) But if \(f(x) = 2x+3 +(x-2)^2\text{,}\) then \(f(x)\) and \(2x+3\) agree only for \(x=2\text{.}\)
1.1.2.3.
Solution.
1.2 Another Limit and Computing Velocity
1.2.2 Exercises
1.2.2.1.
Solution.
1.2.2.2.
Solution.
1.2.2.3.
Solution.
1.2.2.4.
Solution.
1.2.2.5.
Solution.
1.2.2.6.
Solution.
1.2.2.7.
Solution.
- Average velocity:\begin{align*} \dfrac{\mbox{change in position}}{\mbox{change in time}} \amp= \dfrac{s(5)-s(3)}{5-3} = \dfrac{(3\cdot 5^2+5)-(3\cdot 3^2+5)}{5-3}\\ \amp=24 \text{ units per second}. \end{align*}
- From the notes, we know the velocity of an object at time \(a\) is\begin{equation*} v(a)=\lim_{h \rightarrow 0}\frac{s(a+h)-s(a)}{h} \end{equation*}So, in our case:\begin{align*} v(1)\amp=\lim_{h \rightarrow 0}\frac{s(1+h)-s(1)}{h} =\lim_{h \rightarrow 0}\frac{[3(1+h)^2+5]-[3(1)^2+5]}{h} \\ \amp=\lim_{h \rightarrow 0}\frac{6h+3h^2}{h} =\lim_{h \rightarrow 0}6+3h = 6 \end{align*}So the velocity when \(t=1\) is 6 units per second.
1.2.2.8.
Solution.
- Average velocity:\begin{align*} \dfrac{\text{change in position}}{\text{change in time}} \amp= \dfrac{s(9)-s(1)}{9-1} = \dfrac{3-1}{9-1}\\ \amp=\dfrac{1}{4}\text{ units per second}. \end{align*}
- From the notes, we know the velocity of an object at time \(a\) is\begin{equation*} v(a)=\lim_{h \rightarrow 0}\frac{s(a+h)-s(a)}{h} \end{equation*}So, in our case:\begin{align*} v(1)\amp=\lim_{h \rightarrow 0}\frac{s(1+h)-s(1)}{h} =\lim_{h \rightarrow 0}\frac{\sqrt{1+h}-1}{h}\\ &=\lim_{h \rightarrow 0}\frac{\sqrt{1+h}-1}{h}\cdot \left( \dfrac{\sqrt{1+h}+1}{\sqrt{1+h}+1}\right)\\ &=\lim_{h \rightarrow 0}\frac{(1+h)-1}{h(\sqrt{1+h}+1)}\\ &=\lim_{h \rightarrow 0}\frac{1}{\sqrt{1+h}+1}=\frac{1}{2}\mbox{ units per second} \end{align*}
- \begin{align*} v(9)\amp=\lim_{h \rightarrow 0}\frac{s(9+h)-s(9)}{h} =\lim_{h \rightarrow 0}\frac{\sqrt{9+h}-3}{h}\\ &=\lim_{h \rightarrow 0}\frac{\sqrt{9+h}-3}{h}\cdot \left( \dfrac{\sqrt{9+h}+3}{\sqrt{9+h}+3}\right)\\ &=\lim_{h \rightarrow 0}\frac{(9+h)-9}{h(\sqrt{9+h}+3)}\\ &=\lim_{h \rightarrow 0}\frac{1}{\sqrt{9+h}+3}=\frac{1}{6}\mbox{ units per second} \end{align*}
1.3 The Limit of a Function
1.3.2 Exercises
1.3.2.1.
Solution.
- \(\displaystyle \lim_{x \rightarrow -2} f(x)=1\text{:}\) as \(x\) gets very close to \(-2\text{,}\) \(y\) gets very close to \(1\text{.}\)
- \(\displaystyle \lim_{x \rightarrow 0}f(x)=0\text{:}\) as \(x\) gets very close to \(0\text{,}\) \(y\) also gets very close to \(0\text{.}\)
- \(\displaystyle \lim_{x \rightarrow 2}f(x)=2\text{:}\) as \(x\) gets very close to \(2\text{,}\) \(y\) gets very close to \(2\text{.}\) We ignore the value of the function where \(x\) is exactly \(2\text{.}\)
1.3.2.2.
Solution.
1.3.2.3.
Solution.
- \(\displaystyle \lim_{x \rightarrow -1^{-}} f(x)=2\text{:}\) as \(x\) approaches \(-1\) from the left, \(y\) approaches 2. It doesn’t matter that the function isn’t defined at \(x=-1\text{,}\) and it doesn’t matter what happens to the right of \(x=-1\text{.}\)
- \(\displaystyle \lim_{x \rightarrow -1^{+}} f(x)=-2\text{:}\) as \(x\) approaches \(-1\) from the right, \(y\) approaches -2. It doesn’t matter that the function isn’t defined at \(-1\text{,}\) and it doesn’t matter what happens to the left of \(-1\text{.}\)
- \(\displaystyle \lim_{x \rightarrow -1} f(x) =\) DNE: since the limits from the left and right don’t agree, the limit does not exist.
- \(\displaystyle \lim_{x \rightarrow -2^{+}} f(x) =0\text{:}\) as \(x\) approaches \(-2\) from the right, \(y\) approaches 0. It doesn’t matter that the function isn’t defined at 2, or to the left of 2.
- \(\displaystyle \lim_{x \rightarrow 2^{-}} f(x)=0\text{:}\) as \(x\) approaches \(2\) from the left, \(y\) approaches 0. It doesn’t matter that the function isn’t defined at 2, or to the right of 2.
1.3.2.4.
Solution.
1.3.2.5.
Solution.
1.3.2.6.
Solution.
1.3.2.7.
Solution.
1.3.2.8.
Solution.
1.3.2.9.
Solution.
1.3.2.10.
Solution.
1.3.2.11.
Solution.
1.3.2.12.
Solution.
1.3.2.13.
Solution.
1.3.2.14.
Solution.
1.3.2.15.
Solution.
1.3.2.16.
Solution.
1.3.2.17.
Solution.
1.4 Calculating Limits with Limit Laws
1.4.2 Exercises
1.4.2.1.
1.4.2.2.
Solution.
1.4.2.3.
Solution.
- As we saw in Question 1.4.2.2, \(x-3\) is a function with limit 0 at \(x=3\text{.}\) So one way of thinking about this question is to try choosing \(f(x)\) so that \(\frac{f(x)}{g(x)}=g(x)=x-3\) too, which leads us to the solution \(f(x)=(x-3)^2\) and \(g(x)=x-3\text{.}\) This is one of many, many possible answers.
- Another way of thinking about this problem is that \(f(x)\) should go to 0 “more strongly” than \(g(x)\) when \(x\) approaches \(3\text{.}\) One way of a function going to 0 really strongly is to make that function identically zero. So we can set \(f(x)=0\) and \(g(x)=x-3\text{.}\) Now \(\dfrac{f(x)}{g(x)}\) is equal to 0 whenever \(x \neq 3\text{,}\) and is undefined at \(x=3\text{.}\) Since the limit as \(x\) goes to three does not take into account the value of the function at 3, we have \(\displaystyle\lim_{x \rightarrow 3} \dfrac{f(x)}{g(x)}=0\text{.}\)
1.4.2.4.
Solution.
1.4.2.5.
Solution.
1.4.2.6.
Solution.
1.4.2.7.
Solution.
1.4.2.8.
Solution.
1.4.2.9. (✳).
Solution.
1.4.2.10. (✳).
Solution.
1.4.2.11. (✳).
Solution.
1.4.2.12. (✳).
Solution.
1.4.2.13. (✳).
Solution.
1.4.2.14. (✳).
Solution.
1.4.2.15. (✳).
Solution.
1.4.2.16. (✳).
Solution.
1.4.2.17. (✳).
Solution.
1.4.2.18. (✳).
Solution.
1.4.2.19.
Solution.
1.4.2.20. (✳).
Solution.
1.4.2.21. (✳).
Solution.
1.4.2.22. (✳).
Solution.
1.4.2.23. (✳).
Solution.
1.4.2.24. (✳).
Solution.
So there is a cancelation. Hence the limit is
\begin{align*} \lim_{t\to1}\frac{3t-3}{2 - \sqrt{5-t}} &=\lim_{t\to1} \left(2 + \sqrt{5-t}\right) \cdot 3\\ &= 12 \end{align*}1.4.2.25.
Solution.
1.4.2.26.
Solution.
1.4.2.27. (✳).
Solution.
-
Solution 1: We will evaluate separately the limit from the right and from the left.When \(x \gt 0\text{,}\)\begin{equation*} 0 \leq x\sin^2\left(\dfrac{1}{x}\right) \leq x \end{equation*}because \(0 \leq \sin^2\left(\dfrac{1}{x}\right) \leq 1\text{.}\) Since\begin{equation*} \ds\lim_{x \to 0^+} 0 = 0 \qquad\mbox{and}\qquad \ds\lim_{x \to 0^+}x=0 \end{equation*}then by the Squeeze Theorem, also\begin{equation*} \ds\lim_{x \to 0^+} x\sin^2\left(\dfrac{1}{x}\right)=0 . \end{equation*}Similarly, When \(x \lt 0\text{,}\)\begin{equation*} x \leq x\sin^2\left(\dfrac{1}{x}\right) \leq 0 \end{equation*}because \(0 \leq \sin^2\left(\dfrac{1}{x}\right) \leq 1\text{.}\) Since\begin{equation*} \ds\lim_{x \to 0^-} x = 0 \qquad\mbox{and}\qquad \ds\lim_{x \to 0^-}0=0 \end{equation*}then by the Squeeze Theorem, also\begin{equation*} \ds\lim_{x \to 0^-} x\sin^2\left(\dfrac{1}{x}\right)=0. \end{equation*}Since the one-sided limits are both equal to zero,\begin{equation*} \ds\lim_{x\to0}x\sin^2\left(\frac{1}{x}\right)=0. \end{equation*}Remark: this is a perfectly fine proof, but it seems to repeat itself. Since the cases \(x \lt 0\) and \(x \gt 0\) are so similar, we would like to take care of them together. This can be done as shown below.
- Solution 2: If \(x\neq 0\text{,}\) then \(0\leq\sin^2\left(\dfrac{1}{x}\right)\leq1\text{,}\) so\begin{equation*} -|x| \leq x\sin^2\left(\frac{1}{x}\right)\leq|x|. \end{equation*}Since\begin{equation*} \ds\lim_{x \to 0}-|x|=0\qquad\mbox{and}\qquad\lim_{x\to0}|x|=0 \end{equation*}then by the Squeeze Theorem, also\begin{equation*} \lim_{x \to 0} x\sin^2\left(\frac{1}{x}\right)=0. \end{equation*}
1.4.2.28.
Solution.
1.4.2.29.
Solution.
1.4.2.30.
Solution.
1.4.2.31.
Solution.
1.4.2.32.
Solution.
1.4.2.33.
Solution.
1.4.2.34.
Solution.
1.4.2.35. (✳).
Solution.
1.4.2.36. (✳).
Solution.
1.4.2.37.
Solution.
1.4.2.38.
Solution.
Since we cancelled out the term that was causing the numerator and denominator to be zero when \(t=\frac{1}{2}\text{,}\) now \(t=\frac{1}{2}\) is in the domain of our function, so we simply plug it in:
\begin{align*} &=\frac{1+1}{\frac{3}{4}\left(\frac{1}{4}-1\right)}\\ &=\frac{2}{\frac{3}{4}\left(-\frac{3}{4}\right)}\\ &=-\frac{32}{9} \end{align*}1.4.2.39.
Solution.
So,
\begin{align*} \frac{|x|}{x} &= \left\{\begin{array}{rcl} \frac{x}{x}&,&x \gt 0\\ \frac{-x}{x}&,&x \lt 0 \end{array}\right.\\ &= \left\{\begin{array}{rcl} 1&,&x \gt 0\\ -1&,&x \lt 0 \end{array}\right.\\ \end{align*}Therefore,
\begin{align*} 3+\frac{|x|}{x}&=\left\{\begin{array}{rcl} 4&,&x \gt 0\\ 2&,&x \lt 0 \end{array}\right. \end{align*}1.4.2.40.
Solution.
So, with \(X=d+4\text{,}\)
\begin{align*} 3\frac{|d+4|}{d+4} &= \left\{\begin{array}{lcl} 3\frac{d+4}{d+4}&,&d+4 \gt 0\\ &\\ 3\frac{-(d+4)}{d+4}&,&d+4 \lt 0 \end{array}\right.\\ &= \left\{\begin{array}{rcl} 3&,&d \gt -4\\ -3&,&d \lt -4 \end{array}\right. \end{align*}1.4.2.41.
Solution.
1.4.2.42.
Solution.
1.4.2.43. (✳).
Solution.
1.4.2.44.
Solution.
- \(\displaystyle\lim_{x \rightarrow 0} f(x)=0\text{:}\) as \(x\) approaches 0, so does \(2x\text{.}\)
- \(\displaystyle\lim_{x \rightarrow 0} g(x)=\) DNE: the left and right limits do not agree, so the limit does not exist. In particular: \(\displaystyle\lim_{x \rightarrow 0^-} g(x)=-\infty\) and \(\displaystyle\lim_{x \rightarrow 0^+} g(x)=\infty\text{.}\)
- \(\displaystyle\lim_{x \rightarrow 0} f(x)g(x)=\displaystyle\lim_{x \rightarrow 0} 2x\cdot\dfrac{1}{x}=\displaystyle\lim_{x \rightarrow 0} 2=2\text{.}\) Remark: although the limit of \(g(x)\) does not exist here, the limit of \(f(x)g(x)\) does.
- \(\displaystyle \displaystyle\lim_{x \rightarrow 0} \dfrac{f(x)}{g(x)}=\displaystyle\lim_{x \rightarrow 0} \dfrac{2x}{\frac{1}{x}}=\displaystyle\lim_{x \rightarrow 0} 2x^2=0\)
- \(\displaystyle \displaystyle\lim_{x \rightarrow 2} f(x)+g(x)=\displaystyle\lim_{x \rightarrow 2} 2x+\dfrac{1}{x}=4+\frac{1}{2}=\dfrac{9}{2}\)
- \(\displaystyle \displaystyle\lim_{x \rightarrow 0} \dfrac{f(x)+1}{g(x+1)}= \displaystyle\lim_{x \rightarrow 0} \dfrac{2x+1}{\frac{1}{x+1}}=\dfrac{1}{1}= 1\)
1.4.2.45.
Solution.
1.4.2.46.
Solution.
\(x\) | \(f(x)\) | \(g(x)\) | \(\dfrac{f(x)}{g(x)}\) |
\(-3\) | \(-3\) | \(-1.5\) | \(2\) |
\(-2\) | \(0\) | \(0\) | UND |
\(-1\) | \(3\) | \(1.5\) | \(2\) |
\(-0\) | \(3\) | \(1.5\) | \(2\) |
\(1\) | \(1.5\) | \(.75\) | \(2\) |
\(2\) | \(0\) | \(0\) | UND |
\(3\) | \(1\) | \(.5\) | \(2\) |
1.4.2.47.
Solution.
1.4.2.48.
Solution.
1.4.2.49.
Solution.
1.4.2.50.
Solution.
1.5 Limits at Infinity
1.5.2 Exercises
1.5.2.1.
Solution.
1.5.2.2.
Solution.
1.5.2.3.
Solution.
1.5.2.4.
Solution.
1.5.2.5.
Solution.
1.5.2.6.
Solution.
1.5.2.7.
Solution.
because
\begin{align*} \lim\limits_{x\rightarrow\infty} \left(1 -\frac{1}{3x^4} -\frac{100}{3 x^3}\right) &=1-0-0=1. \end{align*}1.5.2.8.
Solution.
1.5.2.9. (✳).
Solution.
Now we divide the numerator and denominator by \(x\text{.}\) In the case of the denominator, since \(x \gt 0\text{,}\) \(x=\sqrt{x^2}\text{.}\)
\begin{align*} &\hskip0.25in= \lim_{x\rightarrow \infty}\dfrac{6(x)}{\sqrt{x^2}\sqrt{1+\frac{5}{x}}+\sqrt{x^2}\sqrt{1-\frac{1}{x}}}\\ &\hskip0.25in= \lim_{x\rightarrow \infty}\dfrac{6(x)}{(x)\sqrt{1+\frac{5}{x}}+(x)\sqrt{1-\frac{1}{x}}}\\ & \hskip0.25in= \lim_{x\rightarrow \infty}\dfrac{6}{\sqrt{1+\frac{5}{x}}+\sqrt{1-\frac{1}{x}}}\\ &\hskip0.25in=\frac{6}{\sqrt{1+0}+\sqrt{1-0}}= 3 \end{align*}1.5.2.10. (✳).
Solution.
1.5.2.11. (✳).
Solution.
1.5.2.12. (✳).
Solution.
1.5.2.13. (✳).
Solution.
1.5.2.14. (✳).
Solution.
1.5.2.15. (✳).
Solution.
1.5.2.16.
Solution.
-
Solution 1We want to factor out \(x\text{,}\) the highest power in the denominator. Since our limit only sees negative values of \(x\text{,}\) we must remember that \(\sqrt[4]{x^4}=|x|=-x\text{,}\) although \(\sqrt[3]{x^3}=x\text{.}\)\begin{align*} \lim_{x \rightarrow -\infty}\dfrac{\sqrt[3]{x^2+x}-\sqrt[4]{x^4+5}}{x+1}&= \lim_{x \rightarrow -\infty}\dfrac{\sqrt[3]{x^3(\frac{1}{x}+\frac{1}{x^2})}-\sqrt[4]{x^4(1+\frac{5}{x^4})}}{x(1+\frac{1}{x})}\\ &= \lim_{x \rightarrow -\infty}\dfrac{\sqrt[3]{x^3}\sqrt[3]{\frac{1}{x}+\frac{1}{x^2}}-\sqrt[4]{x^4}\sqrt[4]{1+\frac{5}{x^4}}}{x(1+\frac{1}{x})}\\ &=\lim_{x \rightarrow -\infty}\dfrac{x\sqrt[3]{\frac{1}{x}+\frac{1}{x^2}}-(-x)\sqrt[4]{1+\frac{5}{x^4}}}{x(1+\frac{1}{x})}\\ &=\lim_{x \rightarrow -\infty}\dfrac{\sqrt[3]{\frac{1}{x}+\frac{1}{x^2}}+\sqrt[4]{1+\frac{5}{x^4}}}{1+\frac{1}{x}}\\ &=\frac{\sqrt[3]{0+0}+\sqrt[4]{1+0}}{1+0}=1 \end{align*}
-
Solution 2Alternately, we can use the transformation \(\displaystyle\lim_{x \rightarrow -\infty} f(x)=\displaystyle\lim_{x \rightarrow \infty} f(-x)\text{.}\) Then we only look at positive values of \(x\text{,}\) so roots behave nicely: \(\sqrt[4]{x^4}=|x|=x\text{.}\)\begin{align*} \lim_{x \rightarrow -\infty}\dfrac{\sqrt[3]{x^2+x}-\sqrt[4]{x^4+5}}{x+1}&= \lim_{x \rightarrow \infty}\dfrac{\sqrt[3]{(-x)^2-x}-\sqrt[4]{(-x)^4+5}}{-x+1}\\ &= \lim_{x \rightarrow \infty}\dfrac{\sqrt[3]{x^2-x}-\sqrt[4]{x^4+5}}{-x+1}\\ &= \lim_{x \rightarrow \infty}\dfrac{\sqrt[3]{x^3}\sqrt[3]{\frac{1}{x}-\frac{1}{x^2}}-\sqrt[4]{x^4}\sqrt[4]{1+\frac{5}{x^4}}}{x(-1+\frac{1}{x})}\\ &= \lim_{x \rightarrow \infty}\dfrac{x\sqrt[3]{\frac{1}{x}-\frac{1}{x^2}}-x\sqrt[4]{1+\frac{5}{x^4}}}{x(-1+\frac{1}{x})}\\ &= \lim_{x \rightarrow \infty}\dfrac{\sqrt[3]{\frac{1}{x}-\frac{1}{x^2}}-\sqrt[4]{1+\frac{5}{x^4}}}{-1+\frac{1}{x}}\\ &=\frac{\sqrt[3]{0-0}-\sqrt[4]{1+0}}{-1+0}=\frac{-1}{-1}=1 \end{align*}
1.5.2.17. (✳).
Solution.
1.5.2.18.
Solution.
1.5.2.19.
Solution.
1.5.2.20. (✳).
Solution.
1.5.2.21. (✳).
Solution.
1.5.2.22. (✳).
Solution.
1.5.2.23.
Solution.
1.5.2.24. (✳).
Solution.
Since \(n \gt 0\text{,}\) we can simplify \(\sqrt{n^2}=n\text{.}\)
\begin{align*} &=\lim_{n\rightarrow\infty}\frac{5\cdot n}{n\sqrt{1+\frac{5}{n}}+n}\\ &=\lim_{n\rightarrow\infty}\frac{5}{\sqrt{1+\frac{5}{n}}+1}\\ &=\frac{5}{\sqrt{1+0}+1}=\frac{5}{2} \end{align*}1.5.2.25.
Solution.
-
Solution 1:When \(a\) approaches 0 from the right, the numerator approaches negative infinity, and the denominator approaches \(-1\text{.}\) So, \(\ds\lim_{a \to 0^+}\dfrac{a^2-\frac{1}{a}}{a-1}=\infty\text{.}\)More precisely, using Theorem 1.5.9:\begin{align*} &\lim_{a \to 0^+} \frac{1}{a}=+\infty\\ \mbox{Also,}& \lim_{a \to 0^+} a^2=0 \end{align*}So, using Theorem 1.5.9,\begin{align*} & \lim_{a \to 0^+} a^2-\frac{1}{a}=-\infty\\ \text{Furthermore,}&\lim_{a \to 0^+}a-1=-1\\ \text{So, using our theorem,}&\lim_{a \to 0^+}\frac{a^2-\frac{1}{a}}{a-1}=\infty \end{align*}
-
Solution 2:Since \(a=0\) is not in the domain of our function, a reasonable impulse is to simplify.\begin{align*} \frac{a^2-\frac{1}{a}}{a-1}\left(\frac{a}{a}\right)&=\frac{a^3-1}{a(a-1)}=\frac{(a-1)(a^2+a+1)}{a(a-1)}\\ \end{align*}
So,
\begin{align*} \lim_{a \to 0^+}\frac{a^2-\frac{1}{a}}{a-1}&= \lim_{a \to 0^+}\frac{(a-1)(a^2+a+1)}{a(a-1)}\\ &=\lim_{a \to 0^+}\frac{a^2+a+1}{a}\\ &=\lim_{a \to 0^+}a+1+\frac{1}{a}=\infty \end{align*}
1.5.2.26.
Solution.
1.5.2.27.
Solution.
1.5.2.28.
Solution.
1.6 Continuity
1.6.4 Exercises
1.6.4.1.
Solution.
1.6.4.2.
Solution.
1.6.4.3.
Solution.
1.6.4.4.
Solution.
1.6.4.5.
Solution.
1.6.4.6.
Solution.
1.6.4.7.
Solution.
1.6.4.8.
Solution.
1.6.4.9.
Solution.
1.6.4.10.
Solution.
1.6.4.11.
Solution.
1.6.4.12.
Solution.
1.6.4.13. (✳).
Solution.
1.6.4.14. (✳).
Solution.
1.6.4.15. (✳).
Solution.
1.6.4.16. (✳).
Solution.
1.6.4.17. (✳).
Solution.
1.6.4.18. (✳).
Solution.
1.6.4.19. (✳).
Solution.
1.6.4.20. (✳).
Solution.
1.6.4.21.
Solution.
1.6.4.22. (✳).
Solution.
1.6.4.23. (✳).
Solution.
1.6.4.24. (✳).
Solution.
1.6.4.25. (✳).
Solution.
1.6.4.26.
Solution.
1.6.4.27.
Solution.
- Let’s start with integers. \(1^3 \lt 7 \lt 2^3\text{,}\) so \(\sqrt[3]{7}\) is in the interval \((1,2)\text{.}\)
- Let’s narrow this down, say by testing \(f(1.5)\text{.}\) \((1.5)^3 = 3.375 \lt 7\text{,}\) so \(\sqrt[3]{7}\) is in the interval \((1.5,2)\text{.}\)
- Let’s narrow further, say by testing \(f(1.75)\text{.}\) \((1.75)^3\approx 5.34 \lt 7\text{,}\) so \(\sqrt[3]{7}\) is in the interval \((1.75,2)\text{.}\)
- Testing various points, we find \(f(1.9) \lt 7 \lt f(2)\text{,}\) so \(\sqrt[3]{7}\) is between \(1.9\) and \(2\text{.}\)
- By testing more, we find \(f(1.91) \lt 7 \lt f(1.92)\text{,}\) so \(\sqrt[3]{7}\) is in \((1.91,1.92)\text{.}\)
- In order to get an approximation for \(\sqrt[3]{7}\) that is rounded to two decimal places, we have to know whether \(\sqrt[3]{7}\) is greater or less than \(1.915\text{;}\) indeed \(f(1.915) \approx 7.02 \gt 7\text{,}\) so \(\sqrt[3]{7} \lt 1.915\text{;}\) then rounded to two decimal places, \(\sqrt[3]{7}\approx 1.91\text{.}\)
1.6.4.28.
Solution.
- If \(f(a)=g(a)\text{,}\) or \(f(b)=g(b)\text{,}\) then we simply take \(c=a\) or \(c=b\text{.}\)
- Suppose \(f(a) \neq g(a)\) and \(f(b) \neq g(b)\text{.}\) Then \(f(a) \lt g(a)\) and \(g(b) \lt f(b)\text{,}\) so if we define \(h(x)=f(x)-g(x)\text{,}\) then \(h(a) \lt 0\) and \(h(b) \gt 0\text{.}\) Since \(h\) is the difference of two functions that are continuous over \([a,b]\text{,}\) also \(h\) is continuous over \([a,b]\text{.}\) So, by the Intermediate Value Theorem, there exists some \(c \in (a,b)\) with \(h(c)=0\text{;}\) that is, \(f(c)=g(c)\text{.}\)
2 Derivatives
2.1 Revisiting Tangent Lines
2.1.2 Exercises
2.1.2.1.
Solution.
2.1.2.2.
Solution.
2.1.2.3.
Solution.
2.1.2.4.
Solution.
2.1.2.5.
Solution.
2.1.2.6.
Solution.
2.2 Definition of the Derivative
2.2.4 Exercises
2.2.4.1.
Solution.
2.2.4.2.
Solution.
2.2.4.3.
Solution.
2.2.4.4. (✳).
Solution.
2.2.4.5.
Solution.
2.2.4.6.
Solution.
2.2.4.7.
Solution.
In particular, this limit does not exist. Since the one-sided limit does not exist,
\begin{align*} \lim_{h \to 0}\frac{f(0+h)-f(0)}{h}&=DNE \end{align*}2.2.4.8.
Solution.
2.2.4.9.
Solution.
2.2.4.10.
Solution.
2.2.4.11. (✳).
Solution.
2.2.4.12. (✳).
Solution.
2.2.4.13. (✳).
Solution.
2.2.4.14.
Solution.
2.2.4.15. (✳).
Solution.
2.2.4.16. (✳).
Solution.
2.2.4.17. (✳).
Solution.
2.2.4.18.
Solution.
2.2.4.19. (✳).
Solution.
2.2.4.20. (✳).
Solution.
2.2.4.21. (✳).
Solution.
2.2.4.22. (✳).
Solution.
2.2.4.23.
Solution.
2.2.4.24.
Solution.
2.2.4.25.
Solution.
2.2.4.26. (✳).
Solution.
So, if \((1,-3)\) is on the tangent line, then
\begin{align*} -3&=(2\alpha)(1)-\alpha^2\\ \iff\qquad 0&=\alpha^2-2\alpha-3\\ \iff\qquad 0&=(\alpha-3)(\alpha+1)\\ \iff\qquad \alpha&=3, \quad\mbox{or}\qquad \alpha=-1.\\ \end{align*}So, the tangent lines \(y=(2\alpha)x-\alpha^2\) are
\begin{align*} y&=6x-9 \quad\mbox{and}\quad y=-2x-1. \end{align*}2.2.4.27. (✳).
Solution.
exists. In particular, this means \(f\) is differentiable at \(0\) if and only if both one-sided limits exist and are equal to each other.
\begin{align*} \\ \end{align*}When \(h \lt 0\text{,}\) \(f(h)=0\text{,}\) so
\begin{align*} \lim_{h \to 0^-}\frac{f(h)-f(0)}{h}&=\lim_{h \to 0^-}\frac{0-0}{h}=0\\ \end{align*}So, \(f\) is differentiable at \(x=0\) if and only if
\begin{align*} \lim_{h \to 0^+}\frac{f(h)-f(0)}{h}&=0.\\ \end{align*}To evaluate the limit above, we note \(f(0)=0\) and, when \(h \gt 0\text{,}\) \(f(h)=h^a\sin\left(\frac{1}{h}\right)\text{,}\) so
\begin{align*} \lim_{h \to 0^+}\frac{f(h)-f(0)}{h}&=\lim_{h \to 0^+}\frac{h^a\sin\left(\frac{1}{h}\right)}{h}\\ &=\lim_{h \to 0^+}h^{a-1}\sin\left(\frac{1}{h}\right) \end{align*}-
If \(a=1\text{,}\) then \(a-1=0\text{,}\) so \(h^{a-1}=h^0=1\) for all values of \(h\text{.}\) Then\begin{equation*} \lim_{h \to 0^+}h^{a-1}\sin\left(\frac{1}{h}\right)=\lim_{h \to 0^+}\sin\left(\frac{1}{h}\right)=DNE \end{equation*}(Recall that the function \(\sin\left(\frac{1}{x}\right)\) oscillates faster and faster as \(x\) goes to 0. We first saw this behaviour in Example 1.3.5.)
-
If \(a \lt 1\text{,}\) then \(a-1 \lt 0\text{,}\) so \(\ds\lim_{h \to 0^+}h^{a-1}=\infty\text{.}\) (Since we have a negative exponent, we are in effect dividing by a smaller and smaller positive number. For example, if \(a=\frac{1}{2}\text{,}\) then \(\ds\lim_{h \to 0^+}h^{a-1}=\ds\lim_{h \to 0^+}h^{-\frac{1}{2}}=\ds\lim_{h \to 0^+}\frac{1}{\sqrt{h}}=\infty\text{.}\)) Since \(\sin\left(\frac{1}{x}\right)\) goes back and forth between one and negative one,\begin{equation*} \lim_{h \to 0^+}h^{a-1}\sin\left(\frac{1}{x}\right)=DNE \end{equation*}since as \(h\) goes to 0, the function oscillates between positive and negative numbers of ever-increasing magnitude.
-
If \(a \gt 1\text{,}\) then \(a-1 \gt 0\text{,}\) so \(\ds\lim_{h \to 0^+}h^{a-1}=0\text{.}\) Although \(\sin\left(\frac{1}{x}\right)\) oscillates wildly near \(x=0\text{,}\) it is bounded by \(-1\) and \(1\text{.}\) So,\begin{equation*} (-1)h^{a-1} \leq h^{a-1}\sin\left(\frac{1}{h}\right) \leq h^{a-1} \end{equation*}Since both \(\ds\lim_{h \to 0^+} (-1)h^{a-1}=0\) and \(\ds\lim_{h \to 0^+} h^{a-1}=0\text{,}\) by the Squeeze Theorem,\begin{equation*} \ds\lim_{h \to 0^+} h^{a-1}\sin\left(\frac{1}{x}\right)=0 \end{equation*}as well.
2.3 Interpretations of the Derivative
2.3.3 Exercises
2.3.3.1.
Solution.
2.3.3.2.
Solution.
2.3.3.3.
Solution.
2.3.3.4.
Solution.
2.3.3.5.
Solution.
2.3.3.6.
Solution.
2.3.3.7.
Solution.
2.3.3.8.
Solution.
2.3.3.9.
Solution.
2.4 Arithmetic of Derivatives - a Differentiation Toolbox
2.4.2 Exercises
2.4.2.1.
Solution.
2.4.2.2.
Solution.
2.4.2.3.
Solution.
2.4.2.4.
Solution.
- Constant multiple: \(g'(x)=3f'(x)\text{.}\)
- Product rule:\begin{equation*} g'(x) = \diff{}{x}\{3\}f(x)+3f'(x)=0f(x)+3f'(x)=3f'(x) \end{equation*}
- Sum rule:\begin{align*} g'(x)\amp=\diff{}{x}\{f(x)+f(x)+f(x)\}=f'(x)+f'(x)+f'(x)\\ \amp=3f'(x) \end{align*}
- Quotient rule:\begin{align*} g'(x)\amp=\diff{}{x}\left\{\frac{f(x)}{\frac{1}{3}}\right\}= \frac{\frac{1}{3}f'(x)-f(x)(0)}{\frac{1}{9}}=\frac{\frac{1}{3}f'(x)}{\frac{1}{9}}\\ \amp=9\left(\frac{1}{3}\right)f'(x) =3f'(x) \end{align*}
2.4.2.5.
2.4.2.6.
Solution.
2.4.2.7. (✳).
Solution.
2.4.2.8. (✳).
Solution.
2.4.2.9. (✳).
Solution.
2.4.2.10.
Solution.
So, plugging in \(x=1\text{:}\)
\begin{align*} y'(1)&=\dfrac{-18(3+1)}{(3-2)^3}=-72 \end{align*}2.4.2.11.
Solution.
2.4.2.12.
Solution.
2.4.2.13. (✳).
Solution.
2.4.2.14. (✳).
Solution.
2.4.2.15.
Solution.
2.4.2.16.
Solution.
2.4.2.17.
Solution.
2.6 Using the Arithmetic of Derivatives – Examples
2.6.2 Exercises
2.6.2.1.
Solution.
2.6.2.2.
Solution.
2.6.2.3.
Solution.
2.6.2.4.
Solution.
2.6.2.5.
Solution.
2.6.2.6.
Solution.
2.6.2.7. (✳).
Solution.
2.6.2.8.
Solution.
2.6.2.9.
Solution.
2.6.2.10. (✳).
Solution.
2.6.2.11. (✳).
Solution.
2.6.2.12. (✳).
Solution.
2.6.2.13. (✳).
Solution.
2.6.2.14.
Solution.
2.6.2.15.
Solution.
Now, product rule:
\begin{align*} f'(x)&=(2x\!+\!5)(x^{-1/2}\!+\!x^{-2/3})+(x^2\!+\!5x\!+\!1) \left(\frac{-1}{2}x^{-3/2}\!-\!\frac{2}{3}x^{-5/3}\right) \end{align*}2.6.2.16.
Solution.
2.6.2.17. (✳).
Solution.
2.6.2.18.
Solution.
2.6.2.19. (✳).
Solution.
2.7 Derivatives of Exponential Functions
2.7.3 Exercises
2.7.3.1.
Solution.
2.7.3.2.
Solution.
2.7.3.3.
Solution.
2.7.3.4.
Solution.
2.7.3.5.
Solution.
2.7.3.6.
Solution.
2.7.3.7.
Solution.
Since \(e^a\) is just a constant,
\begin{align*} \diff{}{x}\{e^{a}e^{x}\}&=e^a\diff{}{x}\{e^x\}=e^ae^x=e^{a+x} \end{align*}2.7.3.8.
Solution.
2.7.3.9.
Solution.
Using the rule for differentiating the reciprocal:
\begin{align*} \diff{}{x}\{e^{-x}\}&=\frac{-e^x}{(e^x)^2}=\frac{-1}{e^x}=-e^{-x} \end{align*}2.7.3.10.
Solution.
2.7.3.11.
Solution.
2.7.3.12.
Solution.
2.7.3.13.
Solution.
2.7.3.14. (✳).
Solution.
In particular, we need the one-sided limits to exist and be equal:
\begin{align*} \textcolor{red}{\lim_{h \to 0^-}\frac{f(1+h)-f(1)}{h}}&=\textcolor{blue}{\lim_{h \to 0^+}\frac{f(1+h)-f(1)}{h}}\\ \end{align*}If \(h \lt 0\text{,}\) then \(1+h \lt 1\text{,}\) so \(f(1+h)=a(1+h)^2+b\text{.}\) If \(h \gt 0\text{,}\) then \(1+h \gt 1\text{,}\) so \(f(1+h)=e^{1+h}\text{.}\) With this in mind, we begin to evaluate the one-sided limits:
\begin{align*} \color{red}{\lim_{h \to 0^-}\frac{f(1+h)-f(1)}{h}}&\color{red}{= \lim_{h \to 0^-}\frac{[a(1+h)^2+b]-[a+b]}{h}}\\ &\color{red}={\lim_{h \to 0^-}\frac{ah^2+2ah}{h}=2a}\\ \color{blue}{\lim_{h \to 0^+}\frac{f(1+h)-f(1)}{h}}&\color{blue}{= \lim_{h \to 0^+}\frac{e^{1+h}-(a+b)}{h}}\\ \end{align*}Since we take \(a+b\) to be equal to \(e\) (to ensure continuity):
\begin{align*} &\color{blue}{= \lim_{h \to 0^+}\frac{e^{1+h}-e^1}{h}}\\ &\color{blue}={\left.\diff{}{x}\{e^x\}\right|_{x=1}=e^1=e} \end{align*}
2.8 Derivatives of Trigonometric Functions
2.8.8 Exercises
2.8.8.1.
Solution.
2.8.8.2.
Solution.
2.8.8.3.
Solution.
2.8.8.4.
Solution.
2.8.8.5.
Solution.
- Solution 1: \(f(x)=\sin^2x+\cos^2x=1\text{,}\) so \(f'(x)=\diff{}{x}\{1\}=0\text{.}\)
- Solution 2: Using the formula for the derivative of a squared function,\begin{align*} f'(x)\amp=2\sin x \cos x + 2\cos x(- \sin x)\\ \amp=2\sin x \cos x - 2 \sin x \cos x =0. \end{align*}
2.8.8.6.
Solution.
2.8.8.7.
2.8.8.8.
Solution.
2.8.8.9.
Solution.
2.8.8.10.
Solution.
2.8.8.11.
Solution.
2.8.8.12.
Solution.
2.8.8.13.
Solution.
2.8.8.14. (✳).
Solution.
and
\begin{align*} \lim_{h\to 0^+}\frac{f(0+h)-f(0)}{h} \amp =\lim_{h\to 0^+}\frac{(ah+b)-\cos(0)}{h} =a\qquad\text{since $b=1$}\\ \end{align*}exist and are equal. Because \(\cos(x)\) is differentiable at \(x=0\) we have
\begin{align*} \lim_{h\to 0^-}\frac{\cos(h)-\cos(0)}{h} \amp = \diff{}{x}\cos(x)\bigg|_{x=0} = -\sin(x)\Big|_{x=0}=0 \amp \end{align*}2.8.8.15. (✳).
Solution.
2.8.8.16. (✳).
Solution.
2.8.8.17. (✳).
Solution.
2.8.8.18. (✳).
Solution.
2.8.8.19.
Solution.
So, using the quotient rule,
\begin{align*} \diff{}{\theta}\{\tan \theta\}&=\frac{\cos\theta\cos\theta-\sin\theta(-\sin\theta)}{\cos^2\theta} =\frac{\cos^2\theta+\sin^2\theta}{\cos^2\theta}\\ &=\left(\frac{1}{\cos \theta}\right)^2=\sec^2\theta \end{align*}2.8.8.20. (✳).
Solution.
2.8.8.21. (✳).
Solution.
2.8.8.22. (✳).
Solution.
2.8.8.23. (✳).
Solution.
2.8.8.24. (✳).
Solution.
2.8.8.25. (✳).
Solution.
2.8.8.26.
Solution.
2.8.8.27.
Solution.
2.8.8.28. (✳).
Solution.
So,
\begin{align*} \sin|x|&=\left\{\begin{array}{rl} \sin x&x\ge 0\\ \sin(-x)&x \lt 0 \end{array}\right. =\left\{\begin{array}{rl} \sin x&x\ge 0\\ -\sin x&x \lt 0 \end{array}\right.\\ \end{align*}where we used the identity \(\sin(-x)=-\sin x\text{.}\) From here, it’s easy to see \(h'(x)\) when \(x\) is anything other than zero.
\begin{align*} \diff{}{x}\{\sin|x|\}&=\left\{\begin{array}{rl} \cos x&x \gt 0\\ ??&x=0\\ -\cos x&x \lt 0 \end{array}\right.\\ \end{align*}To decide whether \(h(x)\) is differentiable at \(x=0\text{,}\) we use the definition of the derivative. One word of explanation: usually in the definition of the derivative, \(h\) is the tiny “change in \(x\)” that is going to zero. Since \(h\) is the name of our function, we need another letter to stand for the tiny change in \(x\text{,}\) the size of which is tending to zero. We chose \(t\text{.}\)
\begin{align*} \lim_{t \to 0} \frac{h(t+0)-h(0)}{t}&=\lim_{t \to 0}\frac{\sin|t|}{t}\\ \end{align*}We consider the behaviour of this function to the left and right of \(t=0\text{:}\)
\begin{align*} \frac{\sin |t|}{t}&=\left\{\begin{array}{ll} \frac{\sin t}{t} & t\ge 0\\ \frac{\sin (-t)}{t} & t \lt 0 \end{array}\right. =\left\{\begin{array}{ll} \frac{\sin t}{t} & t\ge 0\\ -\frac{\sin t}{t} & t \lt 0 \end{array}\right.\\ \end{align*}Since we’re evaluating the limit as \(t\) goes to zero, we need the fact that \(\ds\lim_{t \to 0} \dfrac{\sin t}{t}=1\text{.}\) We saw this in Section 2.7, but also we know enough now to evaluate it another way. Using the definition of the derivative:
\begin{align*} \lim_{t \to 0}\frac{\sin t}{t}&=\lim_{t \to 0}\frac{\sin (t+0)-\sin (0)}{t}=\left.\diff{}{x}\{\sin x\}\right|_{t=0}=\cos 0=1 \end{align*}So, since the one-sided limits disagree,
\begin{align*} \lim_{t \to 0} \frac{h(t+0)-h(0)}{t}&=DNE\\ \end{align*}so \(h(x)\) is not differentiable at \(x=0\text{.}\) Therefore,
\begin{align*} h'(x)&=\left\{\begin{array}{rl} \cos x&x \gt 0\\ -\cos x&x \lt 0 \end{array}\right. \end{align*}2.8.8.29. (✳).
Solution.
So \(f\) is continuous at \(x=0\text{,}\) and so Statement 2.8.8.29.ii does not hold. Now let’s consider \(f'(x)\)
\begin{align*} \lim_{x\rightarrow 0+}\frac{f(x)-f(0)}{x} &=\lim_{x\rightarrow 0+}\frac{\frac{\sin x}{\sqrt{x}}-0}{x}\\ &=\lim_{x\rightarrow 0+}\frac{1}{\sqrt{x}}\frac{\sin x}{x}=+\infty\\ \end{align*}Therefore, using the definition of the derivative,
\begin{align*} f'(0)&=\lim_{x \to 0}\frac{f(x)-f(0)}{x}\quad\mbox{ if it exists, but}\\ \lim_{x \to 0}\frac{f(x)-f(0)}{x}&=DNE \end{align*}2.8.8.30. (✳).
Solution.
2.9 One More Tool – the Chain Rule
2.9.4 Exercises
2.9.4.1.
Solution.
2.9.4.2.
Solution.
2.9.4.3.
Solution.
2.9.4.4.
Solution.
2.9.4.5.
Solution.
2.9.4.6.
Solution.
And now, the quotient rule:
\begin{align*} &=\frac{1}{2}\sqrt{\frac{x^2-1}{x^2+1}}\cdot\left(\frac{(x^2-1)(2x)-(x^2+1)2x}{(x^2-1)^2}\right)\\ &=\frac{1}{2}\sqrt{\frac{x^2-1}{x^2+1}}\cdot\left(\frac{-4x}{(x^2-1)^2}\right)\\ &=\sqrt{\frac{x^2-1}{x^2+1}}\cdot\left(\frac{-2x}{(x^2-1)^2}\right)\\ &=\frac{-2x}{({x^2-1})\sqrt{x^4-1}} \end{align*}2.9.4.7.
Solution.
In order to evaluate \(\diff{}{x}\{{\cos(x^2)}\}\text{,}\) we’ll need the chain rule again.
\begin{align*} &=e^{\cos(x^2)}\cdot [-\sin(\textcolor{orange}{x^2})]\cdot\diff{}{x}\{\textcolor{orange}{x^2}\}\\ &=-e^{\cos(x^2)}\cdot \sin(x^2)\cdot 2x \end{align*}2.9.4.8. (✳).
Solution.
When \(x=2\text{:}\)
\begin{align*} f'(2) &= g'\left(\frac{2}{h(2)}\right)\frac{h(2)-2h'(2)}{h(2)^2}\\ &= 4\frac{2-2\times3}{2^2} =-4 \end{align*}2.9.4.9. (✳).
Solution.
2.9.4.10. (✳).
Solution.
2.9.4.11. (✳).
Solution.
2.9.4.12. (✳).
Solution.
Now, we can use the power rule to differentiate \(\dfrac{1}{x^2}\text{.}\) This will be easier than differentiating \(\dfrac{1}{x^2}\) using quotient rule, but if you prefer, quotient rule will also work.
\begin{align*} f'(x)&=-2x^{-3}+\frac{1}{2}(\textcolor{red}{x^2-1})^{-1/2}\cdot\diff{}{x}\{\textcolor{red}{x^2-1}\}\\ &=-2x^{-3}+\frac{1}{2}({x^2-1})^{-1/2}(2x)\\ &=\frac{-2}{x^3}+\frac{x}{\sqrt{x^2-1}} \end{align*}2.9.4.13. (✳).
Solution.
2.9.4.14.
Solution.
Here, we need the chain rule again:
\begin{align*} &=\sec(e^{2x+7})\tan(e^{2x+7}) \cdot \left[e^{\textcolor{red}{2x+7}}\cdot \diff{}{x}\{\textcolor{red}{2x+7}\}\right]\\ &=\sec(e^{2x+7})\tan(e^{2x+7}) \cdot \left[e^{2x+7}\cdot2\right]\\ &=2e^{2x+7}\sec(e^{2x+7})\tan(e^{2x+7}) \end{align*}2.9.4.15.
Solution.
2.9.4.16.
Solution.
To determine where this function is zero, we factor:
\begin{align*} &=e^{{t^3-7t^2+8t}}\cdot(3t-2)(t-4) \end{align*}2.9.4.17.
Solution.
We find ourselves once more in need of the chain rule:
\begin{align*} &=\sec^2({e^{x^2}})\cdot{e^{\textcolor{red}{x^2}}}\diff{}{x}\{\textcolor{red}{x^2}\}\\ &=\sec^2(e^{x^2})\cdot e^{x^2}\cdot 2x\\ \end{align*}Finally, we evaluate this derivative at the point \(x=1\text{:}\)
\begin{align*} y'(1)&=\sec^2(e)\cdot e \cdot 2\\ &=2e\sec^2e \end{align*}2.9.4.18. (✳).
Solution.
and the chain rule:
\begin{align*} &=e^{\textcolor{red}{4x}}\cdot\diff{}{x}\{\textcolor{red}{4x}\}\cdot\tan x+e^{4x}\sec^2 x\\ &=4e^{4x}\tan x+e^{4x}\sec^2 x \end{align*}2.9.4.19. (✳).
Solution.
Now, the chain rule:
\begin{align*} &=\frac{(3x^2)(1+e^{3x})-(x^3)(3e^{3x})}{{(1+e^{3x})}^2}\\ \end{align*}So, when \(x=1\text{:}\)
\begin{align*} f'(1)&=\frac{3(1+e^{3})-3e^{3}}{{(1+e^{3})}^2}=\frac{3}{{(1+e^{3})}^2} \end{align*}2.9.4.20. (✳).
Solution.
2.9.4.21. (✳).
Solution.
2.9.4.22. (✳).
Solution.
2.9.4.23. (✳).
Solution.
and find ourselves in need of chain rule a second time:
\begin{align*} &=-\sin\big(x^2+\sqrt{x^2+1}\big)\cdot\left(2x+\dfrac{1}{2\sqrt{\textcolor{red}{x^2+1}}}\cdot\diff{}{x}\left\{\textcolor{red}{x^2+1}\right\}\right)\\ &=-\sin\big(x^2+\sqrt{x^2+1}\big)\cdot\left(2x+\dfrac{2x}{2\sqrt{x^2+1}}\right) \end{align*}2.9.4.24. (✳).
Solution.
Using the product rule,
\begin{align*} y'&=(2x)\cos^2 x + (1+x^2)\diff{}{x}\{\cos^2 x\}\\ \end{align*}Here, we’ll need to use the chain rule. Remember \(\cos^2 x = [\cos x]^2\text{.}\)
\begin{align*} &=2x\cos^2x+(1+x^2) 2\textcolor{red}{\cos x} \cdot \diff{}{x}\{\textcolor{red}{\cos x}\}\\ &=2x\cos^2x+(1+x^2) 2\cos x \cdot (-\sin x)\\ &=2x\cos^2x-2(1+x^2) \sin x\cos x \end{align*}2.9.4.25. (✳).
Solution.
2.9.4.26. (✳).
Solution.
Plugging in \(x=2\text{:}\)
\begin{align*} g'(2)&=3(2^2)h(2^2)+2^3h'(2^2)2\times 2\cr &=12h(4)+32h'(4)=12\times 2-32\times 2\cr &=-40 \end{align*}2.9.4.27. (✳).
Solution.
Here, we need the chain rule:
\begin{align*} &=e^{(1-x^2)/2}+x\cdot e^{\textcolor{red}{(1-x^2)/2}}\diff{}{x}\left\{\textcolor{red}{\frac{1}{2}(1-x^2)}\right\}\\ &=e^{(1-x^2)/2}+x\cdot e^{{(1-x^2)/2}}\cdot (-x)\\ &=(1-x^2)e^{(1-x^2)/2} \end{align*}2.9.4.28.
Solution.
2.9.4.29.
Solution.
- Solution 1: We begin with the quotient rule:\begin{align*} f'(x) &= \frac{\cos^3(5x-7)\diff{}{x}\{e^x\}-e^x\diff{}{x}\{\cos^3(5x-7)\}}{\cos^6(5x-7)}\\ &= \frac{\cos^3(5x-7)e^x-e^x\diff{}{x}\{\cos^3(5x-7)\}}{\cos^6(5x-7)}\\ \end{align*}
Now, we use the chain rule. Since \(\cos^3(5x-7)=[\cos(5x-7)]^3\text{,}\) our “outside” function is \(g(x)=x^3\text{,}\) and our “inside” function is \(h(x)=\cos(5x-1)\text{.}\)
\begin{align*} &= \frac{\cos^3(5x-7)e^x-e^x\cdot3\textcolor{red}{\cos}^2\textcolor{red}{(5x-7)} \cdot \diff{}{x}\{\textcolor{red}{\cos(5x-7)}\}}{\cos^6(5x-7)}\\ \end{align*}We need the chain rule again!
\begin{align*} &= \frac{\cos^3(5x\!-\!7)e^x\!-\!e^x\cdot3{\cos}^2{(5x\!-\!7)} \cdot[{-\sin(\textcolor{red}{5x\!-\!7})\cdot \diff{}{x}\{\textcolor{red}{5x\!-\!7}\}}]}{\cos^6(5x-7)}\\ &= \frac{\cos^3(5x-7)e^x-e^x\cdot3{\cos}^2{(5x-7)} \cdot[{-\sin({5x-7})\cdot5}]}{\cos^6(5x-7)}\\ \end{align*}We finish by simplifying:
\begin{align*} &= \frac{e^x\cos^2(5x-7)\left(\cos(5x-7)+15\sin(5x-7)\right)}{\cos^6(5x-7)}\\ &=e^x \frac{\cos(5x-7)+15\sin(5x-7)}{\cos^4(5x-7)}\\ &=e^x(\sec^3(5x-7)+15\tan(5x-7)\sec^3(5x-7))\\ &=e^x\sec^3(5x-7)(1+15\tan(5x-7)) \end{align*} - Solution 2: We simplify to avoid the quotient rule:\begin{align*} f(x)&=\dfrac{e^{x}}{\cos^3 (5x-7)}\\ &=e^x\sec^3(5x-7)\\ \end{align*}
Now we use the product rule to differentiate:
\begin{align*} f'(x)&=e^x\sec^3(5x-7)+e^x\diff{}{x}\{\sec^3(5x-7)\}\\ \end{align*}Here, we’ll need the chain rule. Since \(\sec^3(5x-7)=[\sec (5x-7)]^3\text{,}\) our “outside” function is \(g(x)=x^3\) and our “inside” function is \(h(x)=\sec(5x-7)\text{,}\) so that \(g(h(x))=[\sec(5x-7)]^3=\sec^3(5x-7)\text{.}\)
\begin{align*} &=e^x\sec^3(5x-7)+e^x\cdot3\;\textcolor{red}{\sec}^2\textcolor{red}{(5x-7)} \cdot \diff{}{x}\{\textcolor{red}{\sec(5x-7)}\}\\ \end{align*}We need the chain rule again! Recall \(\diff{}{x}\{\sec x\}=\sec x \tan x\text{.}\)
\begin{align*} &=e^x\sec^3(5x-7)\\ \amp\hskip0.2in+e^x\cdot3\;{\sec}^2{(5x\!-\!7)} \cdot {\sec(\textcolor{orange}{5x\!-\!7})\tan(\textcolor{orange}{5x\!-\!7})\cdot\diff{}{x}\{\textcolor{orange}{5x\!-\!7}\}}\\ &=e^x\sec^3(5x\!-\!7)\\ \amp\hskip0.2in+e^x\cdot3\;{\sec}^2{(5x\!-\!7)} \cdot {\sec({5x\!-\!7})\tan({5x\!-\!7})\cdot 5}\\ \end{align*}We finish by simplifying:
\begin{align*} &=e^x\sec^3(5x-7)(1+15\tan({5x-7})) \end{align*}
2.9.4.30. (✳).
Solution.
-
Solution 1: In Example 2.6.6, we generalized the product rule to three factors:\begin{align*} \diff{}{x}\{f(x)g(x)h(x)\}\amp=f'(x)g(x)h(x)+f(x)g'(x)h(x)\\ \amp\hskip1in+f(x)g(x)h'(x) \end{align*}Using this rule:\begin{align*} \amp\diff{}{x}\left\{(x) \left(e^{2x}\right)( \cos 4x)\right\}\\ &=\diff{}{x}\{x\}\cdot e^{2x} \cos 4x \!+\! x\cdot\diff{}{x}\left\{e^{2x}\right\}\cdot\cos 4x \!+\! xe^{2x}\cdot\diff{}{x}\{\cos 4x\}\\ &=e^{2x}\cos4x+x\left(2e^{2x}\right)\cos4x+ xe^{2x}(-4\sin 4x)\\ &=e^{2x}\cos4x+2xe^{2x}\cos4x-4xe^{2x}\sin 4x \end{align*}
- Solution 2: We can use the product rule twice. In the first step, we split the function \(x e^{2x} \cos 4x\) into the product of two functions.\begin{align*} \amp\diff{}{x}\left\{\left(x e^{2x}\right) (\cos 4x)\right\}\\ &= \diff{}{x}\left\{xe^{2x}\right\}\cdot\cos4x + xe^{2x}\cdot\diff{}{x}\left\{ \cos 4x\right\}\\ &= \left( \diff{}{x}\left\{x\right\}\cdot e^{2x}+ x\cdot\diff{}{x}\left\{e^{2x}\right\} \right)\cdot\cos4x + xe^{2x}\cdot\diff{}{x}\left\{ \cos 4x\right\}\\ &= \left( e^{2x}+ x\left(2e^{2x}\right) \right)\cdot\cos4x + xe^{2x}(-4\sin 4x)\\ &=e^{2x}\cos4x+2xe^{2x}\cos4x-4xe^{2x}\sin4x \end{align*}
2.9.4.31.
Solution.
2.9.4.32. (✳).
Solution.
2.9.4.33.
Solution.
2.9.4.34.
Solution.
2.9.4.35.
Solution.
\(t\) | \((\sin t,\cos^2 t)\) |
\(0\) | \((0,1)\) |
\(\pi/4\) | \((\frac{1}{\sqrt{2}},\frac{1}{2})\) |
\(\pi/2\) | \((1,0)\) |
\(3\pi/4\) | \((\frac{1}{\sqrt{2}},\frac{1}{2})\) |
\(\pi\) | \((0,1)\) |
\(5\pi/4\) | \((-\frac{1}{\sqrt{2}},\frac{1}{2})\) |
\(3\pi/2\) | \((-1,0)\) |
\(7\pi/4\) | \((-\frac{1}{\sqrt{2}},\frac{1}{2})\) |
\(2\pi\) | \((0,1)\) |
Using \(y(t)=\cos^2 t\) and \(x(t)=\sin t\text{:}\)
\begin{align*} \diff{f}{x}&=\left(-2\cos t \sin t \right)\div\left( \cos t \right)=-2\sin t=-2x\\ \end{align*}So, when \(t=\dfrac{10\pi}{3}\) and \(x=-\dfrac{\sqrt{3}}{2},\)
\begin{align*} \diff{f}{x}\left(\dfrac{-\sqrt{3}}{2}\right)&=-2\cdot\frac{-\sqrt{3}}{2}=\sqrt{3}. \end{align*}The slope of the curve is \(\ds\diff{y}{x}\text{.}\) To find \(\ds\diff{y}{x}\text{,}\) we use the chain rule:\begin{align*} \diff{y}{t}&=\diff{y}{x}\cdot\diff{x}{t}\\ \diff{}{t}\left\{\cos^2 t\right\}&=\diff{y}{x}\cdot\diff{}{t}\{\sin t\}\\ -2\cos t \sin t &= \diff{y}{x} \cdot \cos t\\ \diff{y}{x}&=-2\sin t\\ \end{align*}So, when \(t=\dfrac{10\pi}{3}\text{,}\)
\begin{align*} \diff{y}{x}&=-2\sin\left(\frac{10\pi}{3}\right)=-2\left(-\frac{\sqrt{3}}{2}\right)=\sqrt{3}. \end{align*}
2.10 The Natural Logarithm
2.10.3 Exercises
2.10.3.1.
Solution.
So, for ten speakers:
\begin{align*} V(10P)&=10\log_{10}\left(\frac{10P}{S}\right)=10\log_{10}\left(\frac{P}{S}\right)+10\log_{10}\left(10\right)\\ &=3+10(1)=13 \mathrm{dB}\\ \end{align*}and for one hundred speakers:
\begin{align*} V(100P)&=10\log_{10}\left(\frac{100P}{S}\right)=10\log_{10}\left(\frac{P}{S}\right)+10\log_{10}\left(100\right)\\ &=3+10(2)=23 \mathrm{dB} \end{align*}2.10.3.2.
Solution.
2.10.3.3.
Solution.
2.10.3.4.
Solution.
- Solution 1: Using the chain rule, \(\ds\diff{}{x}\left\{\log(10x)\right\}=\dfrac{1}{10x}\cdot 10=\frac{1}{x}\text{.}\)
- Solution 2: Simplifying, \(\ds\diff{}{x}\left\{\log(10x)\right\}=\ds\diff{}{x}\left\{\log(10)+\log x\right\} = 0+\frac{1}{x}=\frac{1}{x}\text{.}\)
2.10.3.5.
Solution.
- Solution 1: Using the chain rule, \(\ds\diff{}{x}\left\{\log(x^2)\right\}=\frac{1}{x^2}\cdot 2x = \frac{2}{x}\text{.}\)
- Solution 2: Simplifying, \(\ds\diff{}{x}\left\{\log(x^2)\right\}= \ds\diff{}{x}\left\{2\log(x)\right\}= \frac{2}{x}\text{.}\)
2.10.3.6.
Solution.
2.10.3.7.
Solution.
Since \(\log 10\) is a constant:
\begin{align*} f'(x)&=\frac{1}{x\log 10}. \end{align*}2.10.3.8. (✳).
Solution.
- Solution 1: Using the quotient rule,\begin{equation*} y'=\frac{x^3\frac{1}{x}-(\log x)\cdot 3x^2}{x^6}= \frac{x^2-3x^2\log x}{x^6}=\frac{1-3\log x}{x^4}. \end{equation*}
- Solution 2: Using the product rule with \(y=\log x \cdot x^{-3}\text{,}\)\begin{equation*} y'=\frac{1}{x}x^{-3}+\log x \cdot(-3)x^{-4}=x^{-4}(1-3\log x) \end{equation*}
2.10.3.9.
Solution.
2.10.3.10.
Solution.
We’ll need the chain rule again:
\begin{align*} &= e^{\cos\left(\log x\right)} (-\sin(\textcolor{orange}{\log x})) \cdot \diff{}{x}\{\textcolor{orange}{ \log x} \}\\ &= e^{\cos\left(\log x\right)} (-\sin(\log x)) \cdot \frac{1}{x}\\ &=\frac{-e^{\cos(\log x)}\sin(\log x)}{x} \end{align*}2.10.3.11. (✳).
Solution.
So, we’ll need the chain rule:
\begin{align*} y'&=\frac{\diff{}{x}\left\{\textcolor{red}{x^2+\sqrt{x^4+1}}\right\}}{\textcolor{red}{x^2+\sqrt{x^4+1}}}\\ &=\frac{2x+\diff{}{x}\left\{\sqrt{x^4+1}\right\}}{x^2+\sqrt{x^4+1}}\\ \end{align*}We need the chain rule again:
\begin{align*} &=\frac{2x+\frac{\diff{}{x}\left\{\textcolor{red}{x^4+1}\right\}}{2\sqrt{\textcolor{red}{x^4+1}}}}{x^2+\sqrt{x^4+1}}\\ &=\frac{2x+\frac{4x^3}{2\sqrt{x^4+1}}}{x^2+\sqrt{x^4+1}}. \end{align*}2.10.3.12. (✳).
Solution.
2.10.3.13. (✳).
Solution.
2.10.3.14. (✳).
Solution.
2.10.3.15. (✳).
Solution.
2.10.3.16.
Solution.
Now, we differentiate using the chain rule:
\begin{align*} f'(x)&=\frac{1}{2}\left[ 3\frac{2x}{x^2+5}-\frac{4x^3}{x^4+10} \right]\\ &=\frac{3x}{x^2+5}-\frac{2x^3}{x^4+10} \end{align*}2.10.3.17.
Solution.
In particular, when \(x=2\text{:}\)
\begin{align*} f'(2) &= \dfrac{1}{g\big(2h(2)\big)}\cdot g'\big(2h(2)\big)\cdot \big[h(2)+2h'(2)\big]\\ &= \dfrac{g'(4)}{g(4)}\big[2+2\times 3\big] = \dfrac{5}{3}\big[2+2\times 3\big]\\ &=\dfrac{40}{3} \end{align*}2.10.3.18. (✳).
Solution.
2.10.3.19.
Solution.
Now, we can use the product rule to differentiate the right side, and the chain rule to differentiate \(\log(f(x))\text{:}\)
\begin{align*} g'(x)=\frac{f'(x)}{f(x)}&=\log x +x\frac{1}{x}=\log x +1\\ \end{align*}Finally, we solve for \(f'(x)\text{:}\)
\begin{align*} f'(x)&=f(x)(\log x + 1) = x^x(\log x + 1) \end{align*}2.10.3.20. (✳).
Solution.
2.10.3.21.
Solution.
Now that we’ve simplified, we can efficiently differentiate both sides. It is important to remember that we aren’t differentiating \(f(x)\) directly--we’re differentiating \(\log(f(x))\text{.}\)
\begin{align*} \frac{f'(x)}{f(x)}&=\frac{1}{4}\left(\frac{4x^3}{x^4+12}+\frac{4x^3-2x}{x^4-x^2+2}-\frac{3}{x}\right) \end{align*}2.10.3.22.
Solution.
Now we can use logarithm rules to change \(g(x)\) into a form that is friendlier to differentiate:
\begin{align*} &=\log(x+1)+\log(x^2+1)^2+\log(x^3+1)^3+\log(x^4+1)^4\\ \amp\hskip2in+\log(x^5+1)^5\\ &=\log(x+1)+2\log(x^2+1)+3\log(x^3+1)+4\log(x^4+1)\\ \amp\hskip2in+5\log(x^5+1)\\ \end{align*}Now, we differentiate \(g(x)\) using the chain rule:
\begin{align*} g'(x)=\frac{f'(x)}{f(x)}&=\frac{1}{x+1}+\frac{4x}{x^2+1} +\frac{9x^2}{x^3+1}+\frac{16x^3}{x^4+1}+\frac{25x^4}{x^5+1} \end{align*}2.10.3.23.
Solution.
2.10.3.24. (✳).
Solution.
Logarithm rules allowed us to simplify. Now, we differentiate both sides of this equation:
\begin{align*} \frac{f'(x)}{f(x)}&=(\cos x ) \log(\cos x)+ \sin x \cdot \frac{-\sin x}{\cos x}\\ &=(\cos x) \log (\cos x) - \sin x \tan x\\ \end{align*}Finally, we solve for \(f'(x)\text{:}\)
\begin{align*} f'(x)&=f(x)\left[(\cos x) \log (\cos x) - \sin x \tan x\right]\\ &= (\cos x)^{\sin x}\left[(\cos x) \log (\cos x) - \sin x \tan x\right] \end{align*}2.10.3.25. (✳).
Solution.
Now, we can differentiate:
\begin{align*} \frac{\diff{}{x}\left\{(\tan x)^x\right\}}{(\tan x)^x}&=\log(\tan x) + x\cdot\frac{\sec^2 x}{\tan x}\\ &=\log(\tan x) + \frac{x}{\sin x \cos x}\\ \end{align*}Finally, we solve for the derivative we want, \(\ds\diff{}{x}\{(\tan x)^x\}\text{:}\)
\begin{align*} {\diff{}{x}\left\{(\tan x)^x\right\}}&={(\tan x)^x}\left(\log(\tan x) + \frac{x}{\sin x \cos x}\right) \end{align*}2.10.3.26. (✳).
Solution.
We differentiate both sides to obtain:
\begin{align*} \dfrac{f'(x)}{f(x)} &= \diff{}{x} \left\{ \log(x^2+1) \cdot (x^2+1) \right\}\\ &= \frac{2x}{x^2+1}(x^2+1)+2x\log(x^2+1)\\ &=2x(1+\log(x^2+1))\\ \end{align*}Now, we solve for \(f'(x)\text{:}\)
\begin{align*} f'(x) &= f(x) \cdot 2x(1+\log(x^2+1))\\ &= (x^2+1)^{x^2+1} \cdot 2x(1+\log(x^2+1)) \end{align*}2.10.3.27. (✳).
Solution.
We differentiate using the product and chain rules:
\begin{align*} \dfrac{f'(x)}{f(x)} &= \diff{}{x} \left\{ \log(x^2+1) \cdot \sin x \right\} = \cos x \cdot \log(x^2+1) + \frac{2x\sin x}{x^2+1}\\ \end{align*}Finally, we solve for \(f'(x)\)
\begin{align*} f'(x) &= f(x) \cdot \left( \cos x \cdot \log(x^2+1) + \frac{2x\sin x}{x^2+1} \right)\\ &= (x^2+1)^{\sin(x)} \cdot \left( \cos x \cdot \log(x^2+1) + \frac{2x\sin x}{x^2+1} \right) \end{align*}2.10.3.28. (✳).
Solution.
Differentiating, we find:
\begin{align*} \dfrac{f'(x)}{f(x)} &= \diff{}{x} \left\{ \log(x) \cdot \cos^3(x) \right\}\\ \amp= 3\cos^2(x)\cdot (-\sin(x)) \cdot \log(x) + \frac{\cos^3(x)}{x}\\ \end{align*}Finally, we solve for \(f'(x)\text{:}\)
\begin{align*} f'(x) &= f(x) \cdot \left( -3\cos^2(x)\sin(x) \log(x) + \frac{\cos^3(x)}{x} \right)\\ &= x^{\cos^3(x)} \cdot \left( -3\cos^2(x)\sin(x) \log(x) + \frac{\cos^3(x)}{x} \right) \end{align*}2.10.3.29. (✳).
Solution.
We differentiate:
\begin{align*} \frac{f'(x)}{f(x)} &= \diff{}{x} \left\{(x^2-3)\cdot \log(3+\sin(x)) \right\}\\ &=2x\log(3+\sin(x)) + (x^2-3)\frac{\cos(x)}{3+\sin(x)}\\ \end{align*}Finally, we solve for \(f'(x)\text{:}\)
\begin{align*} f'(x)&= f(x)\cdot \left[ 2x\log(3+\sin(x)) + \frac{(x^2-3)\cos(x)}{3+\sin(x)}\right]\\ &= (3+\sin(x))^{x^2-3}\cdot \left[ 2x\log(3+\sin(x)) + \frac{(x^2-3)\cos(x)}{3+\sin(x)}\right] \end{align*}2.10.3.30.
Solution.
Now, we differentiate. On the left side we use the chain rule, and on the right side we use product and chain rules.
\begin{align*} \ds\diff{}{x}\left\{\log\left([f(x)]^{g(x)}\right)\right\}&=\ds\diff{}{x}\left\{g(x)\log(f(x))\right\}\\ \frac{\diff{}{x}\{[f(x)]^{g(x)}\}}{[f(x)]^{g(x)}}&= g'(x)\log(f(x))+g(x)\cdot\frac{f'(x)}{f(x)}\\ \end{align*}Finally, we solve for the derivative of our original function.
\begin{align*} {\diff{}{x}\{[f(x)]^{g(x)}\}}&={[f(x)]^{g(x)}}\left( g'(x)\log(f(x))+g(x)\cdot\frac{f'(x)}{f(x)}\right) \end{align*}2.10.3.31.
Solution.
2.11 Implicit Differentiation
2.11.2 Exercises
2.11.2.1.
Solution.
2.11.2.2.
Solution.
2.11.2.3.
Solution.
and solve for \(\ds\diff{y}{x}\)
\begin{align*} \diff{y}{x}&=-\frac{x}{y} \end{align*}2.11.2.4. (✳).
Solution.
Now, we solve for \(\ds\diff{y}{x}\text{.}\)
\begin{align*} x\diff{y}{x}+e^y\diff{y}{x}&=-(e^x+y)\\ (x+e^y)\diff{y}{x}&=-(e^x+y)\\ \diff{y}{x}&=-\frac{e^x+y}{e^y+x} \end{align*}2.11.2.5. (✳).
Solution.
Now, get the derivative on one side and solve
\begin{align*} e^y\diff{y}{x}-2xy\diff{y}{x}&=y^2+1\\ \diff{y}{x}\left(e^y-2xy\right)&=y^2+1\\ \diff{y}{x}&=\frac{y^2+1}{e^y-2xy} \end{align*}2.11.2.6. (✳).
Solution.
- First we find the \(x\)-coordinates where \(y=1\text{.}\)\begin{align*} x^2\tan\left(\frac{\pi}{4}\right)+2x\log(1) &= 16\\ x^2\cdot 1 +2x\cdot 0 &=16\\ x^2 &= 16 \end{align*}So \(x=\pm 4\text{.}\)
- Now we use implicit differentiation to get \(y'\) in terms of \(x,y\text{:}\)\begin{align*} x^2\tan(\pi y/4)+2x\log(y) &= 16\\ 2x\tan(\pi y/4) + x^2 \frac{\pi}{4}\sec^2(\pi y/4)\cdot y' + 2\log(y) + \frac{2x}{y} \cdot y' &= 0\,. \end{align*}
- Now set \(y=1\) and use \(\tan(\pi/4)=1\,, \sec(\pi/4)=\sqrt{2}\) to get\begin{align*} 2x\tan(\pi/4) + x^2 \frac{\pi}{4} \sec^2(\pi/4)y' + 2\log(1) + 2x\cdot y' &= 0\\ 2x + \frac{\pi}{2} x^2 y' +2x y' &= 0\\ y' = -\frac{2x}{x^2 \pi/2 + 2x} &= -\frac{4}{\pi x + 4} \end{align*}
- So at \((x,y)=(4,1)\) we have \(y' = -\dfrac{4}{4\pi+4} = -\dfrac{1}{\pi + 1}\)
- and at \((x,y)=(-4,1)\) we have \(y' = \dfrac{1}{\pi-1}\)
2.11.2.7. (✳).
Solution.
2.11.2.8. (✳).
Solution.
- First we find the \(x\)-coordinates where \(y=0\text{.}\)\begin{align*} x^2e^0+4x\cos(0) &= 5\\ x^2 +4x - 5 &=0\\ (x+5)(x-1)&=0 \end{align*}So \(x=1,-5\text{.}\)
- Now we use implicit differentiation to get \(y'\) in terms of \(x,y\text{.}\) Differentiate both sides of\begin{align*} x^2e^y+4x\cos(y) &= 5 \end{align*}to get\begin{align*} x^2 \cdot e^y \cdot y' + 2x e^y + 4x(-\sin(y)) \cdot y' + 4\cos(y) &= 0 \end{align*}
- Now set \(y=0\) to get\begin{align*} x^2 \cdot e^0 \cdot y' + 2x e^0 + 4x(-\sin(0)) \cdot y' + 4\cos(0) &= 0\\ x^2y' + 2x + 4 &=0\\ y' &= - \frac{4+2x}{x^2}. \end{align*}
- So at \((x,y)=(1,0)\) we have \(y' = -6\text{,}\)
- and at \((x,y)=(-5,0)\) we have \(y' = \frac{6}{25}\text{.}\)
2.11.2.9. (✳).
Solution.
2.11.2.10. (✳).
Solution.
- First we find the \(x\)-coordinates where \(y=0\text{.}\)\begin{align*} x^2\cos(0)+2xe^0 &= 8\\ x^2 +2x - 8 &=0\\ (x+4)(x-2)&=0 \end{align*}So \(x=2,-4\text{.}\)
- Now we use implicit differentiation to get \(y'\) in terms of \(x,y\text{.}\) Differentiate both sides of\begin{align*} x^2\cos(y)+2xe^y &= 8 \end{align*}to get\begin{align*} x^2 \cdot (-\sin y) \cdot y' + 2x \cos y + 2xe^y \cdot y' + 2e^y &= 0 \end{align*}
- Now set \(y=0\) to get\begin{align*} x^2 \cdot (-\sin 0) \cdot y' + 2x \cos 0 + 2xe^0 \cdot y' + 2e^0 &= 0\\ 0 + 2x + 2xy' + 2 &=0\\ y' &= - \frac{2+2x}{2x} \\ \amp= -\frac{1+x}{x} \end{align*}
- So at \((x,y)=(2,0)\) we have \(y' = -\frac{3}{2}\text{,}\)
- and at \((x,y)=(-4,0)\) we have \(y' = -\frac{3}{4}\text{.}\)
2.11.2.11.
Solution.
We could solve for \(\ds\diff{y}{x}\) at this point, but it’s not necessary. We want to know when \(\ds\diff{y}{x}\) is equal to one:
\begin{align*} 2x+6y(1)&=0\\ x&=-3y\\ \end{align*}That is, \(\ds\diff{y}{x}=1\) at those points along the ellipse where \(x=-3y\text{.}\) We plug this into the equation of the ellipse to find the coordinates of these points.
\begin{align*} \left(-3y\right)^2+3y^2&=1\\ 12y^2&=1\\ y=\pm\frac{1}{\sqrt{12}}=\pm\frac{1}{2\sqrt{3}} \end{align*}2.11.2.12. (✳).
Solution.
Now, we plug in \(x=1\text{,}\) \(y=4\text{,}\) and solve for \(\ds\diff{y}{x}\text{:}\)
\begin{align*} \frac{4+\diff{y}{x}}{4}&=8+\diff{y}{x}\\ \diff{y}{x}&=-\frac{28}{3} \end{align*}2.11.2.13. (✳).
Solution.
Then we gather the terms containing \(y'\) on one side, so we can solve for \(y'\text{:}\)
\begin{align*} 2x^2yy'&+xy'\cos y=-2xy^2-\sin y\\ y'(2x^2y&+x\cos y)=-2xy^2-\sin y\\ y'&=-\frac{2xy^2+\sin y}{2x^2y+x\cos y} \end{align*}2.11.2.14. (✳).
Solution.
- First we find the \(x\)-ordinates where \(y=0\text{.}\)\begin{align*} x^2+(1)e^0 &= 5\\ x^2 +1 &=5\\ x^2&=4 \end{align*}So \(x=2,-2\text{.}\)
- Now we use implicit differentiation to get \(y'\) in terms of \(x,y\text{:}\)\begin{align*} 2x+(y+1)e^y\diff{y}{x}+e^y\diff{y}{x}&=0 \end{align*}
- Now set \(y=0\) to get\begin{align*} 2x+(0+1)e^0\diff{y}{x}+e^0\diff{y}{x}&=0\\ 2x+\diff{y}{x}+\diff{y}{x}&=0\\ 2x&=-2\diff{y}{x}\\ x&=-\diff{y}{x} \end{align*}
- So at \((x,y)=(2,0)\) we have \(y' = -2\text{,}\)
- and at \((x,y)=(-2,0)\) we have \(y' = 2\text{.}\)
2.11.2.15.
Solution.
For the circle, we differentiate implicitly
\begin{align*} 2x+2y\diff{y}{x}&=0\\ \end{align*}and solve for \(\ds\diff{y}{x}\)
\begin{align*} \diff{y}{x}&=-\frac{x}{y}\\ \end{align*}For the ellipse, we also differentiate implicitly:
\begin{align*} 2x+6y\diff{y}{x}&=0\\ \end{align*}and solve for \(\ds\diff{y}{x}\)
\begin{align*} \diff{y}{x}&=-\frac{x}{3y}\\ \end{align*}What we want is a value of \(x\) where both derivatives are equal. However, they might have different values of \(y\text{,}\) so let’s let \(y_1\) be the \(y\)-values associated with \(x\) on the circle, and let \(y_2\) be the \(y\)-values associated with \(x\) on the ellipse. That is, \(x^2+ y_1^2=1\) and \(x^2+3y_2^2=1\text{.}\) For the slopes at \((x,y_1)\) on the circle and \((x,y_2)\) on the ellipse to be equal, we need:
\begin{align*} -\frac{x}{y_1}&=-\frac{x}{3y_2}\\ x\left(\frac{1}{y_1}-\frac{1}{3y_2}\right)&=0\\ \end{align*}So \(x=0\) or \(y_1=3y_2\text{.}\) Let’s think about which \(x\)-values will have a \(y\)-coordinate of the circle be three times as large as a \(y\)-coordinate of the ellipse. If \(y_1=3y_2\text{,}\) \((x,y_1)\) is on the circle, and \((x,y_2)\) is on the ellipse, then \(x^2+y_1^2 =x^2+(3y_2)^2=1\) and \(x^2+3y_2^2=1\text{.}\) In this case:
\begin{align*} x^2+9y_2^2&=x^2+3y_2^2\\ 9y_2^2&=3y_2^2\\ y_2&=0\\ x&=\pm 1 \end{align*}
2.12 Inverse Trigonometric Functions
2.12.2 Exercises
2.12.2.1.
Solution.
2.12.2.2.
Solution.
2.12.2.3.
Solution.
2.12.2.4.
Solution.
- If \(|a| \gt 1\text{,}\) then there is no value of \(x\) for which \(a=\sin x\text{,}\) so the curve has no horizontal tangent lines.
- If \(|a| = 1\text{,}\) then there are infinitely many solutions to \(a=\sin x\text{,}\) but only one solution in the interval \([-\pi,\pi]\text{:}\) \(x=\arcsin(a)=\arcsin(\pm1)=\pm\frac{\pi}{2}\text{.}\) Then the values of \(x\) for which \(a=\sin x\) are \(x=2\pi n +a \frac{\pi}{2}\) for any integer \(n\text{.}\)
-
If \(|a| \lt 1\text{,}\) then there are infinitely many solutions to \(a=\sin x\text{.}\) The solution in the interval \(\left(-\frac{\pi}{2},\frac{\pi}{2}\right)\) is given by \(x=\arcsin(a)\text{.}\) The other solution in the interval \(\left(-\pi,\pi\right)\) is given by \(x=\pi-\arcsin(a)\text{,}\) as shown in the unit circles below.So, the values of \(x\) for which \(x=\sin a\) are \(x=2\pi n+\arcsin(a)\) and \(x=2\pi n + \pi - \arcsin (a)\) for any integer \(n\text{.}\)
2.12.2.5.
Solution.
2.12.2.6.
Solution.
2.12.2.7.
Solution.
2.12.2.8.
Solution.
2.12.2.9.
Solution.
2.12.2.10.
Solution.
2.12.2.11.
Solution.
2.12.2.12.
Solution.
where \(x\) and \(\theta\) are the same in both expressions, and \(-\frac{\pi}{2} \leq \theta \leq \frac{\pi}{2}\text{.}\) Then
\begin{align*} \arcsin x &=\theta & \arccos x &= \frac{\pi}{2}-\theta \end{align*}2.12.2.13. (✳).
Solution.
2.12.2.14. (✳).
Solution.
2.12.2.15. (✳).
Solution.
2.12.2.16.
Solution.
2.12.2.17.
Solution.
2.12.2.18. (✳).
Solution.
2.12.2.19.
Solution.
2.12.2.20. (✳).
Solution.
Now, we can differentiate with respect to \(y\) using the chain rule.
\begin{align*} \diff{}{y}\left\{f(g(y))\right\}&=\diff{}{y}\{y\}\\ f'(g(y))\cdot g'(y)&=1\\ g'(y)&=\frac{1}{f'(g(y))}=\frac{1}{1-\sin g(y)} \end{align*}2.12.2.21. (✳).
Solution.
What we want is \(g'(\pi-1)\text{,}\) so we need to figure out which value of \(x\) gives \(f(x)=\pi-1\text{.}\) A little trial and error leads us to \(x=\frac{\pi}{2}\text{.}\)
\begin{align*} g'(\pi-1)\cdot f'\left(\frac{\pi}{2}\right)&=1\\ \end{align*}Since \(f'(x)=2-\cos(x)\text{,}\) \(f'\left(\frac{\pi}{2}\right)=2-0=2\text{:}\)
\begin{align*} g'(\pi-1)\cdot 2&=1\\ g'(\pi-1)=\frac{1}{2} \end{align*}2.12.2.22. (✳).
Solution.
What we want is \(g'(e+1)\text{,}\) so we need to figure out which value of \(x\) gives \(f(x)=e+1\text{.}\) A little trial and error leads us to \(x=1\text{.}\)
\begin{align*} g'(f(1))f'(1)&=1\\ g'(e+1)\cdot f'(1)&=1\\ g'(e+1) &= \frac{1}{f'(1)}\\ \end{align*}It remains only to note that \(f'(x)=e^x+1\text{,}\) so \(f'(1)=e+1\)
\begin{align*} g'(e+1)&=\frac{1}{e+1} \end{align*}2.12.2.23.
Solution.
2.12.2.24.
Solution.
2.12.2.25.
Solution.
2.12.2.26.
Solution.
and so
\begin{align*} 7(3x+7)&=5x-9\\ x&=-\frac{29}{8}\\ \end{align*}So, when \(x=-\dfrac{29}{8}\text{,}\) our equation \(f(2x+1)=\dfrac{5x-9}{3x+7}\) becomes:
\begin{align*} f\left(2\cdot\frac{-29}{8}+1\right)&=\dfrac{5\cdot\frac{-29}{8}-9}{3\cdot\frac{-29}{8}+7}\\ \end{align*}Or, equivalently:
\begin{align*} f\left(-\frac{25}{4}\right)&=7 \end{align*}2.12.2.27.
Solution.
Now, the equation \(f^{-1}(4x-1)=\dfrac{2x+3}{x+1}\) with \(x=\dfrac{-3}{2}\) tells us:
\begin{align*} f^{-1}\left(4\cdot\frac{-3}{2}-1\right)&=\frac{2\cdot\frac{-3}{2}+3}{\frac{-3}{2}+1}\\ \end{align*}Or, equivalently:
\begin{align*} f^{-1}(-7)&=0 \end{align*}2.12.2.28.
Solution.
- Solution 1: We begin by differentiating implicitly. Following the usual convention, we use \(y'\) to mean \(y'(x)\text{.}\) We start with\begin{align*} \arcsin(x+2y)&=x^2+y^2 \end{align*}Using the chain rule,\begin{align*} \frac{1+2y'}{\sqrt{1-(x+2y)^2}}&=2x+2yy'\\ \frac{1}{\sqrt{1-(x+2y)^2}}+\frac{2y'}{\sqrt{1-(x+2y)^2}}&=2x+2yy'\\ \frac{2y'}{\sqrt{1-(x+2y)^2}}-2yy'&=2x-\frac{1}{\sqrt{1-(x+2y)^2}}\\ y'\left(\frac{2}{\sqrt{1-(x+2y)^2}}-2y\right)&=2x-\frac{1}{\sqrt{1-(x+2y)^2}} \end{align*}Finally, solving for \(y'\) gives\begin{align*} y'&=\frac{2x-\frac{1}{\sqrt{1-(x+2y)^2}}}{\frac{2}{\sqrt{1-(x+2y)^2}}-2y} \left(\frac{\sqrt{1-(x+2y)^2}}{\sqrt{1-(x+2y)^2}}\right)\\ y'&=\frac{2x\sqrt{1-(x+2y)^2}-1}{2-2y\sqrt{1-(x+2y)^2}} \end{align*}
- Solution 2: We begin by taking the sine of both sides of the equation.\begin{align*} \arcsin(x+2y)&=x^2+y^2\\ x+2y&=\sin(x^2+y^2)\\ \end{align*}
Now, we differentiate implicitly.
\begin{align*} 1+2y'&=\cos(x^2+y^2)\cdot(2x+2yy')\\ 1+2y'&=2x\cos(x^2+y^2)+2yy'\cos(x^2+y^2)\\ 2y'-2yy'\cos(x^2+y^2)&=2x\cos(x^2+y^2)-1\\ y'\left(2-2y\cos(x^2+y^2)\right)&=2x\cos(x^2+y^2)-1\\ y'&=\frac{2x\cos(x^2+y^2)-1}{2-2y\cos(x^2+y^2)} \end{align*} -
We used two different methods, and got two answers that look pretty different. However, the answers ought to be equivalent. To see this, we remember that for all values of \(x\) and \(y\) that we care about (those pairs \((x,y)\) in the domain of our curve), the equality\begin{equation*} \arcsin(x+2y)=x^2+y^2 \end{equation*}holds. Drawing a triangle:where the adjacent side (in red) come from the Pythagorean Theorem. Then, \(\cos(x^2+y^2)=\sqrt{1-(x+2y)^2}\text{,}\) so using our second solution:\begin{align*} y'&=\frac{2x\cos(x^2+y^2)-1}{2-2y\cos(x^2+y^2)}\\ &=\frac{2x\sqrt{1-(x+2y)^2}-1}{2-2y\sqrt{1-(x+2y)^2}} \end{align*}which is exactly the answer from our first solution.
2.13 The Mean Value Theorem
2.13.5 Exercises
2.13.5.1.
Solution.
2.13.5.2.
Solution.
2.13.5.3.
Solution.
2.13.5.4.
Solution.
2.13.5.5.
Solution.
2.13.5.6.
Solution.
2.13.5.7. (✳).
Solution.
2.13.5.8. (✳).
Solution.
2.13.5.9. (✳).
Solution.
2.13.5.10. (✳).
Solution.
2.13.5.11.
Solution.
2.13.5.12.
Solution.
- As \(x\) tends to \(\pm \infty\text{,}\) \(f(x)\) tends to \(+\infty\text{.}\)
- The derivative \(f'(x)=(4x+1)^3+1\) is negative for \(x \lt -\frac{1}{2}\) and is positive for \(x \gt -\frac{1}{2}\text{.}\) That is, \(f(x)\) is decreasing for \(x \lt -\frac{1}{2}\) and increasing for \(x \gt -\frac{1}{2}\text{.}\)
- \(f\left(-\frac{1}{2}\right)=\frac{1}{16}-\frac{1}{2} \lt 0\text{.}\)
- When \(x\) is large and negative, \(f(x) \gt 0\text{.}\)
- As \(x\) increases, \(f(x)\) decreases continuously until \(x=-\frac{1}{2}\text{,}\) where \(f(x) \lt 0\text{.}\) In particular, since \(f(-1)=\frac{65}{16} \gt 0\) and \(f\big(-\frac{1}{2}\big) \lt 0\) and \(f(x)\) is continuous, the intermediate value theorem guarantees that \(f(x)\) takes the value zero for some \(x\) between \(-\frac{1}{2}\) and \(-1\text{.}\) More descriptively put, as \(x\) increases from hugely negative numbers to \(-\frac{1}{2}\text{,}\) \(f(x)\) passes through zero exactly once.
- As \(x\) increases beyond \(-\frac{1}{2}\text{,}\) \(f(x)\) increases continuously, starting negative and becoming very large and positive when \(x\) becomes large and positive. In particular, since \(f(0)=\frac{1}{16} \gt 0\) and \(f\big(-\frac{1}{2}\big) \lt 0\) and \(f(x)\) is continuous, the intermediate value theorem guarantees that \(f(x)\) takes the value zero for some \(x\) between \(-\frac{1}{2}\) and \(0\text{.}\) So, as \(x\) increases from \(-\frac{1}{2}\) to near \(+\infty\text{,}\) \(f(x)\) again passes through zero exactly once.
2.13.5.13.
Solution.
- We can see by inspection that \(f(0)=0\text{,}\) so there is at least one root. We have to determine how many any other roots there are, if any.
- The function \(f(x)\) is the sum of the two terms \(x^3\) and \(\sin(x^5)\text{.}\) A number \(x\) is a root of \(f(x)\) if and only if the two terms cancel each other exactly for that value of \(x\text{.}\) That is, \(x\) is a root of \(f\) if and only if \(x^3=-\sin(x^5)\text{.}\) To develope some intuition in our hunt for other roots, we sketch, in the same figure, the graphs \(y=x^3\) and \(y=-\sin(x^5)\text{.}\) Then the roots of \(f(x)\) are precisely the \(x\)’s where the two curves \(y=x^3\) and \(y= -\sin(x^5)\) intersect.
- Looking at the sketch, we see that the two curves cannot possibly intersect at any point having \(|x|>1\) — if \(|x|>1\text{,}\) then \(|x^3|>1\) but \(|-\sin (x^5)|\le 1\) and we cannot possibly have \(x^3=-\sin(x^5)\text{.}\) That is, all roots of \(f(x)\) are in the interval \([-1,1]\text{.}\)
- From the sketch, we would probably guess that \(y=x^3\) and \(y=-\sin(x^5)\) cross only at \(x=0\text{.}\) We can use Rolle’s Theorem to verify that that is indeed the case.
- If there is a root \(a \neq 0\text{,}\) then by Rolle’s Theorem (since \(f(x)\) is continuous and differentiable for all real numbers \(x\)) \(f'(c)=0\) for some \(c\) strictly between \(0\) and \(a\text{.}\) In particular, since we already know any roots \(a\) will be between \(-1\) and \(1\text{,}\) if \(f(x)\) has two roots then \(f'(c)=0\) for some \(c \in (-1,0) \cup (0,1)\text{.}\)
- The derivative of \(f\) is\begin{equation*} f'(x)=3x^2+5x^4\cos\big(x^5\big)=x^2\left(3+5x^2\cos\big(x^5\big)\right) \end{equation*}So, if \(f'(x)=0\text{,}\) then \(x=0\) or \(\left(3+5x^2\cos\left(x^5\right)\right)=0\text{.}\) If \(x \in (-1,0) \cup (0,1)\text{,}\) then\begin{align*} |x|\lt 1 \amp\implies |x^5|\lt 1\lt \tfrac{\pi}{2} \implies \cos\big(x^5\big)>0 \\ \amp\implies 3+5x^2\cos\big(x^5\big) > 3 \\ \amp\implies f'(x) \neq 0 \end{align*}That is, there is no \(c \in (-1,0) \cup (0,1)\) with \(f'(c)=0\text{.}\) Therefore, following our last bullet point, \(f(x)\) has only one root.
2.13.5.14.
Solution.
2.13.5.15. (✳).
Solution.
2.13.5.16. (✳).
Solution.
The problem asks us to find this value of \(c\text{.}\) Solving:
\begin{align*} e^c&=\frac{e^T-e^0}{T}\\ e^c&=\frac{e^T-1}{T}\\ c&=\log\left(\frac{e^T-1}{T}\right) \end{align*}2.13.5.17.
Solution.
2.13.5.18. (✳).
Solution.
2.13.5.19.
Solution.
2.13.5.20.
Solution.
2.13.5.21.
Solution.
2.13.5.22.
Solution.
2.13.5.23.
Solution.
- \(f(x)\) is continuous over all real numbers
- \(f(x)=0\) only when \(-\sin x=x^2+5x+1\text{,}\) which only happens when \(|x^2+5x+1| \leq 1\text{.}\) Thus, \(f(x)\) only has roots in the intervals \(\left[-5,\frac{-5-\sqrt{17}}{2}\right]\) and \(\left[\frac{-5+\sqrt{17}}{2},0\right]\text{.}\)
- \(f(-5)=\sin(-5)+1 \gt 0\text{,}\) and \(f\left(\frac{-5-\sqrt{17}}{2}\right)=\sin\left(\frac{-5-\sqrt{17}}{2}\right)-1 \lt 0\text{.}\) So, by the IVT, \(f(c)=0\) for some \(c \in \left(-5,\frac{-5-\sqrt{17}}{2}\right)\text{.}\)
- \(f(0)=1 \gt 0\text{,}\) and \(f\left(\frac{-5+\sqrt{17}}{2}\right)=\sin\left(\frac{-5+\sqrt{17}}{2}\right)-1 \lt 0\text{.}\) So, by the IVT, \(f(c)=0\) for some \(c \in \left(\frac{-5+\sqrt{17}}{2},0\right)\text{.}\)
- \(f'(x)=\cos x + 2x + 5\text{.}\) If \(f'(x)=0\text{,}\) then \(2x+5=-\cos x\text{,}\) so \(|2x+5| \leq 1\text{.}\) So, the only interval that can contain roots of \(f'(x)\) is \([-3,-2]\text{.}\)
-
Suppose \(f(x)\) has more than two roots. Then it has two roots in the interval \(\left[-5,\frac{-5-\sqrt{17}}{2}\right]\) OR it has two roots in the interval \(\left[\frac{-5+\sqrt{17}}{2},0\right]\text{.}\) Since \(f(x)\) is differentiable for all real numbers, Rolle’s Theorem tells us that \(f'(x)\) has a root in \(\left(-5,\frac{-5-\sqrt{17}}{2}\right)\) or in \(\left(\frac{-5+\sqrt{17}}{2},0\right)\text{.}\) However, since all roots of \(f'(x)\) are in the interval \([-3,-2]\text{,}\) and this interval shares no points with \(\left(-5,\frac{-5-\sqrt{17}}{2}\right)\) or \(\left(\frac{-5+\sqrt{17}}{2},0\right)\text{,}\) this cannot be the case. Therefore \(f(x)\) does not have more than two roots.
- Since \(f(x)\) has at least two roots, and not more than two roots, \(f(x)\) has exactly two roots.
2.14 Higher Order Derivatives
2.14.2 Exercises
2.14.2.1.
Solution.
2.14.2.2.
Solution.
2.14.2.3.
Solution.
2.14.2.4.
Solution.
Plugging in \(x=1\text{,}\) \(y=3\text{:}\)
\begin{align*} -28+6+2y'+6y'&=0\\ y'&=\frac{11}{4} \quad\mbox{\textcolor{red}{ at the point $(1,3)$}}\\ \end{align*}Differentiating the equation \(-28x+2y+2xy'+2yy'=0\):
\begin{align*} -28+2y'+2y'+2xy''+2y'y'+2yy''&=0\\ 4y'+2(y')^2+2xy''+2yy''&=28\\ \end{align*}At the point \((1,3)\text{,}\) \(y'=\dfrac{11}{4}\text{.}\) Plugging in:
\begin{align*} 4\left(\frac{11}{4}\right)+2\left(\frac{11}{4}\right)^2+2(1)y''+2(3)y''&=28\\ y''&=\frac{15}{64} \end{align*}2.14.2.5.
Solution.
2.14.2.6.
Solution.
2.14.2.7.
Solution.
So, we need an expression for \(y'\text{.}\) We use the equation \(2x+2yy'=0\) to conclude \(y'=-\dfrac{x}{y}\text{:}\)
\begin{align*} y''&=-\frac{\left(-\frac{x}{y}\right)^2+1}{y}\\ &=-\frac{\frac{x^2}{y^2}+1}{y}\\ &=-\frac{x^2+y^2}{y^3}\\ &=-\frac{1}{y^3} \end{align*}2.14.2.8.
Solution.
2.14.2.9.
Solution.
Using the quotient rule:
\begin{align*} \ddiff{2}{}{x}\{\log(5x^2-12)\}&=\diff{}{x}\left\{\frac{10x}{5x^2-12}\right\}\\ &=\frac{(5x^2-12)(10)-10x(10x)}{(5x^2-12)^2}\\ &=\frac{-10(5x^2+12)}{(5x^2-12)^2} \end{align*}2.14.2.10.
Solution.
2.14.2.11.
Solution.
2.14.2.12.
Solution.
We differentiate implicitly. For ease of notation, we write \(y'\) for \(\ds\diff{y}{x}\text{.}\)
\begin{align*} 2x+1+y'&=\cos(xy)(y+xy')\\ \end{align*}We’re interested in \(y''\text{,}\) so we implicitly differentiate again.
\begin{align*} 2+y''&=-\sin(xy)(y+xy')^2+\cos(xy)(2y'+xy'')\\ \end{align*}We want to know what \(y''\) is when \(x=y=0\text{.}\) Plugging these in yields the following:
\begin{align*} 2+y''&=2y'\\ \end{align*}So, we need to know what \(y'\) is when \(x=y=0\text{.}\) We can get this from the equation \(2x+1+y'=\cos(xy)(y+xy')\text{,}\) which becomes \(1+y'=0\) when \(x=y=0\text{.}\) So, at the origin, \(y'=-1\text{,}\) and
\begin{align*} 2+y''&=2(-1)\\ y''&=-4 \end{align*}2.14.2.13.
Solution.
The fourth derivative is \(\sin x\) is \(\sin x\text{,}\) and the fourth derivative of \(\cos x\) is \(\cos x\text{,}\) so (a) and (b) are true.
\begin{align*} \diff{}{x}\tan x &=\sec^2 x\\ \diff{}{x}\sec^2 x &=2\sec x (\sec x \tan x)=2\sec^2x\tan x\\ \diff{}{x}\{2\sec^2x\tan x\}&=(4\sec x \cdot \sec x \tan x)\tan x+2\sec^2x\sec^2x\\ &=4\sec^2x\tan^2x+2\sec^4x \end{align*}2.14.2.14.
Solution.
2.14.2.15.
Solution.
Every time we differentiate, we multiply the original function by another factor of \(\log 2\text{.}\) So, the \(n\)th derivative is given by:
\begin{align*} \ddiff{n}{}{x}\{2^x\}&=2^x(\log2)^n \end{align*}2.14.2.16.
Solution.
2.14.2.17. (✳).
Solution.
- 2.14.2.17.c \(f\) and \(h\) “start at the same place”, since \(f(0)=h(0)\text{.}\) If it were clear that \(f'(x)\) were greater than \(h'(x)\) for \(x \gt 0\text{,}\) then we would know that \(f\) grows faster than \(h\text{,}\) so we could conclude that \(f(x) \gt h(x)\text{,}\) as desired. Unfortunately, it is not obvious whether \((1+2x)e^{x+x^2}\) is always greater than \(1+3x\) for positive \(x\text{.}\) So, we look to the second derivative. \(f'(0)=h'(0)\text{,}\) and \(f''(x)=(4x^2+4x+3)e^{x+x^2} \gt 3e^{x+x^2} \gt 3=h''(x)\) when \(x \gt 0\text{.}\) Since \(f'(0)=h'(0)\text{,}\) and since \(f'\) grows faster than \(h'\) for positive \(x\text{,}\) we conclude \(f'(x) \gt h'(x)\) for all positive \(x\text{.}\) Now we can conclude that (since \(f(0)=h(0)\) and \(f\) grows faster than \(h\) when \(x \gt 0\)) also \(f(x) \gt h(x)\) for all positive \(x\text{.}\)
2.14.2.18. (✳).
Solution.
- We differentiate implicitly.\begin{align*} x^3y(x)+y(x)^3&=10 x\\ 3x^2y(x)+x^3y'(x)+3y(x)^2y'(x)&=10\\ \end{align*}
Subbing in \(x=1\) and \(y(1)=2\) gives
\begin{align*} (3)(1)( 2)+(1)y'(1)+(3)(4) y'(1)&=10\\ 13y'(1)&=4\\ y'(1)&=\frac{4}{13} \end{align*} -
From part 2.14.2.18.a, the slope of the curve at \(x=1,\ y=2\) is \(\dfrac{4}{13}\text{,}\) so the curve is increasing, but fairly slowly. The angle of the tangent line is \(\tan^{-1}\left(\frac{4}{13}\right)\approx 17^\circ\text{.}\) We are also told that \(y''(1) \lt 0\text{.}\) So the slope of the curve is decreasing as \(x\) passes through 1. That is, the line is more steeply increasing to the left of \(x=1\text{,}\) and its slope is decreasing (getting less steep, then possibly the slope even becomes negative) as we move past \(x=1\text{.}\)
2.14.2.19.
Solution.
- In the first derivative of \(g\text{,}\) the coefficients of \(f\) and \(f'\) correspond to the entries in the second row of Pascal’s Triangle.
- In the second derivative of \(g\text{,}\) the coefficients of \(f\text{,}\) \(f'\text{,}\) and \(f''\) correspond to the entries in the third row of Pascal’s Triangle.
- In the third derivative of \(g\text{,}\) the coefficients of \(f\text{,}\) \(f'\text{,}\) \(f''\text{,}\) and \(f'''\) correspond to the entries in the fourth row of Pascal’s Triangle.
- We guess that, in the fourth derivative of \(g\text{,}\) the coefficients of \(f\text{,}\) \(f'\text{,}\) \(f''\text{,}\) \(f'''\text{,}\) and \(f^{(4)}\) will correspond to the entries in the fifth row of Pascal’s Triangle.
2.14.2.20.
Solution.
2.14.2.21.
Solution.
- Let’s begin by noticing that the domain of \(f(x)\) is \((-1,\infty)\text{.}\)
- By inspection, \(f(0)=0\text{,}\) so \(f(x)\) has at least one root.
- If \(x\in(-1,0)\text{,}\) then \((x+1)\) is positive, \(\log(x+1)\) is negative, \(\sin(x)\) is negative, and \(-x^2\) is negative. Therefore, if \(x \lt 0\) is in the domain of \(f\text{,}\) then \(f(x) \lt 0\text{.}\) So, \(f(x)\) has no negative roots. We focus our attention on the case \(x \gt 0\text{.}\)
- \(f'(x)=1-2x+\log(x+1)+\cos x\text{.}\) We would like to know how many positive roots \(f'(x)\) has, but it isn’t obvious. So, let’s differentiate again.
- \(f''(x)=-2+\frac{1}{x+1}-\sin x\text{.}\) When \(x \gt 0\text{,}\) \(\frac{1}{x+1} \lt 1\text{,}\) so \(f''(x) \lt -1-\sin(x) \leq 0\text{,}\) so \(f''(x)\) has no positive roots. Since \(f'(x)\) is continuous and differentiable over \((0,\infty)\text{,}\) and since \(f''(x) \neq 0\) for all \(x \in (0, \infty)\text{,}\) by Rolle’s Theorem, \(f'(x)\) has at most one root in \([0,\infty)\text{.}\)
- Since \(f(x)\) is continuous and differentiable over \([0,\infty)\text{,}\) and \(f'(x)\) has at most one root in \((0,\infty)\text{,}\) by Rolle’s Theorem \(f(x)\) has at most two distinct roots in \([0,\infty)\text{.}\) (Otherwise, \(f(a)=f(b)=f(c)=0\) for some values \(0\le a \lt b \lt c\text{,}\) so \(f'(d)=f'(e)=0\) for some \(d\in(a,b)\) and some \(e \in (b,c)\text{,}\) but since \(f'(x)\) has at most one root, this is impossible.)
-
We know \(f(0)=0\text{,}\) so the remaining question is whether or not \(f(x)\) has a second root (which would have to be positive). As usual, we can show another root exists using the intermediate value theorem. We see that for large values of \(x\text{,}\) \(f(x)\) is negative, for example:\begin{equation*} f(4)=5\log 5+\sin(4)-(4)^2 \lt 5\log(e^2) + 1 -16 =11-16 \lt 0 \end{equation*}For positive values of \(x\) closer to zero, we hope to find a positive value of \(f(x)\text{.}\) However, it’s quite difficult to get a number \(c\) that obviously gives \(f(c) \gt 0\text{.}\) It suffices to observe that \(f(0)=0\) and \(f'(0)=2 \gt 0\text{.}\) From the definition of the derivative, we can conclude \(f(x) \gt 0\) for some \(x \gt 0\text{.}\) (If it is not true that \(f(x) \gt 0\) for some \(x \gt 0\text{,}\) then \(f(x)\le 0\) for all \(x \gt 0\text{.}\) The definition of the derivative tells us that [since \(f'(0)\) exists] \(f'(0)=\ds\lim_{h \to 0^+}\frac{f(h)-f(0)}{h}=\ds\lim_{h \to 0^+}\frac{f(h)}{h}\text{;}\) the denominator is positive, so if the numerator were always less than or equal to zero, the limit would be less than or equal to zero as well. However, the derivative is positive, so \(f(x) \gt 0\) for some \(x \gt 0\text{.}\)) Therefore, \(f(x)\) has a second root, so \(f(x)\) has precisely two roots.
2.14.2.22. (✳).
Solution.
Since \(f\) looks different to the left and right of 0, in order to evaluate this limit, we look at the corresponding one-sided limits. Note that when \(h\) approaches 0 from the right, \(h \gt 0\) so \(f(h)=h^2\text{.}\) By contrast, when \(h\) approaches 0 from the left, \(h \lt 0\) so \(f(h)=-h^2\text{.}\)
\begin{align*} &\;\; \lim_{h \to 0^+} \frac{f(h)}{h}=\lim_{h \to 0^+}\frac{h^2}{h}=\lim_{h \to 0^+}h=0\\ &\;\; \lim_{h \to 0^-} \frac{f(h)}{h}=\lim_{h \to 0^-}\frac{-h^2}{h}=\lim_{h \to 0^-}-h=0\\ \end{align*}Since both one-sided limits exist and are equal to 0,
\begin{align*} &\;\; \lim_{h \to 0} \frac{f(0+h)-f(0)}{h}=0 \end{align*}exists, then \(f''(0)\) exists.
\begin{align*} \lim_{h \to 0}\frac{f'(0+h)-f'(0)}{h}&=\lim_{h \to 0} \frac{f'(h)-0}{h}=\lim_{h \to 0}\frac{f'(h)}{h}\\ \end{align*}Since \(f(h)\) behaves differently when \(h\) is greater than or less than zero, we look at the one-sided limits.
\begin{align*} \lim_{h \to 0^+}\frac{f'(h)}{h}&=\lim_{h \to 0^+}\frac{2h}{h}=2\\ \lim_{h \to 0^-}\frac{f'(h)}{h}&=\lim_{h \to 0^-}\frac{-2h}{h}=-2\\ \end{align*}Since the one-sided limits do not agree,
\begin{align*} \lim_{h \to 0}\frac{f'(0+h)-f'(0)}{h}&=DNE \end{align*}
3 Applications of derivatives
3.1 Velocity and Acceleration
3.1.2 Exercises
3.1.2.1.
Solution.
3.1.2.2.
Solution.
3.1.2.3.
Solution.
3.1.2.4.
Solution.
3.1.2.5.
Solution.
The velocity of the pot at this time is
\begin{align*} h'\left(\sqrt{\frac{20}{9.8}}\right)&=-9.8\left(\sqrt{\frac{20}{9.8}}\right)=-\sqrt{20\cdot 9.8} =-14 \frac{\mathrm{m}}{\mathrm{s}} \end{align*}3.1.2.6.
Solution.
- Let \(s(t)\) be the distance the stone has fallen \(t\) seconds after dropping it. Since the acceleration due to gravity is \(9.8\;\frac{\mathrm{m}}{\mathrm{s}^2}\text{,}\) \(s''(t)=9.8\text{.}\) (We don’t make this negative, because \(s(t)\) measures how far the stone has fallen, which means the positive direction in our coordinate system is “down”, which is exactly the way gravity is pulling.)
- Then \(s'(t)\) has a constant derivative of 9.8, so \(s'(t)=9.8t+c\) for some constant \(c\text{.}\) Notice \(s'(0)=c\text{,}\) so \(c\) is the velocity of the stone at the very instant you dropped it, which is zero. Therefore, \(s'(t)=9.8t\text{.}\)
-
So, \(s(t)\) is a function with derivative \(9.8t\text{.}\) It’s not too hard to figure out by guessing and checking that \(s(t)=\frac{9.8}{2}t^2+d\) for some constant \(d\text{.}\) Notice \(s(0)=d\text{,}\) so \(d\) is the distance the rock has travelled at the instant you dropped it, which is zero. So, \(s(t)=\frac{9.8}{2}t^2=4.9t^2\text{.}\)Remark: this is exactly the formula found in Example 3.1.2. You may, in general, use that formula without proof, but you need to know where it comes from and be able to apply it in other circumstances where it might be slightly different--like part (b) below.
-
The rock falls for \(x\) seconds, so the distance fallen is\begin{equation*} 4.9x^2 \end{equation*}Remark: this is a decent (if imperfect) way to figure out how deep a well is, or how tall a cliff is, when you’re out and about. Drop a rock, square the time, multiply by 5.
3.1.2.7.
Solution.
3.1.2.8.
Solution.
3.1.2.9.
Solution.
Multiplying both sides by \(d\text{:}\)
\begin{align*} \frac{d}{10}&=120^2-\frac{120^2}{2}\\ d&=5\cdot 120^2 \end{align*}3.1.2.10.
Solution.
3.1.2.11.
Solution.
3.1.2.12.
Solution.
https://www.guinnessworldrecords.com/world-records/fastest-baseball-pitch-(male)
3.1.2.13.
Solution.
3.1.2.14.
Solution.
3.1.2.15.
Solution.
3.1.2.16.
Solution.
-
Let’s consider (1) first, the time during your jump before your feet touch the trampoline. Let \(t=0\) be the moment you jump, and let the rim of the trampoline be height \(0\text{.}\) Then, since your initial velocity was (positive) \(1\) meter per second, your height is given by\begin{equation*} h_1(t)=\frac{-9.8}{2}t^2+t=t\left(\frac{-9.8}{2}t+1\right) \end{equation*}Notice that, because your acceleration is working against your positive velocity, it has a negative sign.
-
We’ll need to know your velocity when your feet first touch the trampoline on your fall. The time your feet first first touch the trampoline after your jump is precisely when \(h_1(t)=0\) and \(t \gt 0\text{.}\) That is, when \(t=\frac{2}{9.8}\text{.}\) Now, since \(h'(t)=-9.8t+1\text{,}\) \(h'\left(\frac{2}{9.8}\right)=-9.8\left(\frac{2}{9.8}\right)+1=-1\text{.}\) So, you are descending at a rate of 1 metre per second at the instant your feet touch the trampoline.Remark: it is not only coincidence that this was your initial speed. Think about the symmetries of parabolas, and conservation of energy.
-
Now we need to think about your height as the trampoline is slowing your fall. One thing to remember about our general equation \(\frac{a}{2}t^2+vt+h\) is that \(v\) is the velocity when \(t=0\text{.}\) But, you don’t hit the trampoline at \(t=0\text{,}\) you hit it at \(t=\frac{2}{9.8}\text{.}\) In order to keep things simple, let’s use a different time scale for this second part of your journey. Let’s let \(h_2(T)\) be your height at time \(T\text{,}\) from the moment your feet touch the trampoline skin (\(T=0\)) to the bottom of your fall. Now, we can use the fact that your initial velocity is \(-1\) metres per second (negative, since your height is decreasing) and your acceleration is \(4.9\) metres per second per second (positive, since your velocity is increasing from a negative number to zero):\begin{equation*} h_2(T)=\frac{4.9}{2}T^2-T \end{equation*}where still the height of the rim of the trampoline is taken to be zero.Remark: if it seems very confusing that your free-falling acceleration is negative, while your acceleration in the trampoline is positive, remember that gravity is pushing you down, but the trampoline is pushing you up.
- How long were you falling in the trampoline? The equation \(h_2(T)\) tells you your height only as long as the trampoline is slowing your fall. You reach the bottom of your fall when your velocity is zero.\begin{equation*} h_2'(T)=4.9T-1 \end{equation*}so you reach the bottom of your fall at \(T=\frac{1}{4.9}\text{.}\) Be careful: this is \(\frac{1}{4.9}\) seconds after you entered the trampoline, not after the peak of your fall, or after you jumped.
-
The last piece of the puzzle is how long it took you to fall from the peak of your jump to the surface of the trampoline. We know the equation of your motion during that time: \(h_1(t)=\frac{-9.8}{2}t^2+t\text{.}\) You reached the peak when your velocity was zero:\begin{equation*} h_1'(t)=-9.8t+1 =0\qquad\Rightarrow\qquad t=\frac{1}{9.8} \end{equation*}So, you fell from your peak at \(t=\frac{1}{9.8}\) and reached the level of the trampoline rim at \(t=\frac{2}{1.98}\text{,}\) which means the fall took \(\frac{1}{9.8}\) seconds.Remark: by the symmetry mentioned early, the time it took to fall from the peak of your jump to the surface of the trampoline is the same as one-half the time from the moment you jumped off the rim to the moment you’re back on the surface of the trampoline.
- So, your time falling from the peak of your jump to its bottom was \(\frac{1}{9.8}+\frac{1}{4.9}\approx 0.3\) seconds.
3.1.2.17.
Solution.
- \(v(0)\) is some value, call it \(v_0\text{,}\)
- \(v(1)=2v_0\) (since the speed doubled in the first second),
- \(v(2)=2(2)v_0\) (since the speed doubled in the second second),
- \(v(3)=2(2)(2)v_0\text{,}\) and so on.
3.2 Related Rates
3.2.2 Exercises
3.2.2.1.
Solution.
We differentiate implicitly with respect to a third variable, \(t\text{:}\)
\begin{align*} \diff{P}{t}&=3Q^2\cdot\diff{Q}{t} \end{align*}3.2.2.2. (✳).
Solution.
3.2.2.3. (✳).
Solution.
3.2.2.4. (✳).
Solution.
3.2.2.5. (✳).
Solution.
- The distance \(z(t)\) between the particles at any moment in time is\begin{align*} z^2(t)&= x(t)^2+y(t)^2, \end{align*}where \(x(t)\) is the position on the \(x\)-axis of the particle A at time \(t\) (measured in seconds) and \(y(t)\) is the position on the \(y\)-axis of the particle B at the same time \(t\text{.}\)
- We differentiate the above equation with respect to \(t\) and get\begin{align*} 2z \cdot z' &= 2x \cdot x' + 2 y \cdot y', \end{align*}
- We are told that \(x'=-2\) and \(y'=-3\text{.}\) (The values are negative because \(x\) and \(y\) are decreasing.) It will take \(3\) seconds for particle \(A\) to reach \(x=4\text{,}\) and in this time particle \(B\) will reach \(y=3\text{.}\)
- At this point \(z = \sqrt{x^2+y^2}=\sqrt{3^2+4^2}=5\text{.}\)
- Hence\begin{align*} 10z' &= 8\cdot(-2)+6\cdot(-3) = -34\\ z' &= -\frac{34}{10} = -\frac{17}{5} \text{ units per second}. \end{align*}
3.2.2.6. (✳).
Solution.
- We compute the distance \(z(t)\) between the two particles after \(t\) seconds as\begin{align*} z^2(t)&= 3^2 + (y_A(t)-y_B(t))^2, \end{align*}where \(y_A(t)\) and \(y_B(t)\) are the \(y\)-coordinates of particles \(A\) and \(B\) after \(t\) seconds, and the horizontal distance between the two particles is always 3 units.
- We are told the distance between the particles is 5 units, this happens when\begin{align*} (y_A-y_B)^2 &= 5^2-3^2 = 16\\ y_A-y_B&=4 \end{align*}That is, when the difference in \(y\)-coordinates is \(4\text{.}\) This happens when \(t=4\text{.}\)
- We differentiate the distance equation (from the first bullet point) with respect to \(t\) and get\begin{align*} 2z \cdot z' &= 2 ({y_A}' - {y_B}') (y_A - y_B), \end{align*}
- We know that \((y_A-y_B)=4\text{,}\) and we are told that \(z=5\text{,}\) \(y_A'=3\text{,}\) and \(y_B'=2\text{.}\) Hence\begin{align*} 10z'(4) &= 2 \times1\times4 = 8 \end{align*}
- Therefore\begin{align*} z'(4) &= \frac{8}{10} = \frac{4}{5} \text{ units per second}. \end{align*}
3.2.2.7. (✳).
Solution.
Differentiating with respect to \(t\text{,}\)
\begin{align*} 2x(t)x'(t)+2y(t)y'(t)&=2z(t)z'(t)\\ x(t)x'(t)+y(t)y'(t)&=z(t)z'(t)\\ \end{align*}At the specified time, \(x(t)\) is decreasing, so \(x'(t)\) is negative, and \(y(t)\) is increasing, so \(y'(t)\) is positive.
\begin{align*} (300)(-15)+(400)(20) &= \sqrt{300^2+400^2}z'(t)\\ 500 z'(t)&=3500\\ z'(t)&=7 \mbox{ mph} \end{align*}3.2.2.8. (✳).
Solution.
- We compute the distance \(d(t)\) between the two snails after \(t\) minutes as\begin{align*} d^2(t)&= 30^2 + (y_1(t)-y_2(t))^2, \end{align*}where \(y_1(t)\) is the altitude of the first snail, and \(y_2(t)\) the altitude of the second snail after \(t\) minutes.
- We differentiate the above equation with respect to \(t\) and get\begin{align*} 2d \cdot d' &= 2 ({y_1}' - {y_2}') (y_1 - y_2)\\ d \cdot d' &= ({y_1}' - {y_2}') (y_1 - y_2) \end{align*}
- We are told that \({y_1}'=25\) and \({y_2}'=15\text{.}\) It will take \(4\) minutes for the first snail to reach \(y_1=100\text{,}\) and in this time the second snail will reach \(y_2=60\text{.}\)
- At this point \(d^2 = 30^2 + (100-60)^2 = 900 + 1600 = 2500\text{,}\) hence \(d = 50\text{.}\)
- Therefore\begin{align*} 50 d' &= (25 - 15) \times (100 - 60)\\ d' &= \frac{400}{50} = 8 \text{ cm per minute}. \end{align*}
3.2.2.9. (✳).
Solution.
- If we write \(z(t)\) for the length of the ladder at time \(t\) and \(y(t)\) for the height of the top end of the ladder at time \(t\) we have\begin{equation*} z(t)^2 = 5^2 + y(t)^2. \end{equation*}
- We differentiate the above equation with respect to \(t\) and get\begin{align*} 2z \cdot z' &= 2y \cdot y', \end{align*}
- We are told that \(z'(t)=-2\text{,}\) so \(z(3.5)=20-3.5\cdot2 = 13\text{.}\)
- At this point \(y = \sqrt{z^2-5^2} = \sqrt{169-25} = \sqrt{144} = 12\text{.}\)
- Hence\begin{align*} 2\cdot 13 \cdot (-2) &= 2 \cdot 12 y'\\ y' &= -\frac{2\cdot 13}{12} = -\frac{13}{6} \text{ meters per second}. \end{align*}
3.2.2.10.
Solution.
so
\begin{align*} V&=100h(60+w)\\ &=100h(120+\frac{4}{5}h)\\ &=80h^2+12000h\\ \end{align*}This is the equation we need, relating \(V\) and \(h\text{.}\) Differentiating implicitly with respect to \(t\text{:}\)
\begin{align*} \diff{V}{t} &=2\cdot80h\cdot\diff{h}{t}+12000\diff{h}{t}\\ &=\left(160h+12000\right)\diff{h}{t}\\ \end{align*}We are given that \(h=25\) and \(\ds\diff{V}{t}=3\) litres per minute. Converting to cubic centimetres, \(\ds\diff{V}{t} = -3000\) cubic centimetres per minute. So:
\begin{align*} -3000&=\left(160\cdot25+12000\right)\diff{h}{t}\\ \diff{h}{t}&=-\frac{3}{16}=-.1875\;\frac{\mathrm{cm}}{\mathrm{min}} \end{align*}3.2.2.11.
Solution.
- The triangle on the left has height \(h\) metres. Let its base be \(a\) metres. It forms a similar triangle with the triangle whose height is 1.25 metres and width is 1 metre, so:\begin{align*} \frac{a}{h}&=\frac{1}{1.25}\\ a&=\frac{4}{5}h\\ \end{align*}
So, the area of the triangle on the left is
\begin{align*} \frac{1}{2}ah&=\frac{2}{5}h^2 \end{align*} - The rectangle in the middle has length 3 metres and height \(h\) metres, so its area is \(3h\) square metres.
- The trapezoid on the right is a portion of a triangle with base 3 metres and height 1.25 metres. So, its area is\begin{align*} &\underbrace{\left(\frac{1}{2}(3)(1.25)\right) }_{\mbox{area of big triangle}}- \underbrace{\left(\frac{1}{2}(b)(1.25-h)\right)}_{\mbox{area of little triangle}}\\ \end{align*}
The little triangle (of base \(b\) and height \(1.25-h\)) is formed by the air on the right side of the tank. It is a similar triangle to the triangle of base 3 and height 1.25, so
\begin{align*} \frac{b}{1.25-h}&=\frac{3}{1.25}\\ b&=\frac{3}{1.25}(1.25-h)\\ \end{align*}So, the area of the trapezoid on the right is
\begin{align*} &\frac{1}{2}(3)(1.25)-\frac{1}{2}\left(\frac{3}{1.25}\right)\left(1.25-h\right)(1.25-h)\\ &=3h-\frac{6}{5}h^2 \end{align*}
So, the volume of water is
\begin{align*} V&=5\left(6h-\frac{4}{5}h^2\right)=30h-4h^2\\ \end{align*}Differentiating with respect to time, \(t\text{:}\)
\begin{align*} \diff{V}{t}&=30\diff{h}{t}-8h\diff{h}{t}\\ \end{align*}When \(h=\dfrac{1}{10}\) metre, and \(\ds\diff{V}{t}=\frac{1}{10^3}\) cubic metres per second,
\begin{align*} \frac{1}{10^3}&=30\diff{h}{t}-8\left(\frac{1}{10}\right)\diff{h}{t}\\ \diff{h}{t}&=\frac{1}{29200} \mbox{ metres per second} \end{align*}3.2.2.12.
Solution.
Differentiating with respect to \(t\text{:}\)
\begin{align*} \sec^2\theta \cdot \diff{\theta}{t}&=\frac{1}{2}\diff{h}{t}\\ \diff{\theta}{t}&=\frac{1}{2}\cos^2\theta\cdot\diff{h}{t} \end{align*}So, the quantities we need to know one minute after liftoff (that is, when \(t = \dfrac{1}{60}\)) are \(h\left(\dfrac{1}{60}\right)\) and \(\ds\diff{h}{t}\left(\frac{1}{60}\right)\text{.}\) Recall \(h(t)=61750t^2\text{.}\)
\begin{align*} h\left(\frac{1}{60}\right)&=\frac{61750}{3600}=\frac{1235}{72}\\ \diff{h}{t}&=2(61750)t\\ \diff{h}{t}\left(\frac{1}{60}\right)&=\frac{2(61750)}{60}=\frac{6175}{3}\\ \end{align*}Returning to the equation \(\ds\diff{\theta}{t}=\left(\dfrac{2}{h^2+4}\right)\ds\diff{h}{t} \text{:}\)
\begin{align*} \diff{\theta}{t}\left(\frac{1}{60}\right)&=\left(\frac{2}{\left(\frac{1235}{72}\right)^2+4}\right)\left(\frac{6175}{3} \right)\approx 13.8\;\frac{\mathrm{rad}}{\mathrm{hour}}\approx 0.0038\;\frac{\mathrm{rad}}{\mathrm{sec}} \end{align*}3.2.2.13. (✳).
Solution.
Differentiating with respect to \(t\text{:}\)
\begin{align*} \theta'(t)\cos\left(\theta(t)\right) &=\frac{x'(t)z(t)-x(t)z'(t)}{z(t)^2}\\ \theta'(t)&=\frac{x'(t)z(t)-x(t)z'(t)}{z(t)^2\cos\left(\theta(t)\right)}\\ \end{align*}From our diagram, we see \(\cos\left(\theta(t)\right)=\dfrac{0.5}{z(t)}\text{,}\) so:
\begin{align*} &=2\frac{x'(t)z(t)-x(t)z'(t)}{z(t)}\\ \end{align*}Substituting in \(x'(t)=2\text{,}\) \(z(t)=1.3\text{,}\) \(x(t)=1.2\text{,}\) and \(z'(t)=\dfrac{2\times1.2}{1.3}\text{:}\)
\begin{align*} \theta'(t) &=2\frac{2\times 1.3-1.2\times\frac{2\times 1.2}{1.3}}{1.3} \approx .592 \mbox{ radians/min} \end{align*}3.2.2.14.
Solution.
Differentiating with respect to time \(t\text{,}\)
\begin{align*} 2D\diff{D}{t}&=100\sin\theta\cdot\diff{\theta}{t} \end{align*}3.2.2.15. (✳).
Solution.
Plugging in the given data,
\begin{align*} A'&=2\pi\big(3\cdot 2-1\cdot 7\big)=-2\pi \end{align*}3.2.2.16.
Solution.
3.2.2.17.
Solution.
Note, \(b\) will change with time as well as \(h\text{.}\) So, differentiating with respect to time, \(t\text{:}\)
\begin{align*} \diff{A}{t}&=\frac{1}{2}\left(\diff{b}{t}\cdot h+b\cdot\diff{h}{t}\right) \end{align*}Differentiating with respect to \(t\text{:}\)
\begin{align*} \diff{b}{t}&=\frac{-2h\diff{h}{t}}{2\sqrt{150^2-h^2}}+ \frac{-2h\diff{h}{t}}{2\sqrt{200^2-h^2}}\\ &=\frac{-h\diff{h}{t}}{\sqrt{150^2-h^2}}+ \frac{-h\diff{h}{t}}{\sqrt{200^2-h^2}}\\ \end{align*}Using \(\ds\diff{h}{t}=-3\) centimetres per minute:
\begin{align*} \diff{b}{t}&=\frac{3h}{\sqrt{150^2-h^2}}+ \frac{3h}{\sqrt{200^2-h^2}}\\ \end{align*}When \(h=120\text{,}\) \(\sqrt{150^2-h^2}=90\) and \(\sqrt{200^2-h^2}=160\text{.}\) So, at this moment in time:
\begin{align*} b&=90+160=250\\ \diff{b}{t}&=\frac{3(120)}{90}+\frac{3(120)}{160} = 4+\frac{9}{4}=\frac{25}{4}\\ \end{align*}We return to our equation relating the derivatives of \(A\text{,}\) \(b\text{,}\) and \(h\text{.}\)
\begin{align*} \diff{A}{t}&=\frac{1}{2}\left(\diff{b}{t}\cdot h + b \cdot \diff{h}{t}\right)\\ \end{align*}When \(h=120\) cm, \(b=250\text{,}\) \(\ds\diff{h}{t}=-3\text{,}\) and \(\ds\diff{b}{t}=\dfrac{25}{4}\text{:}\)
\begin{align*} \diff{A}{t}&=\frac{1}{2}\left(\frac{25}{4}(120)+250(-3)\right)\\ &=0 \end{align*}3.2.2.18.
Solution.
This is the formula we wanted, relating \(A\) and \(\theta\text{.}\) Differentiating with respect to \(t\text{,}\)
\begin{align*} \diff{A}{t}&=\frac{35}{2}\diff{\theta}{t}=\frac{35}{2}\left(-\frac{\pi}{6}\right)=-\frac{35\pi}{12}\\ \end{align*}Since \(\ds\diff{S}{t}=\frac{1}{5}\ds\diff{A}{t}\text{,}\)
\begin{align*} \diff{S}{t}&=-\frac{1}{5}\frac{35\pi}{12}=-\frac{7\pi}{12}\approx -1.8\;\frac{\mathrm{cm}^3}{\mathrm{sec}^2} \end{align*}3.2.2.19.
Solution.
We want to differentiate with respect to \(t\text{.}\) Using the chain rule:
\begin{align*} \diff{A}{t}&=\diff{A}{h}\cdot\diff{h}{t}\\ \diff{A}{t}&=\left((1-h)\frac{2-2h}{2\sqrt{2h-h^2}}+(-1)\sqrt{2h-h^2} +\frac{-1}{\sqrt{1-(1-h)^2}}\right)\diff{h}{t}\\ &=\left(\frac{(1-h)^2}{\sqrt{2h-h^2}}-\sqrt{2h-h^2}-\frac{1}{\sqrt{2h-h^2}}\right)\diff{h}{t}\\ &=\left(\frac{(1-h)^2-1}{\sqrt{2h-h^2}}-\sqrt{2h-h^2}\right)\diff{h}{t}\\ &=\left(\frac{-(2h-h^2)}{\sqrt{2h-h^2}}-\sqrt{2h-h^2}\right)\diff{h}{t}\\ &=\left(-\sqrt{2h-h^2}-\sqrt{2h-h^2}\right)\diff{h}{t}\\ &=-2\sqrt{2h-h^2}\diff{h}{t}\\ \end{align*}We note here that the negative sign makes sense: as the door lowers, \(h\) increases and \(A\) decreases, so \(\ds\diff{h}{t}\) and \(\ds\diff{A}{t}\) should have opposite signs.
\begin{align*} \\ \end{align*}When \(h=\dfrac{1}{4}\) metres, and \(\ds\diff{h}{t}=\frac{1}{100}\) metres per second:
\begin{align*} \diff{A}{t}&=-2\sqrt{\frac{2}{4}-\frac{1}{4^2}}\left(\frac{1}{100}\right)=-\frac{\sqrt{7}}{200}\;\frac{\mathrm{cm}^2}{\mathrm{s}}\\ \end{align*}Since \(\ds\diff{F}{t}=\frac{1}{5}\ds\diff{A}{t}\text{:}\)
\begin{align*} \diff{F}{t}&=-\frac{\sqrt{7}}{1000}\;\frac{\mathrm{m}^3}{\mathrm{sec^2}} \end{align*}3.2.2.20.
Solution.
Differentiating with respect to \(t\text{:}\)
\begin{align*} \diff{V}{t}&=\frac{\pi}{3}h^2\diff{h}{t}\\ \end{align*}When \(h=7\) cm and \(\ds\diff{V}{t}=-5\) mL per minute,
\begin{align*} -5&=\frac{\pi}{3}(49)\diff{h}{t}\\ \diff{h}{t}&=\frac{-15}{49\pi}\approx -0.097 \mbox{ cm per minute} \end{align*}3.2.2.21.
Solution.
Using the chain rule, we differentiate both sides with respect to time, \(t\text{.}\)
\begin{align*} \diff{D}{t}&=2\cos\theta\cdot\diff{\theta}{t}\\ \end{align*}So, if \(\ds\diff{\theta}{t}=0.25\) radians per hour and \(\theta = \dfrac{\pi}{4}\) radians, then
\begin{align*} (a)\qquad\diff{D}{t}&=2\cos\left(\frac{\pi}{4}\right)\cdot 0.25=2\left(\frac{1}{\sqrt{2}}\right)\frac{1}{4}=\frac{1}{2\sqrt{2}}\;\mbox{metres per hour}. \end{align*}3.2.2.22.
Solution.
So, since \(\tan(-\theta)=-\tan(\theta)\text{:}\)
\begin{align*} \tan\theta&=\frac{x}{2} \end{align*}When the point is at \((0,-2)\text{,}\) since the observer is turning at one radian per second, also \(\ds\diff{\theta}{t}=1\text{.}\) Also, looking at the diagram, \(\theta=0\text{.}\) Plugging in these values:
\begin{align*} \sec^2\left(0\right)\cdot(1)&=\frac{1}{2}\cdot\diff{x}{t}\\ 1&=\frac{1}{2}\cdot\diff{x}{t}\\ \diff{x}{t}&=2 \end{align*}Differentiating with respect to \(t\text{:}\)
\begin{align*} 2D\cdot\diff{D}{t}&=2(x-2)\cdot\diff{x}{t}+2(y-1)\cdot\diff{y}{t}\\ \end{align*}We plug in \(x=0\text{,}\) \(y=2\text{,}\) \(\diff{x}{t}=1\text{,}\) \(\diff{y}{t}=-\frac{1}{2}\text{,}\) and \(D=-\sqrt{(0-2)^2+(2-1)^2}=-\sqrt{5}\) (negative because the point is to the left of \((2,1)\)):
\begin{align*} -2\sqrt{5}\cdot\diff{D}{t}&=2(-2)(1)+2(1)\left(-\frac{1}{2}\right)\\ \diff{D}{t}&=\frac{\sqrt{5}}{2}\;\mbox{units per second} \end{align*}3.2.2.23.
Solution.
where we use the fact that \(b\) is a positive number, so \(\sqrt{b^2}=|b|=b\text{.}\)
\begin{align*} \\ \end{align*}Since \(a=2b\text{,}\)
\begin{align*} \color{red}a&\color{red}{=\frac{20}{9-\sqrt{35}}}\\ \end{align*}Now we know \(a\) and \(b\) at the moment when \(a=2b\text{.}\) We still need to know \(\ds\diff{a}{t}\) and \(\ds\diff{b}{t}\) at that moment. Since \(a=5+t\text{,}\) always \(\textcolor{red}{\ds\diff{a}{t}=1}\text{.}\) The equation from (a) relates \(a\) and \(b\text{,}\) so differentiating both sides with respect to \(t\) will give us an equation relating \(\ds\diff{a}{t}\) and \(\ds\diff{b}{t}\text{.}\) When differentiating the portion with a square root, be careful not to forget the chain rule.
\begin{align*} 0&=3\left(\diff{a}{t}+\diff{b}{t}\right)-\frac{\left(\diff{a}{t}+3\diff{b}{t}\right)(3a+b)+(a+3b)\left(3\diff{a}{t}+\diff{b}{t}\right)}{2\sqrt{(a+3b)(3a+b)}}\\ \end{align*}Since \(\ds\diff{a}{t}=1\text{:}\)
\begin{align*} 0&=3\left(1+\diff{b}{t}\right)-\frac{\left(1+3\diff{b}{t}\right)(3a+b)+(a+3b)\left(3+\diff{b}{t}\right)}{2\sqrt{(a+3b)(3a+b)}} \end{align*}3.2.2.24.
Solution.
This will probably be useful information. Since we’re also given the value of a derivative, let’s differentiate the equation relating the variables implicitly with respect to \(t\text{.}\) For ease of notation, we will write \(\ds\diff{A}{t}=A'\text{,}\) etc.
\begin{align*} A'B+AB'&=\frac{2CC'+2DD'}{C^2+D^2+1}\\ \end{align*}At \(t=10\text{,}\) \(A=C=D=0\text{:}\)
\begin{align*} A'B+0&=\frac{0+0}{0+0+1}\\ A'B&=0\\ \end{align*}at \(t=10\text{,}\) \(A'=2\) units per second:
\begin{align*} 2B&=0\\ B&=0. \end{align*}
3.3 Exponential Growth and Decay — a First Look at Differential Equations
3.3.4 Exercises
Exercises for § 3.3.1
3.3.4.1.
Solution.
3.3.4.2.
Solution.
- \(\ds\diff{Q}{t}=0\text{,}\) so \(Q(t)=0=5\cdot 0 = 5\ds\diff{Q}{t}\text{,}\) so (a) is a solution.
- \(\ds\diff{Q}{t}=5e^t=Q(t)\text{,}\) so \(Q(t)=\ds\diff{Q}{t} \neq 5 \ds\diff{Q}{t}\text{,}\) so (b) is not a solution.
- \(\ds\diff{Q}{t}=5e^{5t}=5Q(t)\text{,}\) so \(Q(t)=\dfrac{1}{5}\ds\diff{Q}{t}\neq5\ds\diff{Q}{t}\text{,}\) so (c) is not a solution.
- \(\ds\diff{Q}{t}=\frac{1}{5}e^{t/5}=\frac{1}{5}Q(t)\text{,}\) so \(Q(t)=5\ds\diff{Q}{t}\text{,}\) so (d) is a solution.
- \(\ds\diff{Q}{t}=\frac{1}{5}e^{t/5}=\frac{1}{5}\left(Q(t)-1\right)\text{,}\) so \(Q(t)=5\ds\diff{Q}{t}+1\text{,}\) so (e) is not a solution.
3.3.4.3.
Solution.
That is,
\begin{align*} Ce^{-kt}&=0 \end{align*}3.3.4.4. (✳).
Solution.
3.3.4.5. (✳).
Solution.
3.3.4.6.
Solution.
So:
\begin{align*} Q(t)&=5\left(e^{-k}\right)^t\\ &=5\cdot2^{-\tfrac{t}{5730}}\\ \end{align*}Now, we can evaluate:
\begin{align*} Q(10000)&=5\cdot 2^{-\tfrac{10000}{5730}}\approx 1.5\;\mu g \end{align*}3.3.4.7.
Solution.
for some positive constant \(k\text{.}\) When \(t=100\text{,}\) the amount of Radium-226 left is 0.9576 grams, so
\begin{align*} 0.9576=Q(100)&=e^{-k\cdot 100}=\left(e^{-k}\right)^{100}\\ e^{-k}&=0.9576^{\tfrac{1}{100}}\\ \end{align*}This tells us
\begin{align*} Q(t)&=0.9576^{\tfrac{t}{100}}\\ \end{align*}So, if half the original amount of Radium-226 is left,
\begin{align*} \frac{1}{2}&=0.9576^{\tfrac{t}{100}}\\ \log\left(\frac{1}{2}\right)&=\log\left(0.9576^{\tfrac{t}{100}}\right)\\ -\log 2&=\frac{t}{100}\log(0.9576)\\ t&=-100\frac{\log 2}{\log 0.9576}\approx 1600 \end{align*}3.3.4.8. (✳).
Solution.
So, all together,
\begin{align*} Q(t)&=6\left(e^{-k}\right)^t=6\cdot \left(6^{-1}\right)^t=6^{1-t} \end{align*}3.3.4.9.
Solution.
where \(C\) is the amount in the sample at time \(t=0\text{,}\) and \(k\) is some positive constant. We know the half-life of the isotope, so we can find \(e^{-k}\text{:}\)
\begin{align*} \frac{C}{2}=Q(30)&=Ce^{-k\cdot 30}\\ \frac{1}{2}&=\left(e^{-k}\right)^{30}\\ 2^{-\tfrac{1}{30}}&=e^{-k}\\ \end{align*}So,
\begin{align*} Q(t)&=C\left(e^{-k}\right)^t=C\cdot2^{-\tfrac{t}{30}}\\ \end{align*}When only 0.01% of the original sample is left, \(Q(t)=0.0001C\text{:}\)
\begin{align*} 0.0001C=Q(t)&=C\cdot 2^{-\tfrac{t}{30}}\\ 0.0001&=2^{-\tfrac{t}{30}}\\ \log(0.0001)&=\log\left(2^{-\tfrac{t}{30}}\right)\\ \log\left(10^{-4}\right)&=-\frac{t}{30}\log2\\ -4\log 10&=-\frac{t}{30}\log2\\ t&=120\cdot\frac{\log10}{\log2}\approx 398.6 \end{align*}3.3.4.10.
Solution.
Now, we have a better formula for \(Q(t)\text{:}\)
\begin{align*} Q(t)&=C\left(e^{-k}\right)^t\\ Q(t)&=C\cdot2^{-\tfrac{t}{138}}\\ \end{align*}Finally, we can evaluate what percentage of the sample decays in a day.
\begin{align*} 100\frac{Q(t)-Q(t+1)}{Q(t)}&=100\frac{C\cdot2^{-\tfrac{t}{138}}-C\cdot2^{-\tfrac{t+1}{138}}}{C\cdot2^{-\tfrac{t}{138}}}\left(\frac{\frac{1}{C}}{\frac{1}{C}}\right)\\ &=100\frac{2^{-\tfrac{t}{138}}-2^{-\tfrac{t+1}{138}}}{2^{-\tfrac{t}{138}}}\\ &=100\left(2^{-\tfrac{t}{138}}-2^{-\tfrac{t+1}{138}}\right)2^{\tfrac{t}{138}}\\ &=100\left(1-2^{-\tfrac{1}{138}}\right)\approx 0.5 \end{align*}3.3.4.11.
Solution.
where \(6.9 \leq Q(0) \leq 7.5\text{.}\) We don’t exactly know \(Q(0)\text{,}\) and we don’t exactly know the half-life, so we also won’t exactly know \(Q(10)\text{:}\) we can only say that is it between two numbers. Our strategy is to find the highest and lowest possible values of \(Q(10)\text{,}\) given the information in the problem. In order for the most possible Uranium-232 to be in the sample after 10 years, we should start with the most and have the longest half-life (since this represents the slowest decay). So, we take \(Q(0)=7.5\) and \(Q(70)=\frac{1}{2}(7.5)\text{.}\)
\begin{align*} Q(t)&=7.5e^{-kt}\\ \frac{1}{2}(7.5)=Q(70)&=7.5e^{-k(70)}\\ \frac{1}{2}&=\left(e^{-k}\right)^{70}\\ 2^{-\tfrac{1}{70}}&=e^{-k}\\ \end{align*}So, in this secenario,
\begin{align*} Q(t)&=7.5\cdot 2^{-\tfrac{t}{70}}\\ \end{align*}After ten years,
\begin{align*} Q(10)&=7.5\cdot 2^{-\tfrac{10}{70}}\approx 6.79\\ \end{align*}So after ten years, the sample contains at most 6.8 \(\mu\)g. Now, let’s think about the least possible amount of Uranium-232 that could be left after 10 years. We should start with as little as possible, so take \(Q(0)=6.9\text{,}\) and the sample should decay quickly, so take the half-life to be 68.8 years.
\begin{align*} Q(t)&=6.9e^{-kt}\\ \frac{1}{2}6.9=Q(68.8)&=6.9e^{-k(68.8)}\\ \frac{1}{2}&=\left(e^{-k}\right)^{68.8}\\ 2^{-\tfrac{1}{68.8}}&=e^{-k}\\ \end{align*}In this scenario,
\begin{align*} Q(t)&=6.9\cdot 2^{-\tfrac{t}{68.8}}\\ \end{align*}After ten years,
\begin{align*} Q(10)&=6.9\cdot 2^{-\tfrac{10}{68.8}}\approx 6.24 \end{align*}Exercises for § 3.3.2
3.3.4.1.
Solution.
- \(\ds\diff{T}{t}=0=5\cdot0=5[T(t)-20]\text{,}\) so (a) gives a solution to the differential equation.
- \(\ds\diff{T}{t}=5[20e^{5t}]=5[T+20] \neq 5[T-20]\text{,}\) so (b) does not give a solution to the differential equation.
- \(\ds\diff{T}{t}=5[e^{5t}]=5[T-20]\text{,}\) so (c) gives a solution to the differential equation.
- \(\ds\diff{T}{t}=5[20e^{5t}]=5[T-20]\text{,}\) so (d) gives a solution to the differential equation.
3.3.4.2.
Solution.
3.3.4.3.
Solution.
Since the object is warmer than the room, \(T(0)-A\) is a nonzero constant. So,
\begin{align*} \lim_{t \to \infty}e^{Kt}&=0 \end{align*}3.3.4.4.
Solution.
That is, when
\begin{align*} [T(0)-A]e^{kt}&=0\\ \end{align*}Since \(e^{kt} \gt 0\) for all values of \(k\) and \(t\text{,}\) this happens exactly when
\begin{align*} T(0)-A=0 \end{align*}3.3.4.5.
Solution.
The information given tells us that \(T(10)=90\text{,}\) so
\begin{align*} 90&=-75e^{10K}+100\\ 75\left(e^K\right)^{10}&=10\\ \left(e^K\right)^{10}&=\frac{2}{15}\\ e^K&=\left(\frac{2}{15}\right)^{\tfrac{1}{10}}\\ \end{align*}This lets us describe \(T(t)\) without any unknown constants.
\begin{align*} T(t)&=-75\left(e^K\right)^{t}+100\\ &=-75\left(\frac{2}{15}\right)^{\tfrac{t}{10}}+100\\ \end{align*}The question asks what value of \(t\) gives \(T(t)=99.9\text{.}\)
\begin{align*} 99.9&=-75\left(\frac{2}{15}\right)^{\tfrac{t}{10}}+100\\ 75\left(\frac{2}{15}\right)^{\tfrac{t}{10}}&=0.1\\ \left(\frac{2}{15}\right)^{\tfrac{t}{10}}&=\frac{1}{750}\\ \log\left(\left(\frac{2}{15}\right)^{\tfrac{t}{10}}\right)&=\log\left(\frac{1}{750}\right)\\ \frac{t}{10}\log\left(\frac{2}{15}\right)&=-\log(750)\\ t&=\frac{-10\log(750)}{\log\left(\tfrac{2}{15}\right)}\approx 32.9 \end{align*}3.3.4.6.
Solution.
for some constant \(K\text{.}\) We are given that \(T(10)=100\text{.}\)
\begin{align*} 100=T(10)&=500e^{10K}\\ e^{10K}&=\frac{1}{5}\\ e^K&=5^{-\tfrac{1}{10}}\\ \end{align*}This gives us the more complete picture for the temperature of the stone.
\begin{align*} T(t)&=500\left(e^K\right)^t=500\cdot 5^{-\tfrac{t}{10}}\\ \end{align*}If \(T(t)=50:\)
\begin{align*} 50=T(t)&=500\cdot 5^{-\tfrac{t}{10}}\\ \frac{1}{10}=10^{-1}&=5^{-\tfrac{t}{10}}\\ 10&=5^{\tfrac{t}{10}}\\ \log(10)&=\frac{t}{10}\log(5)\\ t&=10\frac{\log(10)}{\log(5)}\approx 14.3 \end{align*}3.3.4.7. (✳).
Solution.
- First scenario: At time \(0\text{,}\) Newton mixes 9 parts coffee at temperature \(95^\circ\) C with 1 part cream at temperature \(5^\circ\) C. The resulting mixture has temperature\begin{equation*} \frac{9\times 95+1\times 5}{9+1}=86^\circ \end{equation*}The mixture cools according to Newton’s Law of Cooling, with initial temperature 86\(^\circ\) and ambient temperature 22\(^\circ\text{:}\)\begin{align*} T(t)&=[86-22]e^{-kt}+22\\ T(t)&=64e^{-kt}+22\\ \end{align*}We could compute \(k\) from this, but we don’t need it.
After 10 minutes,
\begin{align*} \textcolor{red}{54}=T(10)&=22+64e^{-10 k}\\ e^{-10 k}&=\frac{54-22}{64}=\frac{1}{2} \end{align*} - Second scenario: At time \(0\text{,}\) Newton gets hot coffee at temperature \(95^\circ\) C. It cools according to Newton’s Law of Cooling\begin{gather*} T(t)=[T(0)-22]e^{-kt}+22 \end{gather*}In this second scenario, \(T(0)=95\text{,}\) so\begin{equation*} T(t)=[95-22]e^{-kt}+22=73e^{-kt}+22 \end{equation*}The value of \(k\) is the same as in the first scenario, so after 10 minutes\begin{equation*} T(10)=22+73e^{-10k}=22+73\frac{1}{2}=58.5 \end{equation*}This cooled coffee is mixed with cold cream to yield a mixture of temperature\begin{equation*} \frac{9\times 58.5+1\times 5}{9+1}=\color{red}{53.15} \end{equation*}
3.3.4.8. (✳).
Solution.
-
By Newton’s law of cooling, the rate of change of temperature is proportional to the difference between \(T(t)\) and the ambient temperature, which in this case is \(30^\circ\text{.}\) Thus\begin{equation*} \diff{T}{t}=k[T(t)-30] \end{equation*}for some constant of proportionality \(k\text{.}\) To answer part (a), all we have to do is find \(k\text{.}\)Under Newton’s Law of Cooling, the temperature at time \(t\) will be given by\begin{align*} T(t)&=[T(0)-A]e^{kt}+A\\ &=[5-30]e^{kt}+30\\ &=-25e^{kt}+30\\ \end{align*}
We are told \(T(5)=10\text{:}\)
\begin{align*} 10&=-25e^{5k}+30\\ 25e^{5k}&=20\\ e^{5k}&=\frac{4}{5}\\ 5k&=\log(4/5)\\ k&=\tfrac{1}{5}\log(4/5)\\ \end{align*}So, the differential equation is
\begin{align*} \diff{T}{t}(t)&=\frac{1}{5}\log(4/5)[T(t)-30] \end{align*} - Since \(T(t)=30-25e^{kt}\text{,}\) the temperature of the tea is \(20^\circ\) when\begin{align*} 30-25 e^{kt}&=20\\ e^{kt}&=\frac{10}{25}\\ kt&=\log\left(\frac{10}{25}\right)\\ t&=\frac{1}{k}\log\frac{2}{5}\\ &=\frac{5\log(2/5)}{\log(4/5)}\\ &\approx{20.53\;\mathrm{ min}} \end{align*}
3.3.4.9.
Solution.
- If \(k \lt 0\text{,}\) then \(\ds\lim_{t \to \infty}0.8^{kt} = \infty\text{,}\) since \(0.8 \lt 1\text{.}\) So, \(k \geq 0\text{.}\)
- If \(k=0\text{,}\) then \(T(t)=16\) for all values of \(t\text{.}\) But, in the statement of the question, the object is changing temperature. So, \(k \gt 0\text{.}\)
Exercises for § 3.3.3
3.3.4.1.
Solution.
3.3.4.2.
Solution.
This gives us a better idea of \(P(t)\text{:}\)
\begin{align*} P(t)&=121e^{bt}=121\left(\frac{136}{121}\right)^t\\ \end{align*}2020 is 5 years after 2015, so in 2020 (assuming the population keeps growing according to the Malthusian model) the population will be
\begin{align*} P(5)&=121\left(\frac{136}{121}\right)^5\approx217 \end{align*}3.3.4.3.
Solution.
So, the population at time \(t\) is
\begin{align*} P(t)&=1000\cdot 2^t\\ \end{align*}We want to know at what time the population triples, to 3,000 individuals.
\begin{align*} 3000&=1000\cdot 2^t\\ 3&=2^t\\ \log(3)&=\log\left(2^t\right)=t\log(2)\\ t&=\frac{\log(3)}{\log(2)}\approx 1.6 \end{align*}3.3.4.4.
Solution.
for some constant \(b\text{.}\) We want to get rid of this extraneous variable \(b\text{,}\) so we use the given information. If 1928 is \(a\) years after the wreck:
\begin{align*} 1000=P(a)&=2e^{ba}\\ 1500=P(a+1)&=2e^{b(a+1)}=2e^{ba}e^b\\ \end{align*}So,
\begin{align*} (1000)\left(e^b\right)&=\left(2e^{ba}\right)\left(e^b\right)=1500\\ \end{align*}Which tells us
\begin{align*} e^b&=\frac{1500}{1000}=1.5\\ \end{align*}Now, our model is complete:
\begin{align*} P(t)&=2\left(e^b\right)^t=2\cdot1.5^t\\ \end{align*}Since \(P(a)=1000\text{,}\) we can find \(a\text{:}\)
\begin{align*} 1000=P(a)&=2\cdot1.5^a\\ 500&=1.5^a\\ \log(500)&=\log\left(1.5^a\right)=a\log(1.5)\\ a&=\frac{\log(500)}{\log(1.5)}\approx 15.3 \end{align*}3.3.4.5.
Solution.
for some constant \(b\text{.}\) The constant \(b\) is the net birthrate per population member per unit time. Assuming that the net birthrate for a larger population will be the same as the test population, we can use the data from the test to find \(e^b\text{.}\) Let \(Q(t)\) be the number of individuals in the test population at time \(t\text{.}\)
\begin{align*} Q(t)&=Q(0)e^{bt}=200e^{bt}\\ 1000&=Q(3)=200e^{3b}\\ 5&=e^{3b}\\ e^b&=5^{1/3}\\ \end{align*}Now that we have an idea of the birthrate, we predict
\begin{align*} P(t)&=P(0)\left(e^{b}\right)^t=P(0)\cdot5^{\tfrac{t}{3}}\\ \end{align*}We want \(P(12)=10^6+P(0)\text{.}\)
\begin{align*} 10^6+P(0)=P(12)&=P(0)\cdot 5^{\tfrac{12}{3}}=P(0)\cdot 5^4\\ 10^6&=P(0)\cdot 5^4-P(0)=P(0)\left[5^4-1\right]\\ P(0)&=\frac{10^6}{5^4-1}\approx 1603 \end{align*}3.3.4.6.
Solution.
- [(a)] Since \(f(t)\) gives the amount of the radioactive isotope in the sample at time \(t\text{,}\) the amount of the radioactive isotope in the sample when \(t=0\) is \(f(0)=100e^0=100\) units. Since the sample is decaying, \(f(t)\) is decreasing, so \(f'(t)\) is negative. Differentiating, \(f'(t)=k(100e^{kt})\text{.}\) Since \(100e^{kt}\) is positive and \(f'(t)\) is negative, \(k\) is negative.
- [(b)] Since \(f(t)\) gives the size of the population at time \(t\text{,}\) the number of individuals in the population when \(t=0\) is \(f(0)=100e^0=100\text{.}\) Since the population is growing, \(f(t)\) is increasing, so \(f'(t)\) is positive. Differentiating, \(f'(t)=k(100e^{kt})\text{.}\) Since \(100e^{kt}\) is positive and \(f'(t)\) is positive, \(k\) is also positive.
- [(c)] Newton’s Law of Cooling gives the temperature of an object at time \(t\) as \(f(t)=[f(0)-A]e^{kt}+A\text{,}\) where \(A\) is the ambient temperature surrounding the object. In our case, the ambient temperature is 0 degrees. In an object whose temperature is being modelled by Newton’s Law of Cooling, it doesn’t matter whether the object is heating or cooling, \(k\) is negative. We saw this in Question 3.3.4.3 of Section 3.3.2, but it bears repeating. Since \(f(t)\) approaches the ambient temperature (in this case, 0) as \(t\) goes to infinty:\begin{equation*} \lim_{t \to \infty}100e^{kt}=0 \end{equation*}so \(k\) is negative.
Further problems for § 3.3
3.3.4.1. (✳).
Solution.
for some constant \(C\text{.}\) The second piece of given information tells us \(f(0)=2\text{.}\) Using this, we can solve for \(C\text{:}\)
\begin{align*} 2&=f(0)=Ce^{0}=C\\ \end{align*}Now, we know \(f(x)\) entirely:
\begin{align*} f(x)&=2e^{\pi x}\\ \end{align*}So, we can evaluate \(f(2)\)
\begin{align*} f(2)&=2e^{2\pi} \end{align*}3.3.4.2.
Solution.
We can check that this is reasonable: if
\begin{align*} T(t)&=\left[T(0)+\frac{9}{7}\right]e^{7t}-\frac{9}{7}\\ \end{align*}then
\begin{align*} \diff{T}{t}&=7\left[T(0)+\frac{9}{7}\right]e^{7t}\\ &=7\left[T+\frac{9}{7}\right]\\ &=7T+9. \end{align*}3.3.4.3. (✳).
Solution.
So,
\begin{align*} Q(0)e^{8k}&=0.8\,Q(0)\\ e^{8k}&=0.8\\ e^k&=0.8^{\tfrac{1}{8}}\\ \end{align*}If \(Q(0)=100\text{,}\) the time \(t\) at which \(Q(t)=40\) is determined by
\begin{align*} 40=Q(t)&=Q(0)e^{kt}=100e^{kt}=100\left(0.8^{\tfrac{1}{8}}\right)^t=100\cdot 0.8^{\tfrac{t}{8}}\\ \end{align*}Solving for \(t\text{:}\)
\begin{align*} \frac{40}{100}&=0.8^{\tfrac{t}{8}}\\ \log\left(0.4\right)&=\log\left(0.8^{\tfrac{t}{8}}\right)=\frac{t}{8}\log(0.8)\\ t&=\frac{8\log(0.4)}{\log(0.8)}\approx 32.85\mbox{ days} \end{align*}3.3.4.4.
Solution.
where \(A\) is the room temperature, and \(K\) is some constant. We are told that \(T(15)=85\) and \(T(30)=73\text{,}\) so:
\begin{align*} 85=T(15)&=[100-A]e^{15K}+A\\ 73=T(30)&=[100-A]e^{30K}+A\\ \end{align*}Rearranging both equations:
\begin{align*} \frac{85-A}{100-A}&=e^{15K}\\ \frac{73-A}{100-A}&=e^{30K}=\left(e^{15K}\right)^2\\ \end{align*}Using these equations:
\begin{align*} \left(\frac{85-A}{100-A}\right)^2=\left(e^{15K}\right)^2&=e^{30K}=\frac{73-A}{100-A}\\ \frac{(85-A)^2}{100-A}&=73-A\\ (85-A)^2&=(73-A)(100-A)\\ 85^2-170A+A^2&=7300-173A+A^2\\ 173A-170A&=7300-85^2\\ 3A&=75\\ A&=25 \end{align*}3.3.4.5. (✳).
Solution.
- The amount of money at time \(t\) obeys\begin{align*} \diff{A}{t}&= 0.05 A+2,\!000=0.05[A-(-40,\!000)]\\ \end{align*}
Using Corollary 3.3.8
\begin{align*} A(t)&=[A(0)+40,\!000]e^{0.05t}-40,\!000\\ &=90,\!000\cdot e^{0.05t}-40,\!000\\ \end{align*}where \(t=0\) corresponds to the year when the graduate is 25.
\begin{align*} \\ \end{align*}When the graduate is 65 years old, \(t=40\text{,}\) so
\begin{align*} A(40)&=90,\!000\, e^{0.05 \times 40}-40,000\approx \$625,015.05 \end{align*} - When the graduate stops depositing money and instead starts withdrawing money at a rate \(W\text{,}\) the equation for \(A\) becomes\begin{align*} \diff{A}{t}&= 0.05 A-W= 0.05 [A(t)-20 W]\\ \end{align*}
Using Corollary 3.3.8 and assuming that the interest rate remains 5%
\begin{align*} A(t)&=[A(0)-20W]e^{0.05t}+20W\\ &=[625,015.05-20W]e^{0.05t}+20W\\ \end{align*}Note that, for part (b), we only care about what happens when the graduate starts withdrawing money. We take \(t=0\) to correspond to the year when the graduate is 65--so we’re using a different \(t\) from part (a). Then from part (a), \(A(0)=625,025.05\text{.}\)
\begin{align*} \\ \end{align*}We want the account to be depleted when the graduate is 85. So, we want
\begin{align*} 0&=A(20)\\ 0&=20W+ e^{0.05\times 20}(625,015.05-20W)\\ 0&=20W+ e(625,015.05-20W)\\ 20(e-1)W&= 625,015.05e\\ W&=\frac{625,015.05e}{20(e-1)}\approx\$49,437.96 \end{align*}
3.3.4.6. (✳).
Solution.
Using Corollary 3.3.8
\begin{align*} A(t)&=[A(0)-150,\!000]e^{0.06t}+150,\!000\\ &=[120,\!000-150,\!000]e^{0.06t}+150,\!000\\ &=-30,\!000e^{0.06t}+150,\!000 \end{align*}3.3.4.7. (✳).
Solution.
The doubling time \(t\) obeys:
\begin{align*} Q(t)&=2Q(0)\\ \mbox{So,}\qquad Q(0)e^{kt}&=2Q(0)\\ e^{kt}&=2\\ 3^{\tfrac{t}{9}}&=2\\ \frac{t}{9}\log 3 &=\log 2\\ t&=9\frac{\log 2}{\log 3}\approx5.68 \mbox{ hr} \end{align*}3.3.4.8. (✳).
Solution.
So, we can use the corollary, with \(K=-k\text{,}\) \(T=v\text{,}\) and \(A=-\dfrac{g}{k}\text{.}\)
\begin{align*} v(t)&=\left(v(0)-\left(-\frac{g}{k}\right)\right)e^{-kt}-\frac{g}{k}\\ &=\left(v_0+\frac{g}{k}\right)e^{-kt}-\frac{g}{k} \end{align*}Since \(k\) is positive:
\begin{align*} &=\left(v_0+\frac{g}{k}\right)\left(0\right)-\frac{g}{k}\\ &=-\dfrac{g}{k} \end{align*}
3.4 Approximating Functions Near a Specified Point — Taylor Polynomials
3.4.11 Exercises
Exercises for § 3.4.1
3.4.11.1.
Solution.
3.4.11.2.
Solution.
3.4.11.3.
Solution.
3.4.11.4.
Solution.
3.4.11.5.
Solution.
Exercises for § 3.4.2
3.4.11.1.
Solution.
3.4.11.2.
Solution.
3.4.11.3.
Solution.
3.4.11.4.
Solution.
So, we calculate:
\begin{align*} f(1)&=\log(1)=0\\ f'(x)&=\frac{1}{x}\\ f'(1)&=\frac{1}{1}=1\\ \end{align*}Therefore,
\begin{align*} f(x) &\approx 0+1(x-1)=x-1\\ \end{align*}When \(x=0.93\text{:}\)
\begin{align*} f(0.93) &\approx 0.93-1=-0.07 \end{align*}3.4.11.5.
Solution.
3.4.11.6.
Solution.
\(a\) needs to be a number whose fifth root we know. Since \(\sqrt[5]{32}=2\text{,}\) and 32 is reasonably close to 30, \(a=32\) is a great choice.
\begin{align*} f(32)&=\sqrt[5]{32}=2\\ f'(32)&=\frac{1}{5\cdot 2^4}=\frac{1}{80}\\ \end{align*}The linear approximation of \(f(x)\) about \(x=32\) is
\begin{align*} f(x) &\approx 2+\frac{1}{80}(x-32)\\ \end{align*}When \(x=30\text{:}\)
\begin{align*} f(30) &\approx 2+\frac{1}{80}(30-32)=2-\frac{1}{40}=\frac{79}{40} \end{align*}3.4.11.7.
Solution.
3.4.11.8.
Solution.
3.4.11.9.
Solution.
Exercises for § 3.4.3
3.4.11.1.
Solution.
3.4.11.2.
Solution.
We subsitute \(f(a)=2a+5\text{,}\) \(f'(a)=2\text{,}\) and \(f''(a)=0\text{:}\)
\begin{align*} f(x) &\approx (2a+5)+2(x-a)=2x+5 \end{align*}3.4.11.3.
Solution.
So,
\begin{align*} f(x) &\approx f(1)+f'(1)(x-1)+\frac{1}{2}f''(1)(x-1)^2\\ &=0+(x-1)-\frac{1}{2}(x-1)^2 \end{align*}3.4.11.4.
Solution.
Using the quadratic approximation \(f(x) \approx f(0)+f'(0)(x-0)+\frac{1}{2}f''(0)(x-0)^2\text{:}\)
\begin{align*} f(x)&\approx 1 -\frac{1}{2}x^2\\ f\left(\frac{1}{15}\right)&\approx 1-\frac{1}{2\cdot 15^2}=\frac{449}{450} \end{align*}3.4.11.5.
Solution.
We compute derivatives.
\begin{align*} f(0)&=e^0=1\\ f'(x)&=2e^{2x}\\ f'(0)&=2e^0=2\\ f''(x)&=4e^{2x}\\ f''(0)&=4e^0=4\\ \end{align*}Substituting:
\begin{align*} f(x) &\approx 1+2(x-0)+\frac{4}{2}(x-0)^2\\ f(x) &\approx 1+2x+2x^2 \end{align*}3.4.11.6.
Solution.
- \(f(x)=x^{4/3}\text{.}\) In this case, we would probably choose to approximate \(f(x)\) about \(x=8\) (since 8 is a cube, \(8^{4/3}=2^4=16\) is something we can evaluate) or \(x=1\text{.}\)
- \(f(x)=5^{x}\text{.}\) We can easily figure out \(f(x)\) when \(x\) is a whole number, so we would want to centre our approximation around some whole number \(x=a\text{,}\) but then \(f'(a)=5^a\log(5)\) gives us a problem: without a calculator, it’s hard to know what \(\log(5)\) is.
- Since \(5^{4/3}=5\sqrt[3]{5}\text{,}\) we can use \(f(x)=5\sqrt[3]{x}\text{.}\) As in the first bullet, we would centre about \(x=8\text{,}\) or \(x=1\text{.}\)
Using the quadratic approximation \(f(x) \approx f(a)+f'(a)(x-a)+\frac{1}{2}f''(a)(x-a)^2\text{:}\)
\begin{alignat*}{3} f(x)&\approx 10+\frac{5}{12}(x-8)-\frac{5}{288}(x-8)^2\\ f(5)&\approx 10+\frac{5}{12}(-3)-\frac{5}{288}(9)=\frac{275}{32} \end{alignat*}3.4.11.7.
Solution.
-
[3.4.11.7.a] For every value of \(n\text{,}\) the term being added is simply the constant \(1\text{.}\) So, \(\ds\sum_{n=5}^{30} 1 = 1+1+\cdots+1\text{.}\) The trick is figuring out how many 1s are added. Our index \(n\) takes on all integers from 5 to 30, including 5 and 30, which is 26 numbers. So, \(\ds\sum_{n=5}^{30}=26\text{.}\)If you’re having a hard time seeing why the sum is 26, instead of 25, think of it this way: there are thirty numbers in the collection \(\{1,2,3,4,5,6,\ldots,29,30\}\text{.}\) If we remove the first four, we get \(30-4=26\) numbers in the collection \(\{5,6,\ldots,30\}\text{.}\)
- \begin{align*} \amp\sum_{n=1}^3\left[2(n+3)-n^2\right]\\ &\hskip0.5in= \underbrace{2(1+3)-1^2}_{n=1}+ \underbrace{2(2+3)-2^2}_{n=2}+ \underbrace{2(3+3)-3^2}_{n=3}\\ &\hskip0.5in=8-1+10-4+12-9=16 \end{align*}
- \begin{align*} \ds\sum_{n=1}^{10} \left[\frac{1}{n}-\frac{1}{n+1}\right]&= \underbrace{\frac{1}{1}-\frac{1}{1+1}}_{n=1}+ \underbrace{\frac{1}{2}-\frac{1}{2+1}}_{n=2}+ \underbrace{\frac{1}{3}-\frac{1}{3+1}}_{n=3}\\ \amp+ \underbrace{\frac{1}{4}-\frac{1}{4+1}}_{n=4}+ \underbrace{\frac{1}{5}-\frac{1}{5+1}}_{n=5}+ \underbrace{\frac{1}{6}-\frac{1}{6+1}}_{n=6}\\ &+ \underbrace{\frac{1}{7}-\frac{1}{7+1}}_{n=7}+ \underbrace{\frac{1}{8}-\frac{1}{8+1}}_{n=8}+ \underbrace{\frac{1}{9}-\frac{1}{9+1}}_{n=9}\\ &+ \underbrace{\frac{1}{10}-\frac{1}{10+1}}_{n=10}\\ \end{align*}
Most of these cancel!
\begin{align*} &=\frac{1}{1}\underbrace{-\frac{1}{2}+\frac{1}{2}}_0 \underbrace{-\frac{1}{3}+\frac{1}{3}}_0 \underbrace{-\frac{1}{4}+\frac{1}{4}}_0 \underbrace{-\frac{1}{5}+\frac{1}{5}}_0 \underbrace{-\frac{1}{6}+\frac{1}{6}}_0\\ &\underbrace{-\frac{1}{7}+\frac{1}{7}}_0 \underbrace{-\frac{1}{8}+\frac{1}{8}}_0 \underbrace{-\frac{1}{9}+\frac{1}{9}}_0 \underbrace{-\frac{1}{10}+\frac{1}{10}}_0 -\frac{1}{11}\\ &=1-\frac{1}{11}=\frac{10}{11} \end{align*} - \begin{align*} \ds\sum_{n=1}^{4}\frac{5\cdot 2^n}{4^{n+1}} &= 5\sum_{n=1}^{4}\frac{2^n}{4\cdot 4^n}=\frac{5}{4}\sum_{n=1}^4\frac{2^n}{4^n} =\frac{5}{4}\sum_{n=1}^4\frac{1}{2^n}\\ &=\frac{5}{4}\left( \underbrace{\frac{1}{2}}_{n=1}+ \underbrace{\frac{1}{4}}_{n=2}+ \underbrace{\frac{1}{8}}_{n=3}+ \underbrace{\frac{1}{16}}_{n=4} \right)=\frac{75}{64} \end{align*}
3.4.11.8.
Solution.
- \(\displaystyle 1+2+3+4+5 = \ds\sum_{n=1}^5n\)
- \(\displaystyle 2+4+6+8=\ds\sum_{n=1}^4 2n\)
- \(\displaystyle 3+5+7+9+11=\ds\sum_{n=1}^{5}(2n+1)\)
- \(\displaystyle 9+16+25+36+49=\ds\sum_{n=3}^7 n^2\)
- \(\displaystyle 9+4+16+5+25+6+36+7+49+8=\ds\sum_{n=3}^7 (n^2+n+1)\)
- \(\displaystyle 8+15+24+35+48= \ds\sum_{n=3}^7 (n^2-1)\)
-
\(3-6+9-12+15-18=\ds\sum_{n=1}^6 (-1)^{n+1}3n\)Remark: if we had written \((-1)^n\) instead of \((-1)^{n+1}\text{,}\) with everything else the same, the signs would have been reversed.
3.4.11.9.
Solution.
Now, we can find the quadratic approximation about \(x=0\text{.}\)
\begin{align*} f(x)&\approx f(0)+f'(0)x+\frac{1}{2}f''(0)x^2\\ &=2x\\ f(1)& \approx 2 \end{align*}3.4.11.10.
Solution.
3.4.11.11.
Solution.
- \begin{align*} \knowl{./knowl/xref/s3_4_3equiv2.html}{\text{3.4.11.11.d}}&=2\sum_{n=1}^{10} n = 2(1+2+\cdots+10)\\ \amp=2(1)+2(2)+\cdots +2(10)=\sum_{n=1}^{10} 2n \\ \amp= \knowl{./knowl/xref/s3_4_3equiv1.html}{\text{3.4.11.11.a}}\\ \end{align*}
So 3.4.11.11.a and 3.4.11.11.d are equivalent.
\begin{align*} \knowl{./knowl/xref/s3_4_3equiv3.html}{\text{3.4.11.11.e}}&=2\ds\sum_{n=2}^{11} (n-1)=2(1+2+\cdots + 10) = \knowl{./knowl/xref/s3_4_3equiv2.html}{\text{3.4.11.11.d}} \end{align*}So 3.4.11.11.e and 3.4.11.11.d are equivalent. - Second, we’ll show that 3.4.11.11.b and 3.4.11.11.g are equivalent.\begin{align*} \knowl{./knowl/xref/s3_4_3equiv6.html}{\text{3.4.11.11.g}}&=\dfrac{1}{4}\ds\sum_{n=1}^{10}\left( \frac{4^{n+1}}{2^n}\right) =\frac{1}{4}\sum_{n=1}^{10}\left(\frac{4\cdot4^n}{2^n}\right)\\ \amp=\frac{4}{4}\sum_{n=1}^{10}\left(\frac{4^n}{2^n}\right) =\sum_{n=1}^{10}\left(\frac{4}{2}\right)^n\\ \amp=\sum_{n=1}^{10} 2^n=\knowl{./knowl/xref/s3_4_3equiv5.html}{\text{3.4.11.11.b}}& \end{align*}
- Third, we’ll show that 3.4.11.11.c and 3.4.11.11.f are equivalent.\begin{align*} \knowl{./knowl/xref/s3_4_3equiv10.html}{\text{3.4.11.11.f}} \amp=\ds\sum_{n=5}^{14} (n-4)^2 =1^2+2^2+\cdots + 10^2 =\ds\sum_{n=1}^{10} n^2\\ \amp=\knowl{./knowl/xref/s3_4_3equiv7.html}{\text{3.4.11.11.c}} \end{align*}
-
Now, we have three groups, where each group consists of equivalent expressions. To be quite thorough, we should show that no two of these groups contain expressions that are secretly equivalent. They would be hard to evaluate, but we can bound them and show that no two expressions in two separate groups could possibly be equivalent. Notice that\begin{align*} \color{red}{\sum_{n=1}^{10} 2^n} &=2^1+2^2+\cdots + 2^{10} \gt 2^{10}=1024\\ \color{green}{\sum_{n=1}^{10} n^2} & \lt \sum_{n=1}^{10} 10^2 = 10(100)=1000\\ \color{green}{\sum_{n=1}^{10} n^2} &=1^2+2^2+\cdots 8^2+9^2+10^2 \gt 8^2+9^2+10^2=245\\ \color{blue}{\sum_{n=1}^{10} 2n} & \lt \sum_{n=1}^{10}20 = 200 \end{align*}The expressions in the blue group add to less than 200, but the expressions in the green group add to more than 245, and the expressions in the red group add to more than 1024, so the blue groups expressions can’t possibly simplify to the same number as the red and green group expressions.The expressions in the green group add to less than 1000. Since the expressions in the red group add to more than 1024, the expressions in the green and red groups can’t possibly simplify to the same numbers.
- [3.4.11.11.b=3.4.11.11.g], and
Exercises for § 3.4.4
3.4.11.1.
Solution.
- \(\displaystyle T_3(1)=f(1)\)
- \(\displaystyle T_3'(1)=f'(1)\)
- \(\displaystyle T_3''(1)=f''(1)\)
- \(\displaystyle T_3'''(1)=f'''(1)\)
3.4.11.2.
Solution.
So,
\begin{align*} \sum_{k=0}^{n} \frac{f^{(k)}(5)}{k!}(x-5)^k&=\sum_{k=0}^{n}\frac{2k+1}{3k-9}(x-5)^k\\ \end{align*}For any \(k\) from 0 to \(n\text{,}\)
\begin{align*} \frac{f^{(k)}(5)}{k!}&=\frac{2k+1}{3k-9}\\ \end{align*}In particular, when \(k=10\text{,}\)
\begin{align*} \frac{f^{(10)}(5)}{10!}&=\frac{20+1}{30-9}=1\\ f^{(10)}(5)&=10! \end{align*}3.4.11.3.
Solution.
while the third order Maclaurin polynomial for \(f(x)\) is
\begin{align*} T_3(x)&=f(0)+f'(0)x+\frac{1}{2}f''(0)x^2+\frac{1}{3!}f'''(0)x^3\\ \end{align*}So, we simply “chop off” the part of \(T_4(x)\) that includes \(x^4\text{:}\)
\begin{align*} T_3(x)&=-x^3+x^2-x+1 \end{align*}3.4.11.4.
Solution.
while the third order Taylor polynomial for \(f(x)\) about \(x=1\) is
\begin{align*} T_3(x)&=f(1)+f'(1)(x-1)+\frac{1}{2}f''(1)(x-1)^2+\frac{1}{3!}f'''(1)(x-1)^3 \end{align*}Recall that \(T_4(x)\) and \(f(x)\) have the same values at \(x=1\) (although maybe not anywhere else!), and they also have the same first, second, third, and fourth derivatives at \(x=1\) (but again, maybe not anywhere else, and maybe their fifth derivatives don’t agree). This tells us the following:
\begin{align*} T_4(x)&=x^4+x^3-9&&\Rightarrow& f(1)=T_4(1)&=-7\\ T_4'(x)&=4x^3+3x^2&&\Rightarrow& f'(1)=T_4'(1)&=7\\ T_4''(x)&=12x^2+6x&&\Rightarrow& f''(1)=T_4''(1)&=18\\ T_4'''(x)&=24x+6&&\Rightarrow& f'''(1)=T_4'''(1)&=30 \end{align*}3.4.11.5.
Solution.
3.4.11.6.
Solution.
- \(\displaystyle f(a)=T_3(a)\)
- \(\displaystyle f'(a)=T'_3(a)\)
- \(\displaystyle f''(a)=T''_3(a)\)
- \(\displaystyle f'''(a)=T'''_3(a)\)
Now, let’s move to the Taylor polynomial. Remember that \(e\) is a constant.
\begin{align*} T_3(x)&=-\frac{2}{3}\sqrt{e^3}+3ex-6\sqrt{e}x^2+x^3\\ T_3'(x)&=3e-12\sqrt{e}x+3x^2\\ T_3''(x)&=-12\sqrt{e}+6x\\ T_3'''(x)&=6\\ \color{red}{T_3'''(a)}&\color{red}{=6}\\ \end{align*}The final bullet point gives us the equation:
\begin{align*} \color{blue}{f'''(a)}&=\color{red}{T_3'''(a)}\\ \color{blue}{12\log a} &= \color{red}{6}\\ \log a &= \frac{1}{2}\\ a&=e^{\tfrac{1}{2}} \end{align*}Exercises for § 3.4.5
3.4.11.1.
Solution.
3.4.11.2.
Solution.
3.4.11.3.
Solution.
So, in general,
\begin{align*} f^{(k)}(x)&=2^x\left(\log 2\right)^k\\ \end{align*}We notice that this formula works even when \(k=0\) and \(k=1\text{.}\) When \(x=1\text{,}\)
\begin{align*} f^{(k)}(1)&=2\left(\log 2\right)^k\\ \end{align*}The \(n\)th order Taylor polynomial of \(f(x)\) about \(x=1\) is
\begin{align*} T_{n}(x)&=\sum_{k=0}^n \frac{f^{(k)}(1)}{k!}(x-1)^k\\ &=\sum_{k=0}^n \frac{2(\log 2)^k}{k!}(x-1)^k \end{align*}3.4.11.4.
Solution.
3.4.11.5.
Solution.
Using the chain rule,
\begin{align*} f'(x)&=(-1)(1-x)^{-2}(-1)=(1-x)^{-2}\\ f''(x)&=(-2)(1-x)^{-3}(-1)=2(1-x)^{-3}\\ f^{(3)}(x)&=(-3)(2)(1-x)^{-4}(-1)=2(3)(1-x)^{-4}\\ f^{(4)}(x)&=(-4)(2)(3)(1-x)^{-5}(-1)=2(3)(4)(1-x)^{-5}\\ f^{(5)}(x)&=(-5)(2)(3)(4)(1-x)^{-6}(-1)=2(3)(4)(5)(1-x)^{-6}\\ \end{align*}Recognizing the pattern,
\begin{align*} f^{(k)}(x)&=k!(1-x)^{-(k+1)}\\ f^{(k)}(0)&=k!(1)^{-(k+1)}=k!\\ \end{align*}The \(n\)th order Maclaurin polynomial for \(f(x)\) is
\begin{align*} T_n(x)&=\sum_{k=0}^n \frac{f^{(k)}(0)}{k!}x^k\\ &=\sum_{k=0}^n \frac{k!}{k!}x^k\\ &=\sum_{k=0}^n x^k \end{align*}3.4.11.6.
Solution.
3.4.11.7.
Solution.
Now, if we want to approximate \(f\left(\dfrac{1}{\sqrt{3}}\right)=6\arctan\left(\dfrac{1}{\sqrt{3}}\right)=\pi\text{:}\)
\begin{align*} \pi&=f\left(\dfrac{1}{\sqrt{3}}\right) \approx T_5\left(\dfrac{1}{\sqrt{3}}\right)= \frac{6}{\sqrt{3}}-\frac{2}{\sqrt{3}^3}+\frac{6}{5\sqrt{3}^5}\\ &=2\sqrt{3}\left(1-\frac{1}{3\cdot3}+\frac{1}{5\cdot 9}\right)\approx 3.156 \end{align*}3.4.11.8.
Solution.
When \(k \geq 2\text{,}\) making use of the fact that \(0!=1\) and \((-1)^{k-2}=(-1)^k\text{:}\)
\begin{align*} f^{(k)}(x)&=(-1)^{k-2}(k-2)!x^{-(k-1)}&f^{(k)}(1)&=(-1)^k(k-2)!\\ \end{align*}Now we use the standard form of a Taylor polynomial. Since the first two terms don’t fit the pattern, we add those outside of the sigma.
\begin{align*} T_{100}(x)&=\sum_{k=0}^{100}\frac{f^{(k)}(1)}{k!}(x-1)^k\\ &=f(1)+f'(1)(x-1)+\sum_{k=2}^{100}\frac{f^{(k)}(1)}{k!}(x-1)^k\\ &=-1+0(x-1)+\sum_{k=2}^{100}\frac{(-1)^k(k-2)!}{k!}(x-1)^k\\ &=-1+\sum_{k=2}^{100}\frac{(-1)^k}{k(k-1)}(x-1)^k \end{align*}3.4.11.9.
Solution.
3.4.11.10.
Solution.
If \(n=157\) and \(x=1\text{,}\)
\begin{align*} T_{157}(1)&=\sum_{k=0}^{157}\frac{1}{k!}=1+1+\frac{1}{2}+\frac{1}{3!}+\frac{1}{4!}+\cdots +\frac{1}{157!} \end{align*}3.4.11.11.
Solution.
If \(n=100\) and \(x=\dfrac{5\pi}{4}\text{,}\) this equation becomes
\begin{align*} T_{200}\left(\frac{5\pi}{4}\right)&=\sum_{k=0}^{100} \frac{(-1)^k}{(2k)! }\cdot \left(\frac{5\pi}{4}\right)^{2k} \end{align*}Exercises for § 3.4.6
3.4.11.1.
Solution.
3.4.11.2.
Solution.
3.4.11.3.
Solution.
The linear approximation for \(\Delta y\) when \(\Delta x = \dfrac{1}{10}\) is
\begin{align*} \Delta y &\approx f'(5)\Delta x = \frac{1}{26}\cdot \frac{1}{10}=\frac{1}{260} \approx 0.003846 \end{align*}3.4.11.4.
Solution.
From the problem, \(\Delta x= 5-4=1\text{.}\) Differentiating,
\begin{align*} s'(t)&=\sqrt{19.6}\cdot\frac{1}{2\sqrt{x}}=\sqrt{\frac{4.9}{x}},\mbox{ so }\\ s'(4)&=\sqrt{\frac{4.9}{4}}\\ \end{align*}The linear approximation gives us
\begin{align*} \Delta y& \approx \sqrt{\frac{4.9}{4}}(1)\approx1.1 \end{align*}while the change in terminal speed between the next two jumps is
\begin{align*} \Delta y &\approx \sqrt{\frac{4.9}{x+\Delta x}}\cdot\Delta x \end{align*}Exercises for § 3.4.7
3.4.11.1.
Solution.
3.4.11.2.
Solution.
3.4.11.3.
Solution.
3.4.11.4.
Solution.
Differentiating with respect to \(r\text{,}\)
\begin{align*} A'(r)&=2\pi r\\ \end{align*}The exact area desired is \(A_0\text{.}\) Let the corresponding exact radius desired be \(r_0\text{.}\)
\begin{align*} \\ \end{align*}Using the linear approximation formula, where \(\Delta A\) is the change in \(A\) corresponding to a change in \(r\) of \(\Delta r\text{,}\)
\begin{align*} \Delta A &\approx A'(r_0)\Delta r = 2\pi r_0 \Delta r\\ \Delta r &\approx \dfrac{\Delta A}{2\pi r_0}\\ \end{align*}What we’re interested in is the percent error \(r\) can have. The percent error is:
\begin{align*} 100\frac{\Delta r}{r_0}&\approx100\frac{\Delta A}{2\pi r_0\cdot r_0}\\ &=100\frac{\Delta A}{2(\pi r_0^2)}\\ &=100\frac{\Delta A}{2\cdot A_0}\\ &=\left( 100\frac{\Delta A}{A_0}\right)\frac{1}{2}\\ &\leq\left( 2\right)\frac{1}{2}=1 \end{align*}3.4.11.5.
Solution.
Since the question tells us the sector is no more than a quarter of the circle, we know \(0 \leq \theta \leq \dfrac{\pi}{2}\text{,}\) so \(0 \leq \dfrac{\theta}{2} \leq \dfrac{\pi}{4}\text{.}\) This puts \(\dfrac{\theta}{2}\) well within the range of arcsine.
\begin{align*} \theta&=2\arcsin\left(\frac{d}{6}\right) \end{align*}Differentiating,
\begin{align*} A'(d)&=\frac{9}{\sqrt{1-\left(\frac{d}{6}\right)^2}}\cdot\frac{1}{6}\\ &=\frac{9}{\sqrt{36-d^2}}\\ \end{align*}Let \(\Delta A\) is the error in \(A\) corresponding to an error of \(\Delta d\) in \(d\text{.}\) Since we measured \(d\) to be 0.7 instead of \(0.68\text{,}\) in the linear approximation we take \(\Delta d = 0.02\) and \(d_0=0.68\text{.}\)
\begin{align*} \Delta A &\approx A'(d_0)\cdot\Delta d\\ &=A'\left(0.68\right)\cdot 0.02\\ &=\frac{9}{\sqrt{36-0.68^2}}\cdot0.02\\ &\approx 0.03 \end{align*}3.4.11.6.
Solution.
Using this, we find our equation for the volume of the water in terms of \(h\text{.}\)
\begin{align*} V(h)&=\frac{1}{3}\pi r^2 h = \frac{\pi}{3}\left(\frac{h}{4}\right)^2h=\frac{\pi h^3}{48}\\ \end{align*}Differentiating,
\begin{align*} V'(h)&=\frac{\pi h^2}{16}\\ V'(0.5)&=\frac{0.25\pi}{16}=\frac{\pi}{64}\\ \end{align*}Finally, using the approximation \(\Delta V \approx -0.05V'(0.5)\text{,}\)
\begin{align*} \Delta V &\approx \frac{-0.05\pi}{64}=-\frac{\pi}{1280}\approx -0.00245\;\mathrm{m}^3 \end{align*}3.4.11.7.
Solution.
So,
\begin{align*} \left|\Delta h\right|&\approx\left| h'(0.9)\right|\left|\Delta q\right|= h'(0.9)\cdot 0.05 \end{align*}for some constant \(k\text{.}\) Let’s take \(t=0\) to be the time when precisely one \(\mu\)g was present. Then
\begin{align*} Q(t)&=e^{-kt}\\ \end{align*}After three years, \(q\) is the amount of the isotope remaining, so
\begin{align*} q&=e^{-k\cdot 3}\\ q^{\frac{1}{3}}&=e^{-k}\\ Q(t)&=\left(e^{-k}\right)^t=q^{\frac{t}{3}}\\ \end{align*}The half-life is the value of \(t\) for which \(Q(t)=\frac{1}{2}Q(0)=\frac{1}{2}\text{.}\)
\begin{align*} \frac{1}{2}&=Q(t)=q^{\frac{t}{3}}\\ \log\left(\frac{1}{2}\right)&=\log\left(q^{\frac{t}{3}}\right)\\ -\log 2 &=\frac{t}{3}\log q\\ t&=\frac{-3\log 2}{\log q} \end{align*}Following our plan, we find \(h'(0.9)\text{.}\)
\begin{align*} h'(q)&=\diff{}{q}\left\{\dfrac{-3\log2}{\log q}\right\}\\ &=-3\log 2\cdot\diff{}{q}\left\{\left(\log q\right)^{-1}\right\}\\ &=-3\log 2\cdot(-1)\left(\log q\right)^{-2}\cdot\frac{1}{q}\\ &=\frac{3\log 2}{q\log^2q}\\ h'(0.9)&=\frac{3\log2}{0.9\log^2(0.9)}\approx 208\\ \end{align*}Finally, as outlined in our plan,
\begin{align*} \left|\Delta h\right|&=h'(0.9)\cdot0.05\\ &=\frac{3\log2}{18\log^2(0.9)}\approx 10.4 \end{align*}Exercises for § 3.4.8
3.4.11.1.
Solution.
3.4.11.2.
Solution.
for some \(c\) strictly between \(x\) and \(a\text{.}\) In our case, \(n=3\text{,}\) \(a=0\text{,}\) \(x=2\text{,}\) and \(f^{(4)}(c)=e^c\text{,}\) so
\begin{align*} |f(2)-T_3(2)| &= \left|\frac{f^{(4)}(c)}{4!}(2-0)^4\right|\\ &=\frac{2^4}{4!}e^c=\frac{2}{3}e^c\\ \end{align*}Since \(c\) is strictly between 0 and 2, \(e^c \lt e^2\text{:}\)
\begin{align*} &\leq \frac{2}{3}e^2\\ \end{align*}but this isn’t a number we really know. Indeed: \(e^2\) is the very number we’re trying to approximate. So, we use the estimation \(e \lt 3\text{:}\)
\begin{align*} & \lt \frac{2}{3}\cdot 3^2 =6 \end{align*}3.4.11.3.
Solution.
3.4.11.4.
Solution.
while the linear approximation gives
\begin{align*} f(x)&\approx f(0)+f'(0)x\\ \sin(x)& \approx \sin (0)+\cos(0) x\\ &= x\\ \sin(33)&\approx 33 \end{align*}3.4.11.5.
Solution.
for some \(c\) strictly between \(x\) and \(a\text{.}\) In our case, \(n=5\text{,}\) \(a=11\text{,}\) \(x=11.5\text{,}\) and \(f^{(6)}(c)=\dfrac{6!(2c-5)}{c+3}\text{.}\)
\begin{align*} \left|f(11.5)-T_5(11.5)\right|& = \left|\dfrac{1}{6!}\left(\dfrac{6!(2c-5)}{c+3}\right)(11.5-11)^{6}\right|\\ & = \left|\dfrac{2c-5}{c+3}\right|\cdot\frac{1}{2^6} \end{align*}- \(\displaystyle 2c-5 \lt 2(11.5)-5=18\)
- \(\displaystyle c+3 \gt 11+3=14\)
- Therefore, \(\left|\dfrac{2c-5}{c+3}\right|=\dfrac{2c-5}{c+3} \lt \dfrac{18}{14}=\dfrac{9}{7}\) when \(c \in (11,11.5)\text{.}\)
3.4.11.6.
Solution.
for some \(c\) strictly between \(x\) and \(a\text{.}\) In our case, \(n=2\text{,}\) \(a=0\text{,}\) and \(x=0.1\text{,}\) so
\begin{align*} \left|f(0.1)-T_2(0.1)\right| &= \left|\frac{f^{(3)}(c)}{3!}(0.1-0)^{3}\right|\\ &=\frac{\left|f'''(c)\right|}{6000} \end{align*}- \(\displaystyle \tan c \lt \tan\left(\dfrac{\pi}{6}\right)=\dfrac{1}{\sqrt{3}}\)
- \(\displaystyle \cos c \gt \cos\left(\dfrac{\pi}{6}\right)=\dfrac{\sqrt{3}}{2}\)
- \(\displaystyle \sec c \lt \dfrac{2}{\sqrt{3}}\)
3.4.11.7.
Solution.
for some \(c\) strictly between \(x\) and \(a\text{.}\) In our case, \(n=5\text{,}\) \(a=0\text{,}\) and \(x=-\dfrac{1}{4}\text{,}\) so
\begin{align*} \left|f\left(-\frac{1}{4}\right)-T_5\left(-\frac{1}{4}\right)\right| &= \left|\frac{f^{(6)}(c)}{6!}\left(-\frac{1}{4}-0\right)^6\right|\\ &= \frac{\left|f^{(6)}(c)\right|}{6!\cdot 4^6}\\ \end{align*}for some \(c\) in \(\left(-\frac{1}{4},0\right)\text{.}\) We’ll need to know the sixth derivative of \(f(x)\text{.}\)
\begin{align*} f(x)&=\log (1-x)\\ f'(x)&=-(1-x)^{-1}\\ f''(x)&=-(1-x)^{-2}\\ f'''(x)&=-2(1-x)^{-3}\\ f^{(4)}(x)&=-3!(1-x)^{-4}\\ f^{(5)}(x)&=-4!(1-x)^{-5}\\ f^{(6)}(x)&=-5!(1-x)^{-6}\\ \end{align*}Plugging in \(\left|f^{(6)}(c)\right|=\dfrac{5!}{(1-c)^{6}}\text{:}\)
\begin{align*} \left|f\left(-\frac{1}{4}\right)-T_5\left(-\frac{1}{4}\right)\right| &= \frac{5!}{6!\cdot 4^6\cdot(1-c)^6}=\frac{1}{6\cdot 4^6 \cdot (1-c)^6} \end{align*}- Since \(-\dfrac{1}{4}\leq c \leq 0\text{,}\) the smallest absolute value of \(1-c\) occurs when \(c=0\text{.}\) In other words, \(|1-c| \leq 1\text{.}\)
- That means the smallest possible value of \((1-c)^6\) is \(1^6=1\text{.}\)
- Then the largest possible value of \(\dfrac{1}{(1-c)^6}\) is 1.
- Then the largest possible value of \(\dfrac{1}{6\cdot 4^6}\cdot\dfrac{1}{(1-c)^6}\) is \(\dfrac{1}{6\cdot 4^6}\approx 0.0000407\text{.}\)
3.4.11.8.
Solution.
for some \(c\) strictly between \(x\) and \(a\text{.}\) In our case, \(n=3\text{,}\) \(a=30\text{,}\) and \(x=32\text{,}\) so
\begin{align*} |f(30)-T_3(30)| &= \left|\frac{f^{(4)}(c)}{4!}(30-32)^{4}\right|\\ &=\frac{2}{3}\left|f^{(4)}(c)\right| \end{align*}Since \(30 \lt c \lt 32\text{,}\)
\begin{align*} & \lt \frac{336}{5^4\cdot 30^{\tfrac{19}{5}}} =\frac{336}{5^4\cdot 30^3 \cdot 30^{\tfrac{4}{5}}}\\ &=\frac{14}{5^7\cdot 9 \cdot 30^{\tfrac{4}{5}}}\\ \end{align*}This isn’t a number we know. We’re trying to find the error in our estimation of \(\sqrt[5]{30}\text{,}\) but \(\sqrt[5]{30}\) shows up in our error. From here, we have to be a little creative to get a bound that actually makes sense to us. There are different ways to go about it. You could simply use \(30^{\tfrac{4}{5}} \gt 1\text{.}\) We will be a little more careful, and use the following estimation:
\begin{align*} \frac{14}{5^7 \cdot 9 \cdot \textcolor{red}{30^{\tfrac{4}{5}}}}&= \frac{14\cdot \textcolor{red}{30^{\tfrac{1}{5}}}}{5^7 \cdot 9 \cdot \textcolor{red}{30}}\\ & \lt \frac{14\cdot \textcolor{red}{32^{\tfrac{1}{5}}}}{5^7 \cdot 9 \cdot \textcolor{red}{30}}\\ & \lt \frac{14\cdot \textcolor{red}{2}}{5^7 \cdot 9 \cdot \textcolor{red}{30}}\\ & \lt \frac{14}{5^7\cdot 9 \cdot \textcolor{red}{15}}\\ & \lt 0.000002 \end{align*}3.4.11.9.
Solution.
for some \(c\) strictly between \(x\) and \(a\text{.}\) In our case, \(n=1\text{,}\) \(a=\dfrac{1}{\pi}\text{,}\) and \(x=0.01\text{,}\) so
\begin{align*} |f(0.01)-T_1(0.01)| &= \left|\frac{f''(c)}{2}\left(0.01-\frac{1}{\pi}\right)^{2}\right|\\ &=\frac{1}{2}\left(\frac{100-\pi}{100\pi}\right)^2\cdot\left|f''(c)\right| \end{align*}- Since \(c \geq \dfrac{1}{100}\text{,}\) it follows that \(c^4 \geq \dfrac{1}{100^4}\text{,}\) so \(\dfrac{1}{c^4}\leq 100^4\text{.}\)
- For any value of \(x\text{,}\) \(|\cos x|\) and \(|\sin x|\) are at most 1. Since \(|c| \lt 1\text{,}\) also \(|c\cos\left(\frac{1}{c}\right)| \lt |\cos\left(\frac{1}{c}\right)| \leq 1\text{.}\) So, \(\left|2c\cos\left(\frac{1}{c}\right)-\sin\left(\frac{1}{c}\right)\right| \lt 3\)
- Therefore,\begin{align*} \amp|f(0.01)-T_n(0.01)|\\ \amp\hskip0.5in =\,\dfrac{1}{2}\left(\dfrac{100-\pi}{100\pi}\right)^2\cdot\frac{1}{c^4}\cdot \left|2c\cos\left(\frac{1}{c}\right)-\sin\left(\frac{1}{c}\right)\right|\\ \amp\hskip0.5in \lt\,\dfrac{1}{2}\left(\dfrac{100-\pi}{100\pi}\right)^2\cdot100^4\cdot3\\ \amp\hskip0.5in=\,\frac{3\cdot 100^2}{2}\left(\frac{100}{\pi}-1\right)^2 \end{align*}
3.4.11.10.
Solution.
for some \(c\) strictly between \(x\) and \(a\text{.}\) In our case, \(n=2\text{,}\) \(a=0\text{,}\) and \(x=\dfrac{1}{2}\text{,}\) so
\begin{align*} \left|f\left(\frac{1}{2}\right)-T_2\left(\frac{1}{2}\right)\right| &= \left|\frac{f^{(3)}(c)}{3!}\left(\frac{1}{2}-0\right)^{3}\right|\\ &= \frac{\left|f^{(3)}(c)\right|}{3!\cdot2^3} \end{align*}Since \(\left|f\left(\frac{1}{2}\right)-T_2\left(\frac{1}{2}\right)\right| =\dfrac{\left|f^{(3)}(c)\right|}{3!\cdot2^3}\) for some \(c\) in \(\left(0,\frac{1}{2}\right)\text{,}\)
\begin{align*} \left|f\left(\frac{1}{2}\right)-T_2\left(\frac{1}{2}\right)\right| &=\dfrac{\left|\dfrac{1+2c^2} {\left(\sqrt{1-c^2}\right)^5} \right|}{3!\cdot2^3}= \frac{1+2c^2}{48\left(\sqrt{1-c^2}\right)^5} \end{align*}-
When \(0 \leq c \leq\dfrac{1}{2}\text{,}\) the biggest \(1+2c^2\) can be is \(1+2\left(\dfrac{1}{2}\right)^2=\dfrac{3}{2}\text{.}\)So, the numerator of \(\dfrac{1+2c^2}{48\left(\sqrt{1-c^2}\right)^5}\) is at most \(\dfrac{3}{2}\text{.}\)
- The smallest \(1-c^2\) can be is \(1-\left(\dfrac{1}{2}\right)^2=\dfrac{3}{4}\text{.}\)
- So, the smallest \(\left(\sqrt{1-c^2}\right)^5\) can be is \(\left(\sqrt{\dfrac{3}{4}}\right)^5=\left(\dfrac{\sqrt{3}}{2}\right)^5\text{.}\)
- Then smallest possible value for the denominator of \(\dfrac{1+2c^2}{48\left(\sqrt{1-c^2}\right)^5}\) is \(48\left(\dfrac{\sqrt{3}}{2}\right)^5\)
- Then\begin{align*} \dfrac{1+2c^2}{48\left(\sqrt{1-c^2}\right)^5} & \leq \frac{\dfrac{3}{2}}{48\left(\dfrac{\sqrt{3}}{2}\right)^5}\\ &=\frac{1}{\sqrt{3}^5}=\frac{1}{9\sqrt{3}}\\ & \lt \frac{1}{10} \end{align*}
Conveniently, we’ve already found the first few derivatives of \(f(x)\text{.}\)
\begin{align*} T_2(x)&=\arcsin(0)+\left(\frac{1}{\sqrt{1-0^2}}\right)x+\frac{1}{2}\left(\frac{0}{\left(\sqrt{1-0^2}\right)^3}\right)x^2\\ &=0+x+0\\ &=x\\ T_2\left(\frac{1}{2}\right)&=\frac{1}{2}\\ \end{align*}So, the actual error is
\begin{align*} \left|f\left(\frac{1}{2}\right)-T_2\left(\frac{1}{2}\right)\right|&=\left| \frac{\pi}{6}-\frac{1}{2}\right|= \frac{\pi}{6}-\frac{1}{2} \end{align*}3.4.11.11.
Solution.
So, when \(n \geq 1\text{,}\)
\begin{align*} f^{(n)}(x)&=(-1)^{n-1}(n-1)!\cdot x^{-n} \end{align*}for some \(c\) in \((1,1.1)\)
\begin{align*} & \lt \frac{1}{(n+1)10^{n+1}\cdot 1^{n+1}}\\ &=\frac{1}{(n+1)10^{n+1}}\\ \end{align*}What we’ve shown so far is
\begin{align*} |f(1.1)-T_n(1.1)|& \lt \dfrac{1}{(n+1)10^{n+1}}\\ \end{align*}If we can show that \(\dfrac{1}{(n+1)10^{n+1}}\le10^{-4}\text{,}\) then we’ll be able to conclude
\begin{align*} |f(1.1)-T_n(1.1)|& \lt \dfrac{1}{(n+1)10^{n+1}}\le10^{-4} \end{align*}3.4.11.12.
Solution.
for some \(c\) in \((2187,2200)\text{.}\) In order for this to be less than 0.001, we need
\begin{align*} \left|f^{(n+1)}(c)\right|\cdot\frac{13^{n+1}}{(n+1)!}& \lt 0.001\\ \left|f^{(n+1)}(c)\right|& \lt \frac{ (n+1)!}{1000\cdot13^{n+1}} \end{align*}3.4.11.13.
Solution.
for some \(c\) between 0 and 1. Since \(-1 \leq \cos c \leq 1\text{,}\)
\begin{align*} \frac{4242-1}{7!}\leq \sin(1)&\leq \frac{4242+1}{7!}\\ \frac{4241}{7!}\leq \sin(1)&\leq \frac{4243}{7!}\\ \frac{4241}{5040}\leq \sin(1)&\leq \frac{4243}{5040} \end{align*}3.4.11.14.
Solution.
Since \(e^x\) is a strictly increasing function, and \(0 \lt c \lt 1\text{,}\) we conclude \(e^0 \lt e^c \lt e^1\text{:}\)
\begin{align*} \frac{65}{24} +\frac{1}{120}& \lt e \lt \frac{65}{24} +\frac{e}{120}\\ \end{align*}Simplifying the left inequality, we see
\begin{align*} \frac{326}{120}& \lt e\\ \end{align*}From the right inequality, we see
\begin{align*} e& \lt \frac{65}{24}+\frac{e}{120}\\ e-\frac{e}{120}& \lt \frac{65}{24}\\ e\cdot \frac{119}{120}& \lt \frac{65}{24}\\ e& \lt \frac{65}{24}\cdot \frac{120}{119}=\frac{325}{119}\\ \end{align*}So, we conclude
\begin{align*} \frac{326}{120}& \lt e \lt \frac{325}{119}, \end{align*}Further problems for § 3.4
3.4.11.1. (✳).
Solution.
3.4.11.2. (✳).
Solution.
3.4.11.3. (✳).
Solution.
3.4.11.4. (✳).
Solution.
- For \(x\) near 3,\begin{equation*} f(x)\approx f(3)+f'(3)(x-3)=2+4(x-3) \end{equation*}In particular\begin{equation*} f(2.98)\approx 2+4(2.98-3)=2-0.08=1.92 \end{equation*}
- For \(x\) near 3,\begin{align*} f(x)&\approx f(3)+f'(3)(x-3)+\frac{1}{2} f''(3)(x-3)^2\\ &=2+4(x-3)-\frac{1}{2} 10(x-3)^2\\ \end{align*}
In particular
\begin{align*} f(2.98)&\approx 2+4(2.98-3)-5(2.98-3)^2\\ &=2-0.08-0.002={1.918} \end{align*}
3.4.11.5. (✳).
Solution.
Using the error formula:
\begin{align*} g(10)&=g(8)+g'(8)(10-8)+\half g''(c)(10-8)^2 \end{align*}3.4.11.6. (✳).
Solution.
3.4.11.7. (✳).
Solution.
3.4.11.8. (✳).
Solution.
3.4.11.9. (✳).
Solution.
So, the linear approximation of \(\sin\left(\frac{101\pi}{100}\right) \) is
\begin{align*} f\left(\frac{101\pi}{100}\right)&\approx T_1\left(\frac{101\pi}{100}\right)\\ &= f(\pi) + f'(\pi)\cdot \left(\frac{101\pi}{100}-\pi\right)\\ &=0+(-1)\cdot\frac{\pi}{100}=-\frac{\pi}{100}. \end{align*}3.4.11.10. (✳).
Solution.
3.4.11.11. (✳).
Solution.
3.4.11.12. (✳).
Solution.
3.4.11.13. (✳).
Solution.
3.4.11.14. (✳).
Solution.
- By Equation 3.4.33, the absolute value of the error is\begin{equation*} \left|\frac{f'''(c)}{3!}\cdot (2-1)^3\right| = \left|\frac{c}{6(22-c^2)}\right| \end{equation*}for some \(c \in (1,2)\text{.}\)
- When \(1\leq c\leq2\text{,}\) we know that \(18 \leq 22-c^2 \leq 21\text{,}\) and that numerator and denominator are non-negative, so\begin{align*} \left|\frac{c}{6(22-c^2)}\right| &=\frac{c}{6(22-c^2)} \leq \frac{2}{6(22-c^2)} \leq \frac{2}{6\cdot 18}\\ & = \frac{1}{54} \leq \frac{1}{50} \end{align*}as required.
- Alternatively, notice that \(c\) is an increasing function of \(c\text{,}\) while \(22-c^2\) is a decreasing function of \(c\text{.}\) Hence the fraction is an increasing function of \(c\) and takes its largest value at \(c=2\text{.}\) Hence\begin{align*} \left|\frac{c}{6(22-c^2)}\right| & \leq \frac{2}{6\times 18} = \frac{1}{54} \leq \frac{1}{50}. \end{align*}
3.4.11.15. (✳).
Solution.
- By Equation 3.4.33, there is \(c\in(0,0.5)\) such that the error is\begin{align*} R_4 &= \frac{f^{(4)}(c)}{4!} (0.5-0)^4\\ &= \frac{1}{24\cdot 16} \cdot \frac{\cos(c^2)}{3-c} \end{align*}
- For any \(c\) we have \(|\cos(c^2)| \leq 1\text{,}\) and for \(c \lt 0.5\) we have \(3-c \gt 2.5\text{,}\) so that\begin{equation*} \left|\frac{\cos(c^2)}{3-c}\right| \leq \frac{1}{2.5}\,. \end{equation*}
- We conclude that\begin{equation*} \left| R_4 \right| \leq \frac{1}{2.5\cdot 24\cdot 16} = \frac{1}{60\cdot 16} \lt \frac{1}{60\cdot 10}=\frac{1}{600} \lt \frac{1}{500} \end{equation*}
3.4.11.16. (✳).
Solution.
- By Equation 3.4.33, there is \(c\in(0,1)\) such that the error is\begin{align*} \left|\frac{f'''(c)}{3!}\cdot (1-0)^3\right| &= \left|\frac{e^{-c}}{6(8+c^2)}\right|. \end{align*}
- When \(0 \lt c \lt 1\text{,}\) we know that \(1 \gt e^{-c} \gt e^{-1}\) and \(8 \leq 8+c^2 \lt 9\text{,}\) so\begin{align*} \left|\frac{e^{-c}}{6(8+c^2)}\right| &=\frac{e^{-c}}{6(8+c^2)}\\ & \lt \frac{1}{6 |8+c^2|}\\ & \lt \frac{1}{6\times 8} = \frac{1}{48} \lt \frac{1}{40} \end{align*}as required.
- Alternatively, notice that \(e^{-c}\) is a decreasing function of \(c\text{,}\) while for \(0 \lt c\) \(8+c^2\) is an increasing function of \(c\text{.}\) Hence the fraction is a decreasing function of \(c\) and takes its largest value at \(c=0\text{.}\) Hence\begin{align*} \left|\frac{e^c}{6(8+c^2)}\right| & \leq \frac{1}{6\times 8} = \frac{1}{48} \lt \frac{1}{40}. \end{align*}
3.4.11.17. (✳).
Solution.
3.4.11.18.
Solution.
while the third order Maclaurin polynomial for \(f(x)\) is
\begin{align*} T_3(x)&=f(0)+f'(0)x+\frac{1}{2}f''(0)x^2+\frac{1}{3!}f'''(0)x^3\\ \end{align*}So, we simply “chop off” the part of \(T_4(x)\) that includes \(x^4\text{.}\) Since that’s already 0, in this case \(T_3(x)=T_4(x)\text{.}\)
\begin{align*} T_3(x)&=5x^2-9 \end{align*}3.4.11.19. (✳).
Solution.
By implicit differentiation (and then subbing in \(x=2\text{,}\) \(y(2)=1\))
\begin{align*} 4y(x)^3y'(x)+y(x)+xy'(x)&=2x\\ 4y'(2)+1+2y'(2)&=4\\ y'(2)&=\frac{1}{2} \end{align*}In particular,
\begin{align*} y(2.1)&\approx y(2)+y'(2)(2.1-2)=1+\frac{1}{2}(.1)=\boxed{1.05}\\ \end{align*}The quadratic approximation to \(y(x)\) at \(x=2\) is
\begin{align*} y(x)&\approx y(2)+y'(2)(x-2)+\frac{1}{2} y''(2)(x-2)^2\\ &=1+\frac{1}{2}(x-2)-\frac{1}{6}(x-2)^2\\ \end{align*}In particular,
\begin{align*} y(2.1)&\approx y(2)+y'(2)(2.1-2)+\frac{1}{2} y''(2)(2.1-2)^2\\ &=1+\frac{1}{2}(.1)-\frac{1}{6}(.1)^2=\boxed{1.0483} \end{align*}3.4.11.20. (✳).
Solution.
By implicit differentiation (and then subbing in \(x=-1\text{,}\) \(y(-1)=1\))
\begin{align*} 0&=4x^3+y'(x)+y(x)^4+4xy(x)^3y'(x)\\ 0&= -4+y'(-1)+1-4y'(-1)\\ -1&= y'(-1)\\ \end{align*}Differentiating with respect to \(x\) a second time and then subbing in \(x=-1\text{,}\) \(y(-1)=1\text{,}\) and \(y'(-1)=-1\text{:}\)
\begin{align*} 0&=12x^2+y''(x)+4y(x)^3y'(x)+4y(x)^3y'(x)+12xy(x)^2y'(x)^2\\ \amp\hskip3in+4xy(x)^3y''(x)\\ 0&= 12+y''(-1)-4-4-12-4y''(-1)\\ -8&= 3y''(-1)\\ y''(-1)&=-\frac{8}{3} \end{align*}In particular,
\begin{align*} y(-0.9)&\approx 0.9 \end{align*}In particular,
\begin{align*} y(-0.9)&\approx 1-(.1)-\frac{4}{3}(.1)^2\approx{0.8867} \end{align*}3.4.11.21. (✳).
Solution.
3.4.11.22. (✳).
Solution.
3.5 Optimisation
3.5.4 Exercises
Exercises for § 3.5.1
3.5.4.1.
Solution.
3.5.4.2.
Solution.
3.5.4.3.
Solution.
3.5.4.4.
Solution.
3.5.4.5.
Solution.
3.5.4.6.
Solution.
3.5.4.7.
Solution.
The product \((x-5)(x+7)\) is positive when \((x-5)\) and \((x+7)\) have the same sign, and negative when they have opposite signs, so
\begin{align*} f(x)&=\left\{\begin{array}{ll} \sqrt{(x-5)(x+7)}&\mbox{ if } x\in (-\infty,-7] \cup [5,\infty)\\ \sqrt{-(x-5)(x+7)}&\mbox{ if } x\in(-7,5) \end{array}\right.\\ \end{align*}Now, when \(x \neq -7,5\text{,}\) we can differentiate, using the chain rule.
\begin{align*} f'(x)&=\left\{\begin{array}{ll} \frac{\diff{}{x}\left\{(x-5)(x+7)\right\}}{2\sqrt{(x-5)(x+7)}}&\mbox{ if } x\in (-\infty,-7) \cup (5,\infty)\\ \frac{\diff{}{x}\left\{-(x-5)(x+7)\right\}}{2\sqrt{-(x-5)(x+7)}}&\mbox{ if } x\in(-7,5)\\ ?? & \mbox{ if } x=-7,\,x=5 \end{array}\right.\\ &=\left\{\begin{array}{ll} \frac{2x+2}{2\sqrt{(x-5)(x+7)}}&\mbox{ if } x\in (-\infty,-7) \cup (5,\infty)\\ \frac{-2x-2}{2\sqrt{-(x-5)(x+7)}}&\mbox{ if } x\in(-7,5)\\ ?? & \mbox{ if } x=-7,\,x=5 \end{array}\right. \end{align*}- First, let’s notice that \(f(x)\) is defined for all values of \(x\text{,}\) thanks to that handy absolute value sign.
- Next, notice \(f(x) \geq 0\) for all \(x\text{,}\) since square roots never give a negative value.
- Then if there is some value of \(x\) that gives \(f(x)=0\text{,}\) that \(x\) gives a global minimum, and therefore a local minimum.
- \(f(x)=0\) exactly when \((x-5)(x+7)=0\text{,}\) which occurs at \(x=-7\) and \(x=5\)
- Therefore, \(f(x)\) has global and local minima at \(x=-7\) and \(x=5\)
Let’s first consider the case \(h \gt 0\text{.}\)
\begin{align*} \lim_{h \to 0^+} \frac{\sqrt{|(-13+h)(h)|}}{h}&=\lim_{h \to 0^+}\frac{\sqrt{(13-h)(h)}}{h}\\ &=\lim_{h \to 0^+}\frac{\sqrt{13h-h^2}}{\sqrt{h^2}}\\ &=\lim_{h \to 0^+}\sqrt{\frac{13h-h^2}{h^2}}\\ &=\lim_{h \to 0^+}\sqrt{\frac{13}{h}-1}\\ &=\infty\\ \end{align*}Since one side of the limit doesn’t exist,
\begin{align*} \lim_{h \to 0} \frac{f(-7+h)-(-7)}{h}&=DNE \end{align*}3.5.4.8.
Solution.
Exercises for § 3.5.2
3.5.4.1.
Solution.
3.5.4.2.
Solution.
3.5.4.3.
Solution.
3.5.4.4.
Solution.
\(c\) | \(-3\) | \(-5\) | \(5\) |
type | critical point | endpoint | endpoint |
\(f(c)\) | \(-19\) | \(-15\) | \(45\) |
3.5.4.5.
Solution.
\(c\) | \(-3\) | \(-4\) | \(0\) |
type | critical point | endpoint | endpoint |
\(f(c)\) | \(61\) | \(\frac{157}{3}=52+\frac{1}{3}\) | \(7\) |
Exercises for § 3.5.3
3.5.4.1. (✳).
Solution.
\(c\) | \(-2\) | \(0\) | \(-1\) |
type | endpoint | endpoint | critical point |
\(f(c)\) | \(-20\) | \(2\) | \(6\) |
3.5.4.2. (✳).
Solution.
\(c\) | \(2\) | \(0\) | \(1\) |
type | endpoint | endpoint | critical point |
\(f(c)\) | \(12\) | \(-10\) | \(-14\) |
3.5.4.3. (✳).
Solution.
\(c\) | \(1\) | \(4\) | \(2\) |
type | endpoint | endpoint | critical point |
\(f(c)\) | \(-6\) | \(30\) | \(-10\) |
3.5.4.4. (✳).
Solution.
\(x\) | \((-\infty,-2)\) | \((-2,2)\) | \((2,\infty)\) |
\(h'(x)\) | \(\gt 0\) | \(\lt 0\) | \(\gt 0\) |
\(h(x)\) | increasing | decreasing | increasing |
3.5.4.5. (✳).
Solution.
\(x\) | \((-\infty,-2)\) | \((-2,2)\) | \((2,\infty)\) |
\(h'(x)\) | \(\gt 0\) | \(\lt 0\) | \(\gt 0\) |
\(h(x)\) | increasing | decreasing | increasing |
3.5.4.6. (✳).
Solution.
3.5.4.7. (✳).
Solution.
Therefore,
\begin{align*} \ell&=3w=30\\ h&=\frac{1500}{w^2}=15. \end{align*}3.5.4.8. (✳).
Solution.
3.5.4.9. (✳).
Solution.
3.5.4.10. (✳).
Solution.
So, this is what we optimize. The endpoints of the domain for this function are \(x=0\) and \(x=R\text{.}\) To find the critical points, we differentiate:
\begin{align*} P'(x)&=4-\dfrac{2x}{\sqrt{R^2-x^2}}\\ P'(x)=0\iff 4&=\dfrac{2x}{\sqrt{R^2-x^2}}\\ x&=2\sqrt{R^2-x^2}\\ x^2&=4(R^2-x^2)\\ 5x^2&=4R^2\\ x&=\dfrac{2}{\sqrt{5}}R \end{align*}\(c\) | \(0\) | \(R\) | \(\frac{2}{\sqrt{5}}R\) |
type | endpoint | endpoint | critical point |
\(P(c)\) | \(2R\) | \(4R\) | \(2\sqrt{5}R\) |
3.5.4.11. (✳).
Solution.
This is the function we want to maximize. Let’s find its critical points.
\begin{align*} V'(r)&=\dfrac{1}{2}\left(A-6\pi r^2\right)\\ V'(r)=0 &\iff A=6\pi r^2 \iff r=\sqrt{\dfrac{A}{6\pi}}\\ \end{align*}since negative values of \(r\) don’t make sense. At this critical point,
\begin{align*} V\left(\sqrt{\dfrac{A}{6\pi}}\right)&= \dfrac{1}{2}\left[A\left(\sqrt{\dfrac{A}{6\pi}}\right)-2\pi \left(\sqrt{\dfrac{A}{6\pi}}\right)^3\right]\\ &=\dfrac{1}{2}\left[\dfrac{A^{3/2}}{\sqrt{6\pi}}-\dfrac{2\pi A^{3/2}}{6\pi\sqrt{6\pi}}\right]\\ &=\dfrac{1}{2}\left[\dfrac{A^{3/2}}{\sqrt{6\pi}}-\dfrac{A^{3/2}}{3\sqrt{6\pi}}\right]\\ &=\dfrac{A^{3/2}}{3\sqrt{6\pi}}. \end{align*}3.5.4.12. (✳).
Solution.
Finding all critical points:
\begin{align*} 0=A'(r)&=P-(\pi+4)r\\ r&=\dfrac{P}{\pi+4} \end{align*}3.5.4.13. (✳).
Solution.
- The surface area of the pan is\begin{align*} xy+2xz+2yz&=px^2+2xz+2pxz\\ &=px^2+2(1+p)xz \end{align*}and the volume of the pan is \(xyz=px^2z\text{.}\) Assuming that all \(A\,{\rm cm}^2\) is used, we have the constraint\begin{equation*} px^2+2(1+p)xz=A \qquad\hbox{or}\quad z=\dfrac{A-px^2}{2(1+p)x} \end{equation*}So\begin{align*} V(x)&=xyz=x(px)\left(\frac{A-px^2}{2(1+p)x}\right)\\ &=\dfrac{p}{2(1+p)}x(A-px^2)\\ \end{align*}
Using the product rule,
\begin{align*} V'(x)&=\dfrac{p}{2(1+p)}\left[x(-2px)+(A-px^2)\right]\\ &=\dfrac{p}{2(1+p)}\big[A-3px^2\big] \end{align*}The derivative \(V'(x)\) is 0 when \(x=\sqrt{\dfrac{A}{3p}}\text{.}\) The derivative is positive (i.e. \(V(x)\) is increasing) for \(x \lt \sqrt{\dfrac{A}{3p}}\) and is negative (i.e. \(V(x)\) is decreasing) for \(x \gt \sqrt{\dfrac{A}{3p}}\text{.}\) So the pan of maximum volume has dimensions \(x=\sqrt{\dfrac{A}{3p}}\text{,}\) \(y=p\sqrt{\dfrac{A}{3p}}=\sqrt{\dfrac{Ap}{3}}\) and \(z=\dfrac{2A/3}{2(1+p)\sqrt{A/(3p)}}=\dfrac{\sqrt{Ap}}{\sqrt{3}(1+p)}\text{.}\) - The volume of the pan from part (a) is\begin{equation*} V(p)=\left(\sqrt{\dfrac{A}{3p}}\right)\left(p\sqrt{\dfrac{A}{3p}}\right) \dfrac{\sqrt{Ap}}{\sqrt{3}(1+p)} =\left(\dfrac{A}{3}\right)^{3/2}\dfrac{\sqrt{p}}{1+p} \end{equation*}Since\begin{equation*} \diff{}{p}\left\{\dfrac{\sqrt{p}}{1+p}\right\}=\dfrac{\half (1+p)/\sqrt{p}-\sqrt{p}}{(1+p)^2} =\dfrac{\sqrt{p}\left(\dfrac{1}{p}-1\right)}{2(1+p)^2} \end{equation*}the volume is increasing with \(p\) for \(p \lt 1\) and decreasing with \(p\) for \(p \gt 1\text{.}\) So the maximum volume is achieved for \(p=1\) (a square base).
3.5.4.14. (✳).
Solution.
3.5.4.15. (✳).
Solution.
We want to find the global max and min for this function, given the constraint \(0 \leq \ell \leq L\text{,}\) so we find its derivative:
\begin{align*} A'(\ell)&=\dfrac{\ell}{8} -\dfrac{L-\ell}{2\pi} =\dfrac{\pi+4}{8\pi}\ell-\dfrac{L}{2\pi}\\ \end{align*}Now, we find the critical point.
\begin{align*} A'(\ell)&=0\\ \frac{\pi+4}{8\pi}\ell&=\frac{L}{2\pi}\\ \ell&=\frac{4L}{\pi+4} \end{align*}\(\ell\) | \(0\) | \(L\) | \(\textcolor{white}{\dfrac{.^.}{.}}\frac{4L}{\pi+4}\) |
type | endpoint | endpoint | critical point |
\(A(\ell)\) | \(\textcolor{white}{\dfrac{.^.}{.}}\frac{L^2}{4\pi}\) | \(\frac{L^2}{16}_{}\) | \(A\left(\frac{4L}{\pi+4}\right)\) |
3.6 Sketching Graphs
3.6.7 Exercises
Exercises for § 3.6.1
3.6.7.1.
Solution.
3.6.7.2.
Solution.
3.6.7.3.
Solution.
3.6.7.4.
Solution.
where \(a\text{,}\) \(b\text{,}\) ad \(c\) are some constants. Remember, for rational functions, you can figure out the end behaviour by looking only at the terms with the highest degree--the others won’t matter, so we don’t bother finding them. From here, we divide the numerator and denominator by the highest power of \(x\) in the denominator, \(x^3\text{.}\)
\begin{align*} &=\lim_{x \to \pm \infty}\dfrac{2x^3+ax^2+bx+c}{3x^3-81}\left(\frac{\tfrac{1}{x^3}}{\tfrac{1}{x^3}}\right)\\ &=\lim_{x \to \pm \infty}\dfrac{2+\tfrac{a}{x}+\tfrac{b}{x^2}+\tfrac{c}{x^3}}{3-\tfrac{81}{x^3}}\\ &=\dfrac{2+0+0+0}{3-0}=\frac{2}{3} \end{align*}3.6.7.5.
Solution.
So, there’s no horizontal asymptote as \(x \to \infty\text{.}\)
\begin{align*} \lim_{x \to -\infty}10^{3x-7}&=\underbrace{\lim_{X \to -\infty}10^{X}}_{\mbox{let }X=3x-7}\\ &=\underbrace{\lim_{X' \to \infty}10^{-X'}}_{\mbox{let }X'=-X}\\ &=\lim_{X' \to \infty}\frac{1}{10^{X'}}\\ &=0 \end{align*}Exercises for § 3.6.2
3.6.7.1.
Solution.
3.6.7.2. (✳).
Solution.
3.6.7.3. (✳).
Solution.
3.6.7.4. (✳).
Solution.
Exercises for § 3.6.3
3.6.7.1.
Solution.
3.6.7.2.
Solution.
3.6.7.3.
Solution.
3.6.7.4. (✳).
Solution.
3.6.7.5. (✳).
Solution.
3.6.7.6. (✳).
Solution.
- Since \(f'(x)\) exists everywhere, there are no singular points.
- If the maximum were achieved at a critical point, that critical point would have to provide both the absolute maximum and the absolute minimum (by part a). So, the function would have to be a constant and consequently could not have a nonzero second derivative. So the maximum is not at a critical point.
3.6.7.7.
Solution.
- \(f''(a) \lt 0\) and \(f''(x) \lt 0\) for every \(x\) between \(a\) and 3, and
- \(f''(b) \gt 0\) and \(f''(x) \gt 0\) for every \(x\) between \(b\) and 3.
- \(f''(x) \lt 0\) (and hence \(f''(x) \neq 0\)) for all \(x\) between \(a\) and 3
- \(f''(x) \gt 0\) (and hence \(f''(x) \neq 0\)) for all \(x\) between \(b\) and 3
- So, any number between \(a\) and \(b\) that is not 3 has \(f''(x)\neq 0\text{.}\)
Exercises for § 3.6.4
3.6.7.1.
Solution.
3.6.7.2.
Solution.
3.6.7.3.
Solution.
3.6.7.4.
Solution.
3.6.7.5.
Solution.
3.6.7.6.
Solution.
3.6.7.7.
Solution.
3.6.7.8.
Solution.
- \(f(-x)=f(-1)=4\text{,}\)
- \(f(x)=f(1)=6\text{,}\) and
- \(-f(x)=-f(1)=-6\text{.}\)
3.6.7.9.
Solution.
Replacing \(X\) with \(\pi x\text{:}\)
\begin{align*} \tan(\pi x + \pi)&=\tan(\pi x)&&\mbox{for any $x$ in the domain of $\tan(\pi x)$}\\ \tan(\pi(x+1))&=\tan(\pi x)&&\mbox{for any $x$ in the domain of $\tan(\pi x)$}\\ f(x+1)&=f(x)&&\mbox{for any $x$ in the domain of $\tan(\pi x)$} \end{align*}3.6.7.10.
Solution.
Replacing \(X\) with \(3x\text{:}\)
\begin{align*} \tan(3x)&=\tan(3x+\pi)&&\mbox{for every $x$ in the domain of $\tan 3x$}\\ \tan(3x)&=\tan\left(3\left(x+\frac{\pi}{3}\right)\right)&&\mbox{for every $x$ in the domain of $\tan 3x$}\\ g(x)&=g\left(x+\frac{\pi}{3}\right)&&\mbox{for every $x$ in the domain of $\tan 3x$} \end{align*}Replacing \(X\) with \(4x\text{:}\)
\begin{align*} \sin(4x)&=\sin(4x+2\pi)&&\mbox{for every $x$ in the domain of $\sin(4x)$}\\ \sin(4x)&=\sin\left(4\left(x+\frac{\pi}{2}\right)\right)&&\mbox{for every $x$ in the domain of $\sin(4x)$}\\ h(x)&=h\left(x+\frac{\pi}{2}\right)&&\mbox{for every $x$ in the domain of $\sin(4x)$} \end{align*}Exercises for § 3.6.6
3.6.7.1. (✳).
Solution.
\(x\) | \((-\infty,2)\) | \(2\) | \((2,3)\) | \(3\) |
\(f'(x)\) | positive | 0 | negative | DNE |
\(f(x)\) | increasing | maximum | decreasing | endpoint |
This is undefined at \(x=3\text{.}\) Indeed,
\begin{align*} \ds\lim_{x \rightarrow 3^-} 3\dfrac{2-x}{2\sqrt{3-x}}&=-\infty, \end{align*}3.6.7.2. (✳).
Solution.
-
Asymptotes:\begin{equation*} \ds\lim_{x \to \pm \infty}f(x) = \ds\lim_{x \to \pm \infty}\dfrac{1}{x}-\dfrac{2}{x^4}=0 \end{equation*}So \(y=0\) is a horizontal asymptote both at \(x=\infty\) and \(x=-\infty\text{.}\)\begin{equation*} \ds\lim_{x \to0}f(x) = \ds\lim_{x \to 0}\dfrac{x^3-2}{x^4}=-\infty \end{equation*}So there is a vertical asymptote at \(x=0\text{,}\) where the function plunges downwards from both the right and the left.
-
Intervals of increase and decrease:\begin{equation*} f'(x)=-\frac{1}{x^2}+\frac{8}{x^5}=\frac{8-x^3}{x^5} \end{equation*}The only place where \(f'(x)\) is zero only at \(x=2\text{.}\) So \(f(x)\) has a horizontal tangent at \(x=2\text{,}\) \(y=\frac{3}{8}\text{.}\) This is a critical point.The derivative is undefined at \(x=0\text{,}\) as is the function.
\(x\) \((-\infty,0)\) \(0\) \((0,2)\) \(2\) \((2,\infty)\) \(f'(x)\) negative DNE positive 0 negative \(f(x)\) decreasing vertical asymptote increasing local max decreasing Since the function changes from increasing to decreasing at \(x=2\text{,}\) the only local maximum is at \(x=2\text{.}\)At this point, we get a rough sketch of \(f(x)\text{.}\) -
Concavity:\begin{equation*} f''(x)=\frac{2}{x^3}-\frac{40}{x^6}=\frac{2x^3-40}{x^6} \end{equation*}The second derivative of \(f(x)\) is positive for \(x \gt \root 3\of 20\) and negative for \(x \lt \root 3\of 20\text{.}\) So the curve is concave up for \(x \gt \root 3\of 20\) and concave down for \(x \lt \root 3\of 20\text{.}\) There is an inflection point at \(x=\root 3\of 20\approx 2.7\text{,}\) \(y=\frac{18}{20^{4/3}}\approx 0.3\text{.}\)
-
Intercepts:Since \(f(x)\) is not defined at \(x=0\text{,}\) there is no \(y\)-intercept. The only \(x\)-intercept is \(x=\sqrt[3]{2}\approx 1.3\text{.}\)
-
Sketch:We can add concavity to our skeleton sketched above, and label our intercept and inflection point (the open dot).
3.6.7.3. (✳).
Solution.
-
Asymptotes:When \(x=-1\text{,}\) the denominator \(1+x^3\) of \(f(x)\) is zero while the numerator is 1, so \(x=-1\) is a vertical asymptote. More precisely,\begin{equation*} \lim_{x \to -1^-}f(x)=-\infty \qquad \lim_{x \to -1^+}f(x)=\infty \end{equation*}There are no horizontal asymptotes, because\begin{equation*} \lim_{x \to \infty} \frac{x^4}{1+x^3}=\infty \qquad \lim_{x \to- \infty} \frac{x^4}{1+x^3}=-\infty \end{equation*}
-
Intervals of increase and decrease:We note that \(f'(x)\) is defined for all \(x \neq -1\) and is not defined for \(x=-1\text{.}\) Therefore, the only singular point for \(f(x)\) is \(x=-1\text{.}\)To find critical points, we set\begin{align*} f'(x)&=0\\ 4x^3+x^6 &=0\\ x^3(4+x^3)&=0\\ x^3=0 \qquad &\mbox{or} \qquad 4+x^3=0\\ x=0 \qquad &\mbox{or} \qquad x=-\sqrt[3]{4}\approx-1.6 \end{align*}At these critical points, \(f(0)=0\) and \(f(-\root 3 \of 4)=\frac{4\root 3 \of 4}{-3} \lt 0\text{.}\) The denominator of \(f'(x)\) is never negative, so the sign of \(f'(x)\) is the same as the sign of its numerator, \(x^3(4+x^3)\text{.}\)
\(x\) \((-\infty,-\sqrt[3]{4})\) \(-\sqrt[3]{4}\) \((-\sqrt[3]{4},-1)\) \(-1\) \((-1,0)\) \(0\) \((0,\infty)\) \(f'(x)\) positive 0 negative DNE negative 0 positive \(f(x)\) increasing l. max decreasing VA decreasing l. min increasing Now, we have enough information to make a skeleton of our graph. -
Concavity:The second derivative is undefined when \(x=-1\text{.}\) It is zero when \(12x^2-6x^5=6x^2(2-x^3)=0\text{.}\) That is, at \(x=\root 3\of 2\approx 1.3\) and \(x=0\text{.}\) Notice that the sign of \(f''(x)\) does not change at \(x=0\text{,}\) so \(x=0\) is not an inflection point.
\(x\) \((-\infty,-1)\) \(-1\) \((-1,0)\) \(0\) \((0,\sqrt[3]{2})\) \(\sqrt[3]{2}\) \((\sqrt[3]{2},\infty)\) \(f''(x)\) negative DNE positive 0 positive 0 negative \(f(x)\) concave down VA concave up concave up IP concave down Now we can refine our skeleton by adding concavity. In the sketch below, closed dots are critical points, and the open dot is an inflection point.
3.6.7.4. (✳).
Solution.
-
Asymptotes:\begin{equation*} \lim_{x \to -\infty}\frac{x^3}{1-x^2}=\infty \qquad \lim_{x \to \infty}\frac{x^3}{1-x^2}=-\infty \end{equation*}So, \(f(x)\) has no horizontal asymptotes.On the other hand \(f(x)\) blows up at both \(x=1\) and \(x=-1\text{,}\) so there are vertical asymptotes at \(x=1\) and \(x=-1\text{.}\) More precisely,\begin{align*} \lim_{x \to -1^-}\frac{x^3}{1-x^2}&=\infty & \lim_{x \to -1^+}\frac{x^3}{1-x^2}&=-\infty\\ \lim_{x \to 1^-}\frac{x^3}{1-x^2}&=\infty & \lim_{x \to 1^+}\frac{x^3}{1-x^2}&=-\infty \end{align*}
-
Symmetry:\(f(x)\) is an odd function, because\begin{equation*} f(-x)=\frac{(-x)^3}{1-(-x)^2}=\frac{-x^3}{1-x^2}=-f(x) \end{equation*}
-
Intercepts:The only intercept of \(f(x)\) is the origin. In particular, that means that out of the three intervals where it is continuous, namely \((-\infty,-1)\text{,}\) \((-1,1)\) and \((1,\infty)\text{,}\) in two of them \(f(x)\) is always positive or always negative.
- When \(x \lt -1\text{:}\) \(1-x^2 \lt 0\) and \(x^3 \lt 0\text{,}\) so \(f(x) \gt 0\text{.}\)
- When \(x \gt 1\text{:}\) \(1-x^2 \lt 0\) and \(x^3 \gt 0\text{,}\) so \(f(x) \lt 0\text{.}\)
- When \(-1 \lt x \lt 0\text{,}\) \(1-x^2 \gt 0\) and \(x^3 \lt 0\text{,}\) so \(f(x) \lt 0\text{.}\)
- When \(0 \lt x \lt 1\text{,}\) \(1-x^2 \gt 0\) and \(x^3 \gt 0\text{,}\) so \(f(x) \gt 0\text{.}\)
-
Intervals of increase and decrease:\begin{equation*} f'(x)=\dfrac{3x^2-x^4}{(1-x^2)^2}=\frac{x^2(3-x^2)}{(1-x^2)^2} \end{equation*}The only singular points are \(x=\pm 1\text{,}\) where \(f(x)\text{,}\) and hence \(f'(x)\text{,}\) is not defined. The critical points are:\begin{align*} f'(x)&=0\\ x^2=0\qquad&\mbox{or}\qquad 3-x^2=0\\ x=0\qquad&\mbox{or}\qquad x=\pm\sqrt{3}\approx\pm 1.7 \end{align*}The values of \(f\) at its critical points are \(f(0)=0\text{,}\) \(f(\sqrt{3})=-\dfrac{3\sqrt{3}}{2}\approx -2.6\) and \(f(-\sqrt{3})=\dfrac{3\sqrt{3}}{2}\approx2.6\text{.}\)Notice the sign of \(f'(x)\) is the same as the sign of \(3-x^2\text{.}\)
\(x\) \((-\infty,-\sqrt{3})\) \(-\sqrt{3}\) \((-\sqrt 3,-1)\) \(-1\) \((-1,0)\) \(f'(x)\) negative 0 positive DNE positive \(f(x)\) decreasing local min increasing VA increasing \(x\) \(0\) \((0,1)\) \(1\) \((1,\sqrt 3)\) \(\sqrt{3}\) \((\sqrt{3},\infty)\) \(f'(x)\) 0 positive DNE positive 0 negative \(f(x)\) increasing VA increasing local max decreasing Now we have enough information to sketch a skeleton of \(f(x)\text{.}\) -
Concavity:\begin{equation*} f''(x)=\frac{2x(3+x^2)}{(1-x^2)^3} \end{equation*}The second derivative is zero when \(x=0\text{,}\) and is undefined when \(x=\pm 1\text{.}\)
\(x\) \((-\infty,-1)\) \((-1,0)\) 0 \((0,1)\) \((1,\infty)\) \(f''(x)\) positive negative 0 positive negative \(f(x)\) concave up concave down inflection point concave up concave down Now, we can refine our skeleton. In the sketch below, closed dots are critical points, and the open dot is an inflection point.
3.6.7.5. (✳).
Solution.
Differentiating again,
\begin{align*} \diff{^2}{x^2}\left\{\dfrac{x^2+3}{3(x+1)}\right\}&= \diff{}{x}\left\{\frac{x^2+2x-3}{3(x+1)^2} \right\}\\ &=\frac{3(x+1)^2(2x+2)-(x^2+2x-3)(6)(x+1)}{9(x+1)^4}\\ \amp\hskip3in\left(\frac{\div3(x+1)}{\div3(x+1)}\right)\\ &=\frac{(x+1)(2x+2)-2(x^2+2x-3)}{3(x+1)^3}\\ &=\frac{8}{3(x+1)^3}\qquad\mbox{where $x \neq -1$}\\ \mbox{so}\qquad f''(x)&=\left\{\begin{array}{ll} e^x&x \lt 0\\ DNE&x=0\\ \frac{8}{3(x+1)^3}&x \gt 0 \end{array}\right. \end{align*}\(x\) | \((-\infty,0)\) | \(0\) | \((0,1)\) | 1 | \((1,\infty)\) |
\(f'(x)\) | positive | DNE | negative | 0 | positive |
\(f(x)\) | increasing | local max | decreasing | local min | increasing |
So, there is no horizontal asymptote to the right.
\begin{align*} \lim_{x \to -\infty} f(x)&=\lim_{x \to -\infty}e^x=0 \end{align*}3.6.7.6. (✳).
Solution.
-
Asymptotes: In the problem statement, we are told:\begin{equation*} \lim_{x \to \pm \infty}\frac{1+2x}{e^{x^2}}=0 \end{equation*}So, \(y=0\) is a horizontal asymptote both at \(x=\infty\) and at \(x=-\infty\text{.}\)Since \(f(x)\) is continuous, it has no vertical asymptotes.
-
Intervals of increase and decrease:The critical points are the zeroes of \(1-x-2x^2=(1-2x)(1+x)\text{.}\) That is, \(x=\frac{1}{2},\ -1\text{.}\)
\(x\) \((-\infty,-1)\) \(-1\) \((-1,\frac{1}{2})\) \(\frac{1}{2}\) \((\frac{1}{2},\infty)\) \(f'(x)\) negative 0 positive 0 negative \(f(x)\) decreasing local min increasing local max decreasing At these critical points, \(f\big(\frac{1}{2}\big)=2e^{-1/4} \gt 0\) and \(f(-1)=-e^{-1} \lt 0\text{.}\)From here, we can sketch a skeleton of the graph. -
Concavity:We are told that we don’t have to actually solve for the inflection points. We just need to know enough to get a basic idea. So, we’ll turn the skeleton of the graph into smooth curve.Inflection points are points where the convexity changes from up to down or vice versa. It looks like our graph is convex down for \(x\) from \(-\infty\) to about \(-1.8\text{,}\) convex up from about \(x=-1.8\) to about \(x=-0.1\text{,}\) convex down from about \(x=-0.1\) to about \(x=1.4\) and convex up from about \(x=1.4\) to infinity. So there are three inflection points at roughly \(x=-1.8,\ -0.1,\ 1.4\text{.}\)
3.6.7.7. (✳).
Solution.
\(x\) | \((-\infty,-1)\) | \(-1\) | \((-1,1)\) | \(1\) | \((1,\infty)\) |
\(f'(x)\) | negative | 0 | positive | 0 | negative |
\(f(x)\) | decreasing | local min | increasing | local max | decreasing |
\(x\) | \((-\infty,-\sqrt{3})\) | \(-\sqrt{3}\) | \((-\sqrt{3},0)\) | 0 | \((0,\sqrt{3})\) | \(\sqrt{3}\) | \((\sqrt{3},\infty)\) |
\(f''(x)\) | negative | 0 | positive | 0 | negative | 0 | positive |
\(f(x)\) | concave down | IP | concave up | IP | concave down | IP | concave up |
3.6.7.8.
Solution.
-
Symmetry:\begin{equation*} f(-x)=-x+2\sin(-x)=-x-2\sin x = -f(x) \end{equation*}So, \(f(x)\) is an odd function. If we can sketch \(y=f(x)\) for nonnegative \(x\text{,}\) we can use symmetry to complete the curve for all \(x\text{.}\)
-
Asymptotes:Since \(f(x)\) is continuous, it has no vertical asymptotes. It also has no horizontal asymptotes, since\begin{equation*} \lim_{x \to -\infty}f(x)=-\infty \qquad \lim_{x \to \infty}f(x)=\infty \end{equation*}
-
Intervals of increase and decrease:Since \(f(x)\) is differentiable everywhere, there are no singular points.\begin{align*} f'(x)&=1+2\cos x\\ \end{align*}
So, the critical points of \(f(x)\) occur when
\begin{align*} \cos x &= -\frac{1}{2}\\ x&=2\pi n \pm \frac{2\pi}{3}\mbox{ for any integer } n \end{align*}For instance, \(f(x)\) has critical points at \(x=\dfrac{2\pi}{3}\text{,}\) \(x=\dfrac{4\pi}{3}\text{,}\) \(x=\dfrac{8\pi}{3}\text{,}\) and \(x=\dfrac{10\pi}{3}\text{.}\)From the unit circle, or the graph of \(y=1+2\cos x\text{,}\) we see:\(x\) \(\!\left(-\frac{2\pi}{3},\frac{2\pi}{3}\right)\!\) \(\frac{2\pi}{3}\) \(\left(\frac{2\pi}{3},\frac{4\pi}{3}\right)\) \(\frac{4\pi}{3}\) \(\left(\frac{4\pi}{3},\frac{8\pi}{3}\right)\) \(\frac{8\pi}{3}\) \(\left(\frac{8\pi}{3},\frac{10\pi}{3}\right)\) \(f'(x)\) positive 0 negative 0 positive 0 negative \(f(x)\) increasing\(\!\) l. max decreasing l. min increasing l. max decreasing We have enough information to sketch a skeleton of the curve \(y=f(x)\text{.}\) We use the pattern above for the graph to the right of the \(y\)-axis, and use odd symmetry for the graph to the left of the \(y\)-axis. -
Concavity:\begin{equation*} f''(x)=-2\sin x \end{equation*}So, \(f''(x)\) exists everywhere, and is zero for \(x=\pi+\pi n\) for every integer \(n\text{.}\)
\(x\) \(\left(0,\pi\right)\) \(\pi\) \(\left(\pi,2\pi\right)\) \(2\pi\) \(\left(2\pi,3\pi\right)\) \(3\pi\!\) \(\left(3\pi,4\pi\right)\) \(f''(x)\) negative 0 positive 0 negative 0 positive \(f(x)\) concave down IP concave up IP\(\!\) concave down IP concave up Using these values, and the odd symmetry of \(f(x)\text{,}\) we can refine our skeleton. The closed dots are local extrema, and the open dots are inflection points occurring at every integer multiple of \(\pi\text{.}\)
3.6.7.9. (✳).
Solution.
- Symmetry: \(f(x)\) is periodic of period \(2\pi\text{.}\) We’ll consider only \(-\pi\le x\le \pi\text{.}\) (Any interval of length \(2\pi\) will do.)
- \(y\)-intercept: \(f(0)=-2\)
-
Intervals of increase and decrease: \(f'(x)=0\) when \(\cos x=0\text{,}\) i.e. \(x=\pm\dfrac{\pi}{2}\text{,}\) and when \(\sin x =-\dfrac{1}{2}\text{,}\) i.e. \(x=-\dfrac{\pi}{6}, -\dfrac{5\pi}{6}\text{.}\)
\(x\) \((-\pi,-\frac{5\pi}{6})\) \(\left(-\frac{5\pi}{6},-\frac{\pi}{2}\right)\) \(\left(-\frac{\pi}{2},-\frac{\pi}{6}\right)\) \(\left(-\frac{\pi}{6},\frac{\pi}{2}\right)\) \(\left(\frac{\pi}{2},\pi\right)\) \(f'(x)\) negative positive negative positive negative \(f(x)\) decreasing increasing decreasing increasing decreasing This tells us local maxima occur at \(x=\pm\dfrac{\pi}{2}\) and local minima occur at \(x=-\dfrac{5\pi}{6}\) and \(x=-\dfrac{\pi}{6}\text{.}\)Here is a table giving the value of \(f\) at each of its critical points.\(x\) \(-\dfrac{5}{6}\pi\) \(-\dfrac{\pi}{2}\) \(\dfrac{\pi}{2}\) \(\dfrac{5}{6}\pi\) \(\sin(x)\) \(-\frac{1}{2}\) \(-1\) \(-\frac{1}{2}\) \(1\) \(\cos(2x)\) \(\frac{1}{2}\) \(-1\) \(\frac{1}{2}\) \(-1\) \(f(x)\) \(-3\) \(-2\) \(-3\) \(6\) From here, we can graph a skeleton of of \(f(x)\text{:}\) -
Concavity: To find the points where \(f''(x)=0\text{,}\) set \(y=\sin x\text{,}\) so \(f''(x)=-4(4y^2+y-2)\text{.}\) Then we really need to solve\begin{align*} 4y^2 +y-2 &=0 & \text{which gives us}\\ y &= \dfrac{-1 \pm \sqrt{33}}{8} \end{align*}These two \(y\)-values map to the following two \(x\)-values, which we’ll name \(a\) and \(b\) for convenience:\begin{align*} a &= \arcsin\left(\dfrac{-1 + \sqrt{33}}{8} \right) \approx 0.635\\ b &= \arcsin\left(\dfrac{-1 - \sqrt{33}}{8} \right)\approx -1.003 \end{align*}However, these are not the only values of \(x\) in \([-\pi,\pi]\) with \(\sin x = \frac{-1\pm\sqrt{33}}{8}\text{.}\) The analysis above misses the others because the arcsine function only returns numbers in the range \(\left[-\frac{\pi}{2}, \frac{\pi}{2}\right]\text{.}\) The graph below shows that there should be other values of \(x\) with \(\sin x = \frac{-1\pm\sqrt{33}}{8}\text{,}\) and hence \(f''(x)=0\text{.}\)We can recover the other solutions in \([-\pi,\pi]\) by recalling that\begin{align*} \sin(x) &= \sin(\pi-x) \end{align*}(see CLP appendix A7). So, if we choose \(x =\arcsin\left(\frac{-1 + \sqrt{33}}{8} \right) \approx 0.635\) to make \(\sin(x)=\frac{-1 + \sqrt{33}}{8}\) so that \(f''(x) = 0\text{,}\) then setting\begin{align*} x &= \pi-a=\pi-\arcsin\left(\dfrac{-1 + \sqrt{33}}{8} \right) \approx 2.507 \end{align*}will also give us \(\sin(x)=\frac{-1 + \sqrt{33}}{8}\) and \(f''(x) = 0\text{.}\) Similarly, setting\begin{align*} x &=\pi-b= \pi-\arcsin\left(\dfrac{-1 - \sqrt{33}}{8} \right) \approx 4.145 \end{align*}would give us \(f''(x)=0\text{.}\) However, this value is outside \([-\pi,\pi]\text{.}\) To find another solution inside \([-\pi,\pi]\) we use the identity\begin{align*} \sin(x) &= \sin(-\pi-x) \end{align*}(which we can obtain from the identity we used above and the fact that \(\sin(\theta)=\sin(\theta\pm2\pi)\) for any angle \(\theta\)). Using this, we can show that\begin{align*} x &=-\pi-b= -\pi-\arcsin\left(\dfrac{-1 - \sqrt{33}}{8} \right) \approx -2.139 \end{align*}also gives \(f''(x)=0\text{.}\)So, all together, \(f''(x)=0\) when \(x=-\pi-b\text{,}\) \(x=b\text{,}\) \(x=a\text{,}\) and \(x=\pi-a\text{.}\)Now, we should compute the sign of \(f''(x)\) while \(x\) is between \(-\pi\) and \(\pi\text{.}\) Recall that, if \(y=\sin x\text{,}\) then \(f''(x)=-4(4y^2+y-2)\text{.}\) So, in terms of \(y\text{,}\) \(f''\) is a parabola pointing down, with intercepts \(y=\frac{-1\pm\sqrt{33}}{8}\text{.}\) Then \(f''\) is positive when \(y\) is in the interval \(\left(\frac{-1-\sqrt{33}}{8},\frac{-1+\sqrt{33}}{8}\right)\text{,}\) and \(f''\) is negative otherwise. From the graph of sine, we see that \(y\) is between \(\frac{-1-\sqrt{33}}{8}\) and \(\frac{-1+\sqrt{33}}{8}\) precisely on the intervals \((-\pi,-\pi-b)\text{,}\) \((b,a)\text{,}\) and \((\pi-a,\pi)\text{.}\)Therefore, \(f(x)\) is concave up on the intervals \((-\pi,-\pi-b)\text{,}\) \((b,a)\text{,}\) and \((\pi-a,\pi)\text{,}\) and \(f(x)\) is concave down on the intervals \((-\pi-b,b)\) and \((a,\pi-a)\text{.}\) So, the inflection points of \(f\) occur at \(x=-\pi-b\text{,}\) \(x=b\text{,}\) \(x=a\text{,}\) and \(x=\pi-a\text{.}\)
3.6.7.10.
Solution.
-
Asymptotes: Since \(\ds\lim_{x \to 0} f(x) = \infty\text{,}\) \(f(x)\) has a vertical asymptote at \(x=0\) where the curve reaches steeply upward from both the left and the right.\(\ds\lim_{x \to \pm \infty} f(x)=0\text{,}\) so \(y=0\) is a horizontal asymptote for \(x \to \pm \infty\text{.}\)
- Intercepts: \(f(-1)=0\text{.}\)
-
Intervals of increase and decrease:\begin{equation*} f'(x)=\dfrac{-(x+2)}{3x^{5/3}(x+1)^{2/3}} \end{equation*}There is a singular point at \(x=-1\) and a critical point at \(x=-2\text{,}\) in addition to a discontinuity at \(x=0\text{.}\) Note that \((x+1)^{2/3}=\left(\sqrt[3]{x+1}\right)^2\text{,}\) which is never negative. Note also that \(\ds\lim_{x \to -1}f'(x)=\infty\text{,}\) so \(f(x)\) has a vertical tangent line at \(x=-1\text{.}\)
\(x\) \((-\infty,-2)\) \(-2\) \((-2,-1)\) \(-1\) \((-1,0)\) 0 \((0,\infty)\) \(f'(x)\) negative 0 positive DNE positive DNE negative \(f(x)\) decreasing l. min increasing vertical increasing VA decreasing This gives us enough information to sketch a skeleton of the curve. -
Concavity:\begin{equation*} f''(x)=\dfrac{4x^2+16x+10}{9x^{8/3}(x+1)^{5/3}} \end{equation*}We still have a discontinuity at \(x=0\text{,}\) and \(f''(x)\) does not exist at \(x=-1\text{.}\) The second derivative is zero when \(4x^2+16x+10=0\text{.}\) Using the quadratic formula, we find this occurs when \(x=-2\pm\sqrt{1.5} \approx -0.8,-3.2\text{.}\) Note \(x^{8/3}=\left(\sqrt[3]{x}\right)^8\) is never negative.
\(x\) \(\left(-\infty,-2-\sqrt{1.5}\right)\) \(-2-\sqrt{1.5}\) \((-2-\sqrt{1.5},-1)\) \(-1\) \(f''(x)\) negative 0 positive DNE \(f(x)\) concave down IP concave up IP \(x\) \((-1,-2+\sqrt{1.5})\) \(-2+\sqrt{1.5}\) \((-2+\sqrt{1.5},0)\) \((0,\infty)\) \(f''(x)\) negative 0 positive positive \(f(x)\) concave down IP concave up concave up Now, we can refine our skeleton. The closed dot is the local minimum, and the open dots are inflection points.
3.6.7.11. (✳).
Solution.
-
Asymptotes:Since the function has a derivative at every real number, the function is continuous for every real number, so it has no vertical asymptotes. In the problem statement, you are told \(\ds\lim_{x \to \infty} f(x)=0\text{,}\) so \(y=0\) is a horizontal asymptote as \(x\) goes to infinity. It remains to evaluate \(\ds\lim_{x \to -\infty} f(x)\text{.}\) Let’s consider the limit of \(f'(x)\) instead. Recall \(K\) is a positive constant.\begin{align*} &\lim_{x \to -\infty}e^{-x}=\lim_{x \to \infty}e^x=\infty\\ &\lim_{x \to -\infty}K(2x-x^2)=-\infty\\ \end{align*}
So,
\begin{align*} &\lim_{x \to -\infty} K(2x-x^2)e^{-x}=-\infty \end{align*}That is, as \(x\) becomes a hugely negative number, \(f'(x)\) also becomes a hugely negative number. As we move left along the \(x\)-axis, \(f(x)\) is decreasing with a steeper and steeper slope, as in the sketch below. That means \(\ds\lim_{x \to -\infty}f(x)=\infty\text{.}\) -
Intervals of increase and decrease:We are given \(f'(x)\) (although we don’t know \(f(x)\)):\begin{equation*} f'(x)=Kx(2-x)e^{-x} \end{equation*}The critical points of \(f(x)\) are \(x=0\) and \(x=2\text{,}\) and there are no singular points. Recall \(e^{-x}\) is always positive, and \(K\) is a positive constant.
\(x\) \((-\infty,0)\) 0 \((0,2)\) 2 \((2,\infty)\) \(f'(x)\) negative 0 positive 0 negative \(f(x)\) decreasing local min increasing local max decreasing So, \(f(0)=0\) is a local minimum, and \(f(2)=2\) is a local maximum.Looking ahead to part 3.6.7.11.d, we have a skeleton of the curve. -
Concavity:Since we’re given \(f'(x)\text{,}\) we can find \(f''(x)\text{.}\)\begin{align*} f''(x)&=K(2-2x-2x+x^2)e^{-x}\\ &=K(2-4x+x^2)e^{-x}\\ &=K\big(x-2-\sqrt{2}\big)\big(x-2+\sqrt{2}\big)e^{-x} \end{align*}where the last line can be found using the quadratic equation. So, \(f''(x)=0\) for \(x=2\pm\sqrt{2}\text{,}\) and \(f''(x)\) exists everywhere.
\(x\) \((-\infty,2-\sqrt{2})\) \(2-\sqrt{2}\) \((2-\sqrt{2},2+\sqrt{2})\) \(2+\sqrt 2\) \((2+\sqrt 2,\infty)\) \(f'(x)\) positive 0 negative 0 positive \(f(x)\) concave up IP concave down IP concave up Now, we can add concavity to our sketch.
3.6.7.12. (✳).
Solution.
\((y,x)\) is on the graph of \(g\) if and only if \((x,y)\) is on the graph of \(f\text{.}\)
Using the chain rule,
\begin{align*} g'(f(x))\cdot f'(x)&=1\\ \end{align*}Since \(f'(x)=-e^{-x}=-f(x)\text{:}\)
\begin{align*} g'(f(x))\cdot f(x)&=-1\\ \end{align*}We plug in \(f(x)=\frac{1}{2}\text{.}\)
\begin{align*} g'\left(\frac{1}{2}\right)\cdot \frac{1}{2}&=-1\\ g'\left(\frac{1}{2}\right)&=-2 \end{align*}3.6.7.13. (✳).
Solution.
- \(\ds\lim_{x \to \infty} f(x)=\infty\text{,}\) \(\ds\lim_{x \to -\infty} f(x)=-\infty\)
- \(f'(x) \gt 0\) (i.e. \(f\) is increasing) for \(|x| \gt \dfrac{1}{\root{4}\of{5}}\)
- \(f'(x)=0\) (i.e. \(f\) has critical points) for \(x= \pm\dfrac{1}{\root{4}\of{5}} \approx \pm 0.67\)
- \(f'(x) \lt 0\) (i.e. \(f\) is decreasing) for \(|x| \lt \dfrac{1}{\root{4}\of{5}}\)
- \(\displaystyle f\left(\pm\dfrac{1}{\sqrt[4]{5}}\right)=\mp\dfrac{4}{5\sqrt[4]{5}}\approx\mp0.53\)
- \(f''(x) \gt 0\) (i.e. \(f\) is concave up) for \(x \gt 0\text{,}\)
- \(f''(x)=0\) (i.e. \(f\) has an inflection point) for \(x=0\text{,}\) and
- \(f''(x) \lt 0\) (i.e. \(f\) is concave down) for \(x \lt 0\)
- \(f\) has no asymptotes
- \(f\) has a local maximum at \(x=-\dfrac{1}{\root{4}\of{5}}\) and a local minimum at \(x=\dfrac{1}{\root{4}\of{5}}\)
- \(f\) has an inflection point at \(x=0\)
- \(f\) is concave down for \(x \lt 0\) and concave up for \(x \gt 0\)
- 1 when \(|k| \gt \dfrac{4}{5\root{4}\of{5}}\)
- 2 when \(|k|=\dfrac{4}{5\root{4}\of{5}}\)
- 3 when \(|k| \lt \dfrac{4}{5\root{4}\of{5}}\)
3.6.7.14. (✳).
Solution.
- \(\sinh(x)\) has a derivative that is always positive, so \(\sinh(x)\) is always increasing. The second derivative of \(\sinh(x)\) is negative to the left of \(x=0\) and positive to the right of \(x=0\text{,}\) so \(\sinh(x)\) is concave down to the left of the \(y\)-axis and concave up to its right, with an inflection point at \(x=0\text{.}\)
- \(\cosh(x)\) has a derivative that is positive when \(x \gt 0\) and negative when \(x \lt 0\text{.}\) The second derivative of \(\cosh(x)\) is always positive, so it is always concave up.
- \(\cosh(0)=1\) and \(\sinh(0)=0\text{.}\)
- \(\ds\lim_{x\rightarrow\infty}\sinh x = \ds\lim_{x\rightarrow\infty}\cosh x =\ds\lim_{x\rightarrow\infty}\frac{e^x}{2}=\infty\text{,}\) since \(\ds\lim_{x \to \infty}e^{-x}=0\)
- \begin{equation*} \ds\lim_{x \to -\infty}\sinh x =\ds\lim_{x \to -\infty}\left(\frac{e^x}{2}-\frac{e^{-x}}{2}\right) =\ds\lim_{x \to \infty}\left(\frac{e^{-x}}{2}-\frac{e^x}{2}\right)=-\infty \end{equation*}and\begin{equation*} \ds\lim_{x \to -\infty}\cosh x =\ds\lim_{x \to -\infty}\left(\frac{e^x}{2}+\frac{e^{-x}}{2}\right) =\ds\lim_{x \to \infty}\left(\frac{e^{-x}}{2}+\frac{e^x}{2}\right)=\infty \end{equation*}
- \(\cosh(x)\) is even, since\begin{equation*} \cosh(-x)=\dfrac{e^{-x}+e^{-(-x)}}{2}=\dfrac{e^{-x}+e^x}{2}=\cosh (x) \end{equation*}and \(\sinh (x)\) is odd, since\begin{align*} \sinh(-x)\amp=\dfrac{e^{-x}-e^{-(-x)}}{2}=\dfrac{e^{-x}-e^x}{2} =\dfrac{-\left(e^x-e^{-x}\right)}{2}\\ \amp=-\sinh(x) \end{align*}
- As \(y\) runs over \((-\infty,\infty)\) the function \(\sinh(y)\) takes every real value exactly once. So, for each \(x\in(-\infty,\infty)\text{,}\) define \(\sinh^{-1}(x)\) to be the unique solution of \(\sinh(y)=x\text{.}\)
- As \(y\) runs over \([0,\infty)\) the function \(\cosh(y)\) takes every real value in \([1,\infty)\) exactly once. In particular, the smallest value of \(\cosh(y)\) is \(\cosh(0)=1\text{.}\) So, for each \(x\in[1,\infty)\text{,}\) define \(\cosh^{-1}(x)\) to be the unique \(y\in[0,\infty)\) that obeys \(\cosh(y)=x\text{.}\)
We differentiate with respect to \(x\) using the chain rule.
\begin{align*} \diff{}{x}\left\{\cosh y(x)\right\}&=\diff{}{x}\{x\}\\ y'(x) \sinh y(x)&=1\\ \end{align*}We solve for \(y'(x)\text{.}\)
\begin{align*} y'(x) &=\dfrac{1}{\sinh y(x)}\\ \end{align*}We want to have our answer in terms of \(x\text{,}\) not \(y\text{.}\) We know that \(\cosh y=x\text{,}\) so if we can convert hyperbolic sine into hyperbolic cosine, we can get rid of \(y\text{.}\) Our tool for this is the identity, given in the question statement, \(\cosh^2(x)-\sinh^2(x)=1\text{.}\) This tells us \(\sinh^2(y)=1-\cosh^2(y)\text{.}\) Now we have to decide whether \(\sinh(y)\) is the positive or negative square root of \(1-\cosh^2(y)\) in our context. Looking at the graph of \(y(x)=\cosh^{-1}(x)\text{,}\) we see \(y'(x) \gt 0\text{.}\) So we use the positive square root:
\begin{align*} y'(x) & =\dfrac{1}{\sqrt{\cosh^2 y(x)-1}} =\dfrac{1}{\sqrt{x^2-1}} \end{align*}
3.7 L’Hôpital’s Rule, Indeterminate Forms
3.7.4 Exercises
3.7.4.1.
Solution.
3.7.4.2.
Solution.
3.7.4.3.
Solution.
- 3.7.4.3.i \(\ds\lim_{x \to \infty} f(x)=\ds\lim_{x \to \infty } \left(1+\frac{1}{x}\right)=1\)
- 3.7.4.3.ii \(\ds\lim_{x \to \infty} g(x)=\ds\lim_{x \to \infty} x\log 5=\infty\)
- 3.7.4.3.iii Let us name \(\dfrac{1}{x}=X\text{.}\) Then as \(x \to \infty\text{,}\) \(X \to 0^+\text{,}\) so:\begin{align*} \lim_{x \to \infty} [f(x)]^{g(x)} \amp=\lim_{x \to \infty} \left[1+\frac{1}{x}\right]^{x\log 5} =\lim_{x \to \infty} \left[1+\frac{1}{x}\right]^{\frac{\log 5}{\frac{1}{x}}}\\ \amp=\lim_{X \to 0^+} \left[1+X\right]^{\frac{\log 5}{X}} =e^{\log 5}=5\text{,} \end{align*}where in the penultimate step, we used the result of Example 3.7.20.
3.7.4.4. (✳).
Solution.
3.7.4.5. (✳).
Solution.
3.7.4.6. (✳).
Solution.
3.7.4.7. (✳).
Solution.
3.7.4.8. (✳).
Solution.
3.7.4.9.
Solution.
3.7.4.10. (✳).
Solution.
3.7.4.11. (✳).
Solution.
3.7.4.12.
Solution.
3.7.4.13.
Solution.
Since plugging in \(x=0\) makes both the numerator and the denominator equal to zero, this is a candidate for l’Hôspital’s Rule. However, a much easier way is to simplify the trig first.
\begin{align*} \lim_{x\to0}\frac{\tan x\cdot (x^2+5)}{\sin x \cdot e^x}&= \lim_{x\to0}\frac{\sin x\cdot (x^2+5)}{\cos x \cdot\sin x \cdot e^x}\\ &= \lim_{x\to0}\frac{x^2+5}{\cos x \cdot e^x}\\ &=\frac{0^2+5}{\cos(0)\cdot e^0}=5 \end{align*}3.7.4.14.
Solution.
Here, we have the indeterminate form \(\frac{\infty}{\infty}\text{,}\) so l’Hôpital’s Rule applies. However, if we try to use it here, we quickly get a huge mess. Instead, remember how we dealt with these kinds of limits in the past: factor out the highest power of the denominator, which is \(x\text{.}\)
\begin{align*} &=\lim_{x \to \infty} \frac{x\left(x-1+\frac{1}{x}\right)}{\sqrt{x^2(2+\frac{1}{x^2})}+\sqrt{x^2(1+\frac{1}{x})}}\\ &=\lim_{x \to \infty} \frac{x\left(x-1+\frac{1}{x}\right)}{x\left(\sqrt{2+\frac{1}{x^2}}+\sqrt{1+\frac{1}{x}}\right)}\\ &=\lim_{x \to \infty} \underbrace{\frac{x-1+\frac{1}{x}}{\sqrt{2+\frac{1}{x^2}}+\sqrt{1+\frac{1}{x}}}}_{\atp {\mathrm{num}\to \infty} {\mathrm{den}\to \sqrt{2}+1}}\\ &=\infty \end{align*}3.7.4.15. (✳).
Solution.
3.7.4.16. (✳).
Solution.
3.7.4.17. (✳).
Solution.
- Solution 1.\begin{align*} \lim_{x\rightarrow 0}\frac{e^{-1/x^2}}{x^4} \amp=\lim_{x\rightarrow 0}\underbrace{\frac{\frac{1}{x^4}}{e^{1/x^2}}}_{\atp {\mathrm{num}\to\infty} {\mathrm{den}\to\infty}} =\lim_{x\rightarrow 0}\frac{\frac{-4}{x^5}}{\frac{-2}{x^3}e^{1/x^2}} =\lim_{x\rightarrow 0}\underbrace{\frac{\frac{2}{x^2}}{e^{1/x^2}}}_{\atp {\mathrm{num}\to\infty} {\mathrm{den}\to\infty}}\\ \amp=\lim_{x\rightarrow 0}\frac{\frac{-4}{x^3}}{\frac{-2}{x^3}e^{1/x^2}} =\lim_{x\rightarrow 0}\frac{2}{e^{1/x^2}} =0 \end{align*}since, as \(x\rightarrow 0\text{,}\) the exponent \(\frac{1}{x^2}\rightarrow\infty\) so that \(e^{1/x^2}\rightarrow\infty\) and \(e^{-1/x^2}\rightarrow 0\text{.}\)
- Solution 2.\begin{equation*} \lim_{x\rightarrow 0}\frac{e^{-1/x^2}}{x^4} =\lim_{t=\frac{1}{x^2}\rightarrow \infty}\frac{e^{-t}}{t^{-2}} =\lim_{t\rightarrow \infty}\underbrace{\frac{t^2}{e^{t}}}_{\atp {\mathrm{num}\to\infty} {\mathrm{den}\to\infty}} =\lim_{t\rightarrow \infty}\underbrace{\frac{2t}{e^{t}}}_{\atp {\mathrm{num}\to\infty} {\mathrm{den}\to\infty}} =\lim_{t\rightarrow \infty}\frac{2}{e^{t}} ={0} \end{equation*}
3.7.4.18. (✳).
Solution.
3.7.4.19.
Solution.
3.7.4.20.
Solution.
3.7.4.21.
Solution.
- Solution 1\begin{align*} y:&= e^{x \log x} = (e^x)^{\log x}\\ \lim_{x \to 0^+} y &=\lim_{x \to 0^+} (e^x)^{\log x} \end{align*}This has the form \(1^{-\infty} = \frac{1}{1^\infty}\text{,}\) and \(1^{\infty}\) is an indeterminate form. We want to use l’Hôpital, but we need to get a different indeterminate form. So, we’ll use logarithms.\begin{align*} \lim_{x \to 0^+} \log y &=\lim_{x \to 0 ^+} \log\left((e^x)^{\log x} \right) =\lim_{x \to 0 ^+} \log x \log\left(e^x \right)\\ \amp=\lim_{x \to 0 ^+} (\log x)\cdot x \end{align*}This has the indeterminate form \(0 \cdot \infty\text{,}\) so we need one last adjustment before we can use l’Hôpital’s Rule.\begin{align*} \lim_{x \to 0 ^+} (\log x)\cdot x &=\lim_{x \to 0 ^+} \underbrace{\frac{\log x}{\frac{1}{x}}}_{\atp {\mathrm{num} \to -\infty} {\mathrm{den} \to \infty}} =\lim_{x \to 0 ^+}\frac{\frac{1}{x}}{\frac{-1}{x^2}} =\lim_{x \to 0 ^+}-x=0 \end{align*}Now, we can figure out what happens to our original function, \(y\text{:}\)\begin{align*} \lim_{x \to 0^+} y &=\lim_{x \to 0^+} e^{\log y} = e^0=1 \end{align*}
- Solution 2\begin{align*} y:&=e^{x\log x}=\left(e^{\log x}\right)^x=x^x\\ \lim_{x \to 0^+} y &=\lim_{x \to 0^+}x^x\\ \end{align*}
We have the indeterminate form \(0^0\text{.}\) We want to use l’Hôpital, but we need a different indeterminate form. So, we’ll use logarithms.
\begin{align*} \lim_{x \to 0^+}\log y &=\lim_{x \to 0^+}\log(x^x)=\lim_{x \to 0^+}x\log x\\ \end{align*}Now we have the indeterminate form \(0 \cdot \infty\text{,}\) so we need one last adjustment before we can use l’Hôpital’s Rule.
\begin{align*} \lim_{x \to 0^+} y &=\lim_{x \to 0^+}\underbrace{\frac{\log x}{\frac{1}{x}}}_{\atp {\mathrm{num}\to 0} {\mathrm{den} \to -\infty}} =\lim_{x \to 0^+}\frac{\frac{1}{x}}{\frac{-1}{x^2}} =\lim_{x \to 0^+}-x=0\\ \end{align*}Now, we can figure out what happens to our original function, \(y\text{:}\)
\begin{align*} \lim_{x \to 0^+} y &=\lim_{x \to 0^+} e^{\log y} = e^0=1 \end{align*}
3.7.4.22.
Solution.
Now, we’re ready to figure out our original limit.
\begin{align*} \lim_{x\to 0} y &= \lim_{x \to 0} e^{\log y}=e^0=1 \end{align*}3.7.4.23. (✳).
Solution.
3.7.4.24. (✳).
Solution.
3.7.4.25.
Solution.
-
We want to find the limit as \(n\) goes to infinity of the percentage error, \(\ds\lim_{n \rightarrow \infty} 100\frac{|S(n)-A(n)|}{|S(n)|}\text{.}\) Since \(A(n)\) is a nicer function than \(S(n)\text{,}\) let’s simplify: \(\ds\lim_{n \rightarrow \infty} 100\frac{|S(n)-A(n)|}{|S(n)|} = 100\left|1-\ds\lim_{n \to \infty}\frac{A(n)}{S(n)}\right|\text{.}\)We figure out this limit the natural way:\begin{align*} 100\left|1-\ds\lim_{n \to \infty}\frac{A(n)}{S(n)}\right|&= 100\left|1-\ds\lim_{n \rightarrow \infty}\underbrace{\frac{5n^4}{5n^4-13n^3-4n+\log (n)}}_{\atp {\mathrm{num}\to\infty} {\mathrm{den}\to\infty}}\right|\\ &= 100\left|1-\ds\lim_{n \rightarrow \infty}\frac{20n^3}{20n^3-39n^2-4+\frac{1}{n}}\right|\\ &= 100\left|1-\ds\lim_{n \rightarrow \infty}\frac{n^3}{n^3}\cdot\frac{20}{20-\frac{39}{n}-\frac{4}{n^3}+\frac{1}{n^4}}\right|\\ &=100|1-1|=0 \end{align*}So, as \(n\) gets larger and larger, the relative error in the approximation gets closer and closer to 0.
- Now, let’s look at the absolute error.\begin{align*} \lim_{n \rightarrow \infty} \left| S(n)-A(n)\right|&=\lim_{n \rightarrow \infty} |-13n^3-4n+\log n|=\infty \end{align*}So although the error gets small relative to the giant numbers we’re talking about, the absolute error grows without bound.
4 Towards Integral Calculus
4.1 Introduction to Antiderivatives
4.1.2 Exercises
4.1.2.1.
Solution.
4.1.2.2.
Solution.
4.1.2.3.
Solution.
4.1.2.4.
Solution.
4.1.2.5.
Solution.
4.1.2.6.
Solution.
- Solution 1: We can re-write \(f(x)\) to make it a power of \(x\text{.}\)\begin{align*} F'(x)&=\frac{1}{7}x^{-\tfrac{1}{2}}\\ F(x)&=\left(\frac{1}{7}\right)\left(\frac{1}{-\frac{1}{2}+1}\right)x^{\left(-\tfrac{1}{2}+1\right)}+C\\ &=\left(\frac{1}{7}\right)(2)x^{\tfrac{1}{2}}+C\\ &=\frac{2}{7}\sqrt{x}+C \end{align*}
- Solution 2: We notice that \(\dfrac{1}{7\sqrt{x}}\) looks a lot like \(\dfrac{1}{2\sqrt{x}}\text{,}\) which is the derivative of \(\sqrt{x}\text{.}\) So:\begin{align*} \diff{}{x}\left\{\sqrt{x}\right\}&=\frac{1}{2\sqrt{x}}\\ \diff{}{x}\left\{\frac{2}{7}\sqrt{x}\right\}&=\left(\frac{2}{7}\right)\frac{1}{2\sqrt{x}}=f(x) \end{align*}So, an antiderivative of \(f(x)\) is \(\dfrac{2}{7}\sqrt{x}\text{.}\) Then the most general antiderivative is \(F(x)=\dfrac{2}{7}\sqrt{x}+C\text{.}\)
4.1.2.7.
Solution.
This isn’t exactly right, so we modify it by multiplying by a constant.
\begin{align*} \diff{}{x}\left\{\frac{1}{5}e^{5x+11}\right\}&=e^{5x+11} \end{align*}4.1.2.8.
Solution.
So, an antiderivative of \(7\cos(13x)\) is \(\dfrac{7}{13}\sin(13x)\text{.}\)
\begin{align*} \diff{}{x}\{\cos x\}&=-\sin x\\ \diff{}{x}\{-\cos x\}&=\sin x\\ \diff{}{x}\{-\cos (5x)\}&=5\sin (5x)\\ \diff{}{x}\left\{-\frac{3}{5}\cos (5x)\right\}&=\left(\frac{3}{5}\right)5\sin (5x)=3\sin(5x) \end{align*}4.1.2.9.
Solution.
4.1.2.10.
Solution.
4.1.2.11.
Solution.
4.1.2.12.
Solution.
This gives us a first guess for our antiderivative: perhaps \(\arctan(5x)\) will work. We test it by differentiating, making sure we don’t forget the chain rule.
\begin{align*} \diff{}{x}\left\{\arctan(\textcolor{red}{5x})\right\}&=\frac{1}{1+\left(\textcolor{red}{5x}\right)^2}\cdot\textcolor{red}{5}\\ \end{align*}We’re close to \(f(x)\text{,}\) but we’ve multiplied by 5. That’s easy to take care of: we can divide our guess by 5.
\begin{align*} \diff{}{x}\left\{\frac{1}{5}\arctan(\textcolor{red}{5x})\right\}&= \frac{1}{5}\left(\frac{1}{1+\left(\textcolor{red}{5x}\right)^2}\right)\cdot\textcolor{red}{5}\\ &=\frac{1}{1+25x^2}=f(x) \end{align*}4.1.2.13.
Solution.
Use the fact that \(f(1)=10\) to solve for \(C\text{.}\)
\begin{align*} 10&=1-\frac{9}{2}+4+C\\ C&=\frac{19}{2}\\ \end{align*}All together,
\begin{align*} f(x)&=x^3-\frac{9}{2}x^2+4x+\frac{19}{2}. \end{align*}4.1.2.14.
Solution.
This is close to \(f'(x)\text{,}\) but we need to divide by 2.
\begin{align*} \diff{}{x}\left\{\frac{1}{2}\sin(2x)\right\}&=\cos(2x)\\ \end{align*}So, \(f(x)=\dfrac{1}{2}\sin(2x)+C\) for some constant \(C\text{.}\) We can find \(C\) using the given information \(f(\pi)=\pi\text{.}\)
\begin{align*} \pi=f(\pi)&=\frac{1}{2}\sin(2\pi)+C\\ \pi&=C \end{align*}4.1.2.15.
Solution.
4.1.2.16.
Solution.
4.1.2.17.
Solution.
4.1.2.18.
Solution.
Antidifferentiating,
\begin{align*} A(t)&=75000e^{\tfrac{t}{50}}+C \end{align*}4.1.2.19.
Solution.
Antidifferentiating,
\begin{align*} P(t)&=-\frac{12}{\pi}\cos\left(\frac{\pi}{24}t\right)+0.25t+C\\ \end{align*}Since \(P(0)\) is the amount of energy consumed after 0 hours, \(P(0)=0\text{,}\) so
\begin{align*} 0=P(0)&=-\frac{12}{\pi}+C\\ C&=\frac{12}{\pi}\\ P(t)&=\frac{12}{\pi}\left[1-\cos\left(\frac{\pi}{24}t\right)\right]+0.25t\\ \end{align*}After 24 hours, your energy consumed is
\begin{align*} P(24)&=\frac{12}{\pi}\left[1-\cos\left(\pi\right)\right]+0.25(24)\\ &=\frac{24}{\pi}+6\approx13.6\; \mathrm{kWh} \end{align*}4.1.2.20. (✳).
Solution.
4.1.2.21.
Solution.
4.1.2.22.
Solution.
Then, since the denominator of \(f(x)\) is \((x^2+1)^2\text{,}\) we might guess \(v(x)=(x^2+1)\text{.}\) That leaves \(u(x)=e^x\text{.}\)
\begin{align*} \diff{}{x}\left\{\frac{e^x}{x^2+1}\right\}&=\frac{(x^2+1)e^x-e^x(2x)}{(x^2+1)^2}=f(x) \end{align*}4.1.2.23.
Solution.
4.1.2.24.
Solution.
This differs from \(f(x)\) only by a constant multiple.
\begin{align*} \diff{}{x}\left\{-\frac{5}{2}\cos\left(x^2\right)\right\}&=\frac{5}{2}\sin\left(x^2\right)\cdot2x=5x\sin(x^2)=f(x) \end{align*}4.1.2.25.
Solution.
4.1.2.26.
Solution.
What we really want under that square root, instead of \(\dfrac{x^2}{3}\text{,}\) is simply \(x^2\text{.}\) We can get close: we can get something squared.
\begin{align*} f(x)&=\frac{7}{\sqrt{3}}\left(\frac{1}{\sqrt{1-\left(\frac{x}{\sqrt{3}}\right)^2}}\right)\\ \end{align*}Now, the thing that’s squared isn’t \(x\text{,}\) it’s \(\dfrac{x}{\sqrt{3}}\text{.}\) This gives us a first guess for an antiderivative: perhaps \(F(x)=\dfrac{7}{\sqrt{3}}\arcsin\left(\dfrac{x}{\sqrt{3}}\right)\) will work. Let’s try it! Remember to use the chain rule when you differentiate.
\begin{align*} \diff{}{x}\left\{\frac{7}{\sqrt{3}}\arcsin\left(\textcolor{red}{\frac{x}{\sqrt{3}}}\right)\right\}&= \frac{7}{\sqrt{3}}\left(\frac{1}{\sqrt{1-\left(\textcolor{red}{\frac{x}{\sqrt{3}}}\right)^2}}\right)\cdot\textcolor{red}{\frac{1}{\sqrt{3}}}\\ \end{align*}We’re very close! We’re only off by a constant, and those are easy to fix. We’re dividing by \(\sqrt{3}\) when we differentiate, so let’s multiply our function by \(\sqrt{3}\text{.}\)
\begin{align*} \diff{}{x}\left\{7\arcsin\left(\textcolor{red}{\frac{x}{\sqrt{3}}}\right)\right\}&= 7\left(\frac{1}{\sqrt{1-\left(\textcolor{red}{\frac{x}{\sqrt{3}}}\right)^2}}\right)\cdot\textcolor{red}{\frac{1}{\sqrt{3}}}\\ &=\frac{7}{\sqrt{3}\sqrt{1-\left(\frac{x}{\sqrt{3}}\right)^2}}\\ &=\frac{7}{\sqrt{3-x^2}}=f(x) \end{align*}4.1.2.27.
Solution.
When \(H=0\text{,}\) there is no solid, so \(V(0)=0\text{.}\) Therefore,
\begin{align*} V(H)&=2\pi\left(\frac{1}{5}H^5+\frac{2}{3}H^3+H\right). \end{align*}